Sei sulla pagina 1di 118

Chapter 1.

Smooth Manifolds
Theorem 1. [Exercise 1.18] Let M be a topological manifold. Then any two smooth
atlases for M determine the same smooth structure if and only if their union is a smooth
atlas.
Proof. Suppose A1 and A2 are two smooth atlases for M that determine the same
smooth structure A. Then A1 , A2 A, so A1 A2 must be a smooth atlas since every
chart in A1 is compatible with every chart in A2 . Conversely, if A1 A2 is a smooth
atlas then the smooth structures determined by A1 and A2 both contain A1 A2 . But
there is exactly one smooth structure containing A1 A2 , so A1 and A2 determine the
same smooth structure.

Theorem 2. [Exercise 1.44] Let M be a smooth n-manifold with boundary and let U
be an open subset of M .
(1) U is a topological n-manifold with boundary, and the atlas consisting of all
smooth charts (V, ) for M such that V U defines a smooth structure on
U . With this topology and smooth structure, U is called an open submanifold
with boundary.
(2) If U Int M , then U is actually a smooth manifold (without boundary); in this
case we call it an open submanifold of M .
(3) Int M is an open submanifold of M (without boundary).
Proof. Parts (1) and (2) are obvious. Part (3) follows from (2) and the fact that Int M
is an open subset of M .

Theorem 3. [Problem 1-6] Let M be a nonempty topological manifold of dimension
n 1. If M has a smooth structure, then it has uncountably many distinct ones.
Proof. We use the fact that for any s > 0, Fs (x) = |x|s1 x defines a homeomorphism
from Bn to itself, which is a diffeomorphism if and only if s = 1. Let A be a smooth
structure for M and choose some coordinate map : U Bn centered at some p U .
Let A0 be the smooth atlas obtained by replacing every coordinate map : V R in
A with 0 : V (M \ {p}) R, except when = . For any s > 0, let As be the
smooth atlas obtained from A0 by replacing with Fs . Since Fs is a diffeomorphism
if and only if s = 1, the smooth structures determined by As and At are the same if and
only if s = t. This shows that there are uncountably many distinct smooth structures
on M .

Theorem 4. [Problem 1-8] An angle function on a subset U S1 is a continuous
function : U R such that ei(z) = z for all z U . There exists an angle function
on an open subset U S1 if and only if U 6= S1 . For any such angle function, (U, ) is
a smooth coordinate chart for S1 with its standard smooth structure.
1

Proof. The first part follows from the lifting criterion (Proposition A.78). By rotating
R2 appropriately, we may assume that N = (0, 1)
/ U . Let : S1 \ {N } R be the
1
2
1
stereographic projection given by (x , x ) = x /(1 x2 ). We can compute
cos
( 1 )() =
,
1 sin
which is a diffeomorphism on (U ).

Theorem 5. [Problem 1-10] Let k and n be integers satisfying 0 < k < n, and let
P, Q Rn be the linear subspaces spanned by (e1 , . . . , ek ) and (ek+1 , . . . , en ), respectively, where ei is the ith standard basis vector for Rn . For any k-dimensional subspace
S Rn that has trivial intersection with Q, the coordinate representation (S) constructed in Example 1.36 istheunique (n k) k matrix B such that S is spanned by
Ik
the columns of the matrix
, where Ik denotes the k k identity matrix.
B
Proof. Let B = {e1 , . . . , ek }. The matrix of (S) represents the linear map (Q |S )
(P |S )1 . Since P |S is an isomorphism, the vectors (P |S )1 (B) form a basis for S.
But then
(IdRn |S (P |S )1 )(B) = ((P |S + Q |S ) (P |S )1 )(B)
= (IdP +Q |S (P |S )1 )(B)
is a basis for S. This is precisely the result desired.

Theorem 6. [Problem 1-12] Suppose M1 , . . . , Mk are smooth manifolds and N is a


smooth manifold with boundary. Then M1 Mk N is a smooth manifold with
boundary, and (M1 Mk N ) = M1 Mk N .
Proof. Denote the dimensions of M1 , . . . , Mk , N by n1 , . . . , nk , d. Let p = (p1 , . . . , pk , q)
M1 Mk N , let i : Mi Rni be coordinate maps around pi for i = 1, . . . , k,
and let : N Hd be a coordinate map around q. Then 1 k : M1
Mk N Hn1 ++nk +d is a coordinate map, and (1 k )(p1 , . . . , pk , q)
Hn1 ++nk +d if and only if (q) Hd , which is true if and only if q N .

Chapter 2. Smooth Maps
Theorem 7. [Exercise 2.1] Let M be a smooth manifold with or without boundary.
Then pointwise multiplication turns C (M ) into a commutative ring and a commutative
and associative algebra over R.
Proof. The product of two smooth functions is also smooth.

Theorem 8. [Exercise 2.2] Let U be an open submanifold of Rn with its standard


smooth manifold structure. Then a function f : U Rk is smooth in the sense of
smooth manifolds if and only if it is smooth in the sense of ordinary calculus.
Proof. Obvious since the single chart IdRn covers Rn .

Theorem 9. [Exercise 2.3] Let M be a smooth manifold with or without boundary, and
suppose f : M Rk is a smooth function. Then f 1 : (U ) Rk is smooth for
every smooth chart (U, ) for M .
Proof. If p U then there is a smooth chart (V, ) for M such that f 1 is smooth
and p V . But on (U V ) we have
f 1 = f 1 1 ,
which is smooth since 1 is smooth. This shows that f 1 is smooth in a
neighborhood of every point in (U ), so f 1 is smooth.

Theorem 10. [Exercise 2.7(1)] Suppose M and N are smooth manifolds with or without
boundary, and F : M N is a map. Then F is smooth if and only if either of the
following conditions is satisfied:
(1) For every p M , there exist smooth charts (U, ) containing p and (V, )
containing F (p) such that U F 1 (V ) is open in M and the composite map
F 1 is smooth from (U F 1 (V )) to (V ).
(2) F is continuous and there exist smooth atlases {(U , )} and {(V , )} for M
and N , respectively, such that for each and , F 1
is a smooth map
1
from (U F (V )) to (V ).
Proof. (1) is obvious, as is (2) (1). If (1) holds then F is continuous by Proposition
2.4, and we can choose the smooth atlases to be the smooth structures for M and N .
For every and the map F 1
is smooth since for all p M we can find
smooth coordinate maps containing p and containing F (p) such that
1
F 1 1
F 1
=

on a small neighborhood around (p); this map is smooth since the above charts are
all smoothly compatible.

Theorem 11. [Exercise 2.7(2)] Let M and N be smooth manifolds with or without
boundary, and let F : M N be a map.
(1) If every point p M has a neighborhood U such that the restriction F |U is
smooth, then F is smooth.
(2) Conversely, if F is smooth, then its restriction to every open subset is smooth.

Proof. (1) is obvious. For (2), let E be an open subset of M . Let p E and choose
smooth charts (U, ) containing p and (V, ) containing F (p) such that F (U ) V and
F 1 is smooth. By restricting to U E, we find that F |E is also smooth. 
Theorem 12. [Exercise 2.9] Suppose F : M N is a smooth map between smooth
manifolds with or without boundary. Then the coordinate representation of F with
respect to every pair of smooth charts for M and N is smooth.
Proof. Almost identical to Theorem 9.

Theorem 13. [Exercise 2.11] Let M , N and P be smooth manifolds with or without
boundary.
(1) Every constant map c : M N is smooth.
(2) The identity map of M is smooth.
(3) If U M is an open submanifold with or without boundary, then the inclusion
map U , M is smooth.
Proof. If M 6= then let y be the constant value that c assumes and let (V, ) be a
smooth chart containing y. If x M and (U, ) is any smooth chart containing x then
c 1 is constant and therefore smooth. This proves (1). Part (2) follows from the
fact that any two coordinate maps for M are smoothly compatible. For (3), if p U
and (E, ) is any smooth chart in U containing p then 1 = Id(E) is smooth. 
Theorem 14. [Exercise 2.16]
(1) Every composition of diffeomorphisms is a diffeomorphism.
(2) Every finite product of diffeomorphisms between smooth manifolds is a diffeomorphism.
(3) Every diffeomorphism is a homeomorphism and an open map.
(4) The restriction of a diffeomorphism to an open submanifold with or without
boundary is a diffeomorphism onto its image.
(5) Diffeomorphic is an equivalence relation on the class of all smooth manifolds
with or without boundary.
Proof. If f : M N and g : N P are diffeomorphisms then f 1 g 1 is a smooth
inverse to g f , which proves (1). If fi : Mi Ni are diffeomorphisms for i = 1, . . . , n
then f11 fn1 is a smooth inverse to f1 fn , which proves (2). Part (3) follows
from the fact that smooth maps are continuous, and part (4) follows immediately from
(3) and Theorem 11. Part (5) follows from (1).

Theorem 15. [Exercise 2.19] Suppose M and N are smooth manifolds with boundary
and F : M N is a diffeomorphism. Then F (M ) = N , and F restricts to a
diffeomorphism from Int M to Int N .

Proof. It suffices to show that F (M ) N , for then F (Int M ) = Int N by Theorem


1.46 and F |Int M is a diffeomorphism by Theorem 14. Let x M and choose smooth
charts (U, ) containing x and (V, ) containing F (x) such that (x) Hn , F (U ) = V
and F 1 is a diffeomorphism. Then F 1 is a smooth coordinate map sending
F (x) to (x) Hn , so F (x) N by definition.

Example 16. [Exercise 2.27] Give a counterexample to show that the conclusion of
the extension lemma can be false if A is not closed.
Take f (x) = 1/x defined on R \ {0}.
Example 17. [Problem 2-1] Define f : R R by
(
1, x 0,
f (x) =
0, x < 0.
Show that for every x R, there are smooth coordinate charts (U, ) containing x and
(V, ) containing f (x) such that f 1 is smooth as a map from (U f 1 (V ))
to (V ), but f is not smooth as defined in this chapter.
f is clearly smooth on R\{0}. If x = 0 then = Id(1,1) is a coordinate map containing
x and = Id(1/2,3/2) is a coordinate map containing f (x) = 1 such that f 1 is
smooth on ((1, 1) f 1 ((1/2, 3/2))) = ([0, 1)) = [0, 1). However, f is clearly not
smooth since it is not continuous at 0.
Theorem 18. Let M1 , . . . , Mk1 be smooth manifolds without boundary and let Mk be
a smooth manifold with boundary. Then each projection map i : M1 Mk Mi
is smooth.
Proof. Let ni be the dimension of Mi . Let x = (x1 , . . . , xk ) M1 Mk . For each
i = 1, . . . , k, choose a smooth chart (Ui , i ) containing xi . Write U = U1 Uk and
= 1 k . Then (U, ) is a smooth chart containing x and i i 1 = pi
is smooth, where pi is a projection from Rn1 Hnk to Rni (or Hni if i = k). This
shows that i is smooth.

Theorem 19. [Problem 2-2] Suppose M1 , . . . , Mk and N are smooth manifolds with
or without boundary, such that at most one of M1 , . . . , Mk has nonempty boundary.
For each i, let i : M1 Mk Mi denote the projection onto the Mi factor. A
map F : N M1 Mk is smooth if and only if each of the component maps
Fi = i F : N Mi is smooth.
Proof. For simplicity we assume that Mk is a smooth manifold with boundary. From
Proposition 2.10(d) it is clear that every Fi is smooth if F is smooth. Suppose that
each Fi is smooth and let x N . For each i, choose smooth charts (Ui , i ) containing x
1
and (Vi , i ) containing Fi (x) such that i Fi 1
i = i i F i . Replace U1 with

U = U1 Uk and replace 1 with 1 |U ; by Theorem 12, each map i i F 1


1
is smooth. Write V = V1 Vk and = 1 k so that (V, ) is a smooth
1
chart containing F (x). Since the ith component of F 1
1 is just i i F 1 ,
the map F 1

1 is smooth. This shows that F is smooth.
Theorem 20. [Problem 2-4] The inclusion map f : Bn , Rn is smooth when Bn is
regarded as a smooth manifold with boundary.
Proof. It is clear that f is smooth on Int Bn = Bn . For points on Bn , the map provided
by Problem 3-4 in [[1]] is a suitable coordinate map.

Theorem 21. [Problem 2-7] Let M be a nonempty smooth n-manifold with or without
boundary, and suppose n 1. Then the vector space C (M ) is infinite-dimensional.
Proof. We first show that any collection {f } from C (M ) with nonempty disjoint
supports is linearly independent. Suppose there exist r1 , . . . , rk R such that
r1 f1 + + rk fk = 0
where 1 , . . . , k are distinct. For each i, choose some xi supp fi such that fi (xi ) 6=
0. Then
ri fi (xi ) = (r1 f1 + + rk fk )(xi ) = 0,
so ri = 0 since fi (xi ) 6= 0.
Choose some coordinate cube U M and a homeomorphism : U (0, 1)n . For each
i = 1, 2, . . . , let Bi = (0, 1)n1 (1/(i + 1), 1/i) and let Ai be any nonempty closed
subset of Bi . By Proposition 2.25, there is a smooth bump function fi for 1 (Ai )
supported in 1 (Bi ). Furthermore, since the sets {1 (Bi )} are disjoint, the functions
{fi } are linearly independent. This shows that C (M ) is infinite-dimensional.

Theorem 22. [Problem 2-8] Define F : Rn RPn by F (x1 , . . . , xn ) = [x1 , . . . , xn , 1].
Then F is a diffeomorphism onto a dense open subset of RPn . A similar statement
holds for G : Cn CPn defined by G(z 1 , . . . , z n ) = [z 1 , . . . , z n , 1].
Proof. The inverse to F is given by
F 1 [y 1 , . . . , y n , y n+1 ] = (y 1 /y n+1 , . . . , y n /y n+1 );
it is easy to check that the two maps are smooth since F 1 itself is a coordinate map
for RPn . The image of F is dense in RPn since any point [x1 , . . . , xn , 0] not in the image
of F can be approximated by some sequence [x1 , . . . , xn , ak ] where ak 0.

Theorem 23. [Problem 2-9] Given a polynomial p in one variable with complex coefficients, not identically zero, there is a unique smooth map pe : CP1 CP1 that makes
the following diagram commute, where CP1 is 1-dimensional complex projective space
and G : C CP1 is the map of Theorem 22:

G
C
p
C

CP1
pe

CP1

Proof. We can assume that p is not constant, for otherwise the result is clear. For any
z = [z1 , z2 ] CP1 , define
(
[p(z1 /z2 ), 1], z2 6= 0,
pe(z) =
[1, 0],
z2 = 0.
The map pe clearly satisfies the given diagram and is smooth on the image of G, so it
remains to show that pe is smooth in a neighborhood of [1, 0]. Define a diffeomorphism
H : C CP1 by
H(z) = [1, z];
by definition, (H(C), H 1 ) is a smooth chart for CP1 . We can compute
(H 1 pe H)(z) = (H 1 pe)[1, z]
(
H 1 [p(1/z), 1], z =
6 0,
=
1
H [1, 0],
z = 0,
(
1/p(1/z), z 6= 0,
=
0,
z = 0,
and it is easy to check that this map is smooth in a neighborhood of 0 whenever
p is non-constant (since 1/p(1/z) is a rational function in z). This shows that pe is
smooth.

Theorem 24. [Problem 2-10] For any topological space M , let C(M ) denote the algebra
of continuous functions f : M R. Given a continuous map F : M N , define
F : C(N ) C(M ) by F (f ) = f F .
(1) F is a linear map.
(2) Suppose M and N are smooth manifolds. Then F : M N is smooth if and
only if F (C (N )) C (M ).
(3) Suppose F : M N is a homeomorphism between smooth manifolds. Then it
is a diffeomorphism if and only if F restricts to an isomorphism from C (N )
to C (M ).

Proof. (1) is obvious. For (2), if F is smooth and f C (N ) then F (f ) = f F


C (M ) by Proposition 2.10(d). Conversely, suppose that F (C (N )) C (M ). Let
x M and let n be the dimension of N . Choose smooth charts (U, ) containing
x and (V, ) containing F (x) such that F (U ) V . For each i = 1, . . . , n the map
i F = F (i ) is smooth, where i : Rn R is the projection onto the ith
coordinate. By 9, i F 1 is smooth for i = 1, . . . , n, so F 1 must be
smooth. This shows that F is smooth. Part (3) follows directly from part (2): if F
is a diffeomorphism then (F 1 ) : C (M ) C (N ) is clearly an inverse to F , and
conversely if F |C (N ) is an isomorphism then F and F 1 are both smooth.

Theorem 25. [Problem 2-11] Suppose V is a real vector space of dimension n 1.
Define the projectivization of V , denoted by P(V ), to be the set of 1-dimensional
linear subspaces of V , with the quotient topology induced by the map : V \{0} P(V )
that sends x to its span. Then P(V ) is a topological (n 1)-manifold, and has a unique
smooth structure with the property that for each basis (E1 , . . . , En ) for V , the map
E : RPn1 P(V ) defined by E[v 1 , . . . , v n ] = [v i Ei ] is a diffeomorphism.
Proof. Obvious.

Theorem 26. [Problem 2-14] Suppose A and B are disjoint closed subsets of a smooth
manifold M . There exists f C (M ) such that 0 f (x) 1 for all x M ,
f 1 (0) = A, and f 1 (1) = B.
Proof. By Theorem 2.29, there are functions fA , fB : M R such that fA1 ({0}) = A
and fB1 ({0}) = B; the function defined by
f (x) =

fA (x)
fA (x) + fB (x)

satisfies the required properties.

Chapter 3. Tangent Vectors


Theorem 27. [Exercise 3.7] Let M , N and P be smooth manifolds with or without
boundary, let F : M N and G : N P be smooth maps, and let p M .
(1)
(2)
(3)
(4)

dFp : Tp M TF (p) N is linear.


d(G F )p = dGF (p) dFp : Tp M TGF (p) P .
d(IdM )p = IdTp M : Tp M Tp M .
If F is a diffeomorphism, then dFp : Tp M TF (p) N is an isomorphism, and
(dFp )1 = d(F 1 )F (p) .

Proof. If v, w Tp M and c R then


dFp (v + w)(f ) = (v + w)(f F ) = v(f F ) + w(f F ) = dFp (v)(f ) + dFp (w)(f )
and
dFp (cv)(f ) = (cv)(f F ) = cdFp (v)(f )

for all f C (N ), which proves (1). To prove (2), we have


d(G F )p (v)(f ) = v(f G F ) = dFp (v)(f G) = dGF (p) (dFp (v))(f )
for all f C (N ) and v Tp M , so d(G F )p = dGF (p) dFp . For (3),
d(IdM )p (v)(f ) = v(f IdM ) = v(f ),
so d(IdM )p (v) = v for all v Tp M and therefore d(IdM )p = IdTp M . Part (4) follows
immediately, since
d(F 1 )F (p) dFp = d(F 1 F )p = d(IdM )p = IdTp M
and similarly dFp d(F 1 )F (p) = IdTF (p) N .

Example 28. [Exercise 3.17] Let (x, y) denote the standard coordinates on R2 . Verify
that (e
x, ye) are global smooth coordinates on R2 , where
x
e = x,

ye = y + x3 .

Let p be the point (1, 0) R2 (in standard coordinates), and show that




6=
,
x p
e
x p
even though the coordinate functions x and x
e are identically equal.
Since an inverse to (x, y) 7 (x, y + x3 ) is given by (e
x, ye) 7 (e
x, ye x
e3 ), the coordinates
are smooth and global. Now






=
+3
,
x p
e
x p
e
y p




=
,
y p
e
y p
so /x|p 6= /e
x|p .
Theorem 29. Let X be a connected topological space and let be an equivalence
relation on X. If every x X has a neighborhood U such that p q for every p, q U ,
then p q for every p, q X.

10

Proof. Let p X and let S = {q X : p q}. If q S then there is a neighborhood


U of q such that q1 q2 for every q1 , q2 U . In particular, for every r U we have
p q and q r which implies that p r, and U S. This shows that S is open. If
q X \ S then there is a neighborhood U of q such that q1 q2 for every q1 , q2 U . If
p r for some r U then p q since q r, which contradicts the fact that q X \ S.
Therefore U X \ S, which shows that S is closed. Since X is connected, S = X. 
Corollary 30. Let X be a connected topological space and let f : X Y be a continuous map. If every x X has a neighborhood U on which f is constant, then f is
constant on X.
Theorem 31. [Problem 3-1] Suppose M and N are smooth manifolds with or without
boundary, and F : M N is a smooth map. Then dFp : Tp M TF (p) N is the zero
map for each p M if and only if F is constant on each component of M .
Proof. Suppose F is constant on each component of M and let p M . For all v Tp M
we have dFp (v)(f ) = v(f F ) = 0 since f F is constant on a neighborhood of p (see
Proposition 3.8), so dFp = 0. Conversely, suppose that dF = 0, let p M , and choose
smooth charts (U, ) containing p and (V, ) containing F (p) such that (U ) is an open
ball and F (U ) V . Let Fb = F 1 be the coordinate representation of F . From
equation (3.10), the derivative of Fb is zero since dFp = 0 and therefore Fb is constant
on (U ). This implies that F = 1 Fb is constant on U . By Corollary 30, F is
constant on each component of M .

Theorem 32. [Problem 3-2] Let M1 , . . . , Mk be smooth manifolds, and for each j,
let j : M1 Mk Mj be the projection onto the Mj factor. For any point
p = (p1 , . . . , pk ) M1 Mk , the map
: Tp (M1 Mk ) Tp1 M1 Tpk Mk
defined by
(v) = (d(1 )p (v), . . . , d(k )p (v))
is an isomorphism. The same is true if one of the spaces Mi is a smooth manifold with
boundary.
Proof. It is clear that is a linear map. For each j = 1, . . . , k, define ej : Mj
M1 Mk by ej (x) = (p1 , . . . , x, . . . , pk ) where x is in the jth position. Each ej
induces a linear map d(ej )pj : Tpj Mj Tp (M1 Mk ), so we can define a linear
map
: Tp1 M1 Tpk Mk Tp (M1 Mk )
(v1 , . . . , vk ) 7 d(e1 )p1 (v1 ) + + d(ek )pk (vk ).

11

The jth component of ( )(v1 , . . . , vk ) is then given by


!
k
k
X
X
d(j )p
d(ei )pi (vi ) =
d(j ei )pi (vi )
i=1

i=1

= d(j ej )pj (vj )


= vj
since j ei is constant when i 6= j, and j ej = IdMj . This shows that is surjective.
But
dim(Tp (M1 Mk )) = dim(Tp1 M1 Tpk Mk ),
so is an isomorphism.

Theorem 33. If (Uj , j ) are smooth charts containing pj as defined in Theorem 32,
then = 1 k is a smooth coordinate map containing p. Let nj be the dimension
n
of Mj , and write j = (x1j , . . . , xj j ) for each j so that
= 1 k = (e
x11 , . . . , x
en1 1 , . . . , x
e1k , . . . , x
enk k )
where x
eij = xij j .
(1) The collection
(

)


: 1 j k, 1 i nj
e
xij p

forms a basis for Tp (M1 Mk ).



P ji
(2) Let v Tp (M1 Mk ) and write v =
xij p . Then the jth
i,j v /e

P
component of (v) is i v ji /xij p (the variable j is fixed).
j

n
/xj j p
j

is a basis for Tpj Mj , the set 1 (B1


,...,
Proof. Since Bj =
Bk ) = (B1 Bk ) is a basis for Tp (M1 Mk ). But
!
!

(f ) = d(ej )pj
(f )
xij p
xij p
j
j
!


= d(ej 1
(f )
j )j (pj )
xij (p )
/x1j p
j

1
j )

(f ej
xij

(f 1 )
((p))
e
xij

(j (pj ))

12

= d(1 )(p)

!

(f )
e
xij (p)



=
(f ),
e
xij p
which proves (1). For (2), we compute the jth component of (v) as
!
!
X
X


sr
sr
(f ) =
v d(j )p
(f )
d(j )p
v
e
xrs p
e
xrs p
r,s
r,s
=

v jr

(f j 1 )
((p))
e
xrj

(f 1
j )
v
=
(j (pj ))
r
xj
r
!
X


=
v ji
(f ).
i
x
j
p
i
X

jr


Theorem 34. [Problem 3-3] If M and N are smooth manifolds, then T (M N ) is
diffeomorphic to T M T N .
Proof. Let m and n denote the dimensions of M and N respectively. For each (p, q)
M N , there is an isomorphism (p,q) : T(p,q) (M N ) Tp M Tq N as in Theorem
32. Define a bijection F : T (M N ) T M T N by sending each tangent vector
v T(p,q) (M N ) to (p,q) (v). For any (p, q) M N , choose smooth charts (U, )
containing p and (V, ) containing q. Let : T (M N ) M N be the natural
projection map. Then = is a smooth coordinate map for M N , e : 1 (U
V ) R2(m+n) is a smooth coordinate map for T (M N ), and
e e : 1 (U ) 1 (V )
2(m+n)
R
is a smooth coordinate map for T M T N . By Theorem 33 the coordinate
representation of F is
F (x1 , . . . , xm , y 1 , . . . , y n , v 1 , . . . , v m+n )
= (x1 , . . . , xm , v 1 , . . . , v m , y 1 , . . . , y n , v m+1 , . . . , v m+n )
which is clearly smooth. Similarly, F 1 is also smooth.

Theorem 35. [Problem 3-4] T S1 is diffeomorphic to S1 R.


Proof. We begin by showing that for each z S1 we can choose an associated tangent
vector vz Tz S1 . Let : U R be any angle function defined on a neighborhood of

13
0
0
z, and set vz = d/dx|z = (d)1
z d/dx|(z) . If : U R is any other angle function
defined around z then 0 is constant on U U 0 and (0 1 )(t) = t + c for some
0
0 1
c R. Therefore (d0 )1
z d/dx|0 (z) = (d )z d/dx |0 (z) , and vz is well-defined.

Define a map F : S1 R T S1 by sending (z, r) to rvz . If : U R is an angle


function defined around z then the coordinate representation of F is F (z, r) = ((z), r),
so F is smooth. An inverse to F is defined by sending v Tz S1 to (z, r), where v = rvz
(since T S1 is one-dimensional). The coordinate representation of F 1 is F 1 (x, r) =
(1 (x), r), so F 1 is also smooth.

Theorem 36. [Problem 3-5] Let S1 R2 be the unit circle, and let K R2 be the
boundary of the square of side 2 centered at the origin: K = {(x, y) : max(|x| , |y|) = 1}.
There is a homeomorphism F : R2 R2 such that F (S1 ) = K, but there is no diffeomorphism with the same property.
Proof. The map defined by
p
x2 + y 2
F (x, y) =
(x, y)
max(|x| , |y|)
is a homeomorphism such that F (S1 ) = K. Suppose there is a diffeomorphism F : R2
R2 such that F (S1 ) = K. Let : (1, 1) S1 be given by s 7 exp(2i(s + c)), where
c is chosen so that (F )(0) = (1, 1). We can assume without loss of generality that
(F )((, 0)) lies on the right edge of K and that (F )((0, )) lies on the top edge of
K, for small . Then there are smooth functions 1 , 2 such that (F )(t) = (1, 1 (t))
for t (, 0) and (F )(t) = (2 (t), 1) for t (0, ). Since (F )0 (t) = (0, 10 (t))
and (F )0 (t) = (20 (t), 0), taking t 0 shows that (F )0 (0) = 0. But dF ( 0 (0)) 6= 0
since 0 (0) 6= 0 and dF is an isomorphism, so this is a contradiction.

Theorem 37. [Problem 3-7] Let M be a smooth manifold with or without boundary
and p be a point of M . Let Cp (M ) denote the algebra of germs of smooth real-valued
functions at p, and let Dp M denote the vector space of derivations of Cp (M ). Define
a map : Dp M Tp M by (v)f = v([f ]p ). Then is an isomorphism.
Proof. It is clear that is a homomorphism. If v = 0 then v([f ]p ) = 0 for all
f C (M ), so v = 0 and therefore is injective. If v Tp M then we can define
v 0 Dp M by setting v 0 ([f ]p ) = v(f ). This is well-defined due to Proposition 3.8. Then
v 0 = v, so is surjective.

Theorem 38. [Problem 3-8] Let M be a smooth manifold with or without boundary
and p M . Let Vp M denote the set of equivalence classes of smooth curves starting
at p under the relation 1 2 if (f 1 )0 (0) = (f 2 )0 (0) for every smooth realvalued function f defined in a neighborhood of p. The map : Vp M Tp M defined by
[] = 0 (0) is well-defined and bijective.

14

Proof. If 1 , 2 [] then 10 (0)(f ) = (f 1 )0 (0) = (f 2 )0 (0) = 20 (0)(f ), which


shows that is well-defined. The same argument shows that is injective. Finally,
Proposition 3.23 shows that is surjective.

Chapter 4. Submersions, Immersions, and Embeddings
Theorem 39. [Exercise 4.4]
(1) The composition of two smooth submersions is a smooth submersion.
(2) The composition of two smooth immersions is a smooth immersion.
(3) The composition of two maps of constant rank need not have constant rank.
Proof. Let F : M N and G : N P be smooth maps; parts (1) and (2) follow from
the fact that d(G F )x = dGF (x) dFx . For (3), define F : R R2 and G : R2 R by
F (x) = (1, x),
so that

 
0
DF (x) =
,
1

G(x, y) = x + y 2



DG(x, y) = 1 2y ,

D(G F )(x) = 2x.




Theorem 40. [Exercise 4.7]


(1) Every composition of local diffeomorphisms is a local diffeomorphism.
(2) Every finite product of local diffeomorphisms between smooth manifolds is a local
diffeomorphism.
(3) Every local diffeomorphism is a local homeomorphism and an open map.
(4) The restriction of a local diffeomorphism to an open submanifold with or without
boundary is a local diffeomorphism.
(5) Every diffeomorphism is a local diffeomorphism.
(6) Every bijective local diffeomorphism is a diffeomorphism.
(7) A map between smooth manifolds with or without boundary is a local diffeomorphism if and only if in a neighborhood of each point of its domain, it has a
coordinate representation that is a local diffeomorphism.
Proof. Let F : M N and G : N P be local diffeomorphisms. If x M then there
are neighborhoods U of x and V of F (x) such that F |U : U V is a diffeomorphism,
and there are neighborhoods V 0 of F (x) and W of (G F )(x) such that G|V 0 : V 0
W is a diffeomorphism. Then (G F )|F 1 (V V 0 ) is a diffeomorphism onto its image,
which proves (1). Parts (2)-(5) are obvious. Suppose F : M N is a bijective local
diffeomorphism. If y N then there are neighborhoods U of F 1 (y) and V of y such
that F |U : U V is a diffeomorphism, so (F 1 )|V is smooth. Theorem 11 then shows
that F 1 is smooth, which proves (6). Part (7) is obvious.


15

Theorem 41. [Exercise 4.10] Suppose M , N , P are smooth manifolds with or without
boundary, and F : M N is a local diffeomorphism.
(1) If G : P M is continuous, then G is smooth if and only if F G is smooth.
(2) If in addition F is surjective and G : N P is any map, then G is smooth if
and only if G F is smooth.
Proof. For (1), if F G is smooth and x P then there are neighborhoods U of G(x)
and V of (F G)(x) such that F |U : U V is a diffeomorphism, so G|G1 (U ) =
(F |U )1 (F G)|G1 (U ) is smooth. By Theorem 11, G is smooth. For (2), if G F
is smooth and x N then there is a point p M such that x = F (p), and there are
neighborhoods U of p and V of x such that F |U : U V is a diffeomorphism. Then

G|V = G F (F |U )1 is smooth, so G is smooth by Theorem 11.
Example 42. [Exercise 4.24] Give an example of a smooth embedding that is neither
an open nor a closed map.
Let X = [0, 1) and Y = [1, 1], and let f : X Y be the identity map on X. Then
f is a smooth embedding, X is both open and closed, but f (X) is neither open nor
closed.
Example 43. [Exercise 4.27] Give an example of a smooth map that is a topological
submersion but not a smooth submersion.
Consider f (x) = x3 at x = 0.
Theorem 44. [Exercise 4.32] Suppose that M , N1 , and N2 are smooth manifolds, and
1 : M N1 and 2 : M N2 are surjective submersions that are constant on
each others fibers. Then there exists a unique diffeomorphism F : N1 N2 such that
F 1 = 2 .
Proof. Theorem 4.30 shows that there are unique smooth maps
e1 : N2 N1 and

e2 : N1 N2 such that 1 =
e1 2 and 2 =
e2 1 . Then
e2
e1 2 =
e2 1 = 2 ,
so
e2
e1 = IdN2 by the uniqueness part of Theorem 4.30. Similarly,
e1
e2 = IdN1 . 
Theorem 45. [Exercise 4.33]
(1) Every smooth covering map is a local diffeomorphism, a smooth submersion, an
open map, and a quotient map.
(2) An injective smooth covering map is a diffeomorphism.
(3) A topological covering map is a smooth covering map if and only if it is a local
diffeomorphism.
Proof. Let : E M be a smooth covering map. Let e E and x = (e), let U be an
e be the component of 1 (U ) containing
evenly covered neighborhood of x, and let U

16

e U is a diffeomorphism, so is a local diffeomorphism and


e. By definition |Ue : U
the other properties in (1) follow. If is injective then is a bijection. Every bijective
local diffeomorphism is a diffeomorphism, proving (2). Part (3) is obvious.

Theorem 46. [Exercise 4.37] Suppose : E M is a smooth covering map. Every
local section of is smooth.
Proof. Let : V E be a local section of where V is open in M . Let x V and let
U be an evenly covered neighborhood of x contained in V . By Proposition 4.36, there
is a unique smooth local section : U E such that (x) = (x). Therefore |U = ,
and is smooth by Theorem 11.

Theorem 47. [Exercise 4.38] Suppose E1 , . . . , Ek and M1 , . . . , Mk are smooth manifolds (without boundary), and i : Ei Mi is a smooth covering map for each
i = 1, . . . , k. Then 1 k : E1 Ek M1 Mk is a smooth
covering map.
Proof. This is true for topological covering maps. But any finite product of local diffeomorphisms is a local diffeomorphism, so the result follows from Theorem 45.

Theorem 48. Suppose : E M is a smooth covering map. If U M is evenly covered in the topological sense, then maps each component of 1 (U ) diffeomorphically
onto U .
e U is a homeomorphism
e be a component of 1 (U ) and suppose | e : U
Proof. Let U
U
(or merely a bijection). Since |Ue is a local diffeomorphism and it is bijective, it is a
diffeomorphism.

Theorem 49. [Exercise 4.42] Suppose M is a connected smooth n-manifold with boundary, and : E M is a topological covering map. Then E is a topological n-manifold
with boundary such that E = 1 (M ), and it has a unique smooth structure such
that is a smooth covering map.
Proof. Identical to Proposition 4.40.

Theorem 50. [Exercise 4.44] If M is a connected smooth manifold, there exists a


f, called the universal covering manifold of M , and
simply connected smooth manifold M
f M . The universal covering manifold is unique in the
a smooth covering map : M
f0 is any other simply connected smooth manifold that admits a
following sense: if M
f0 M , then there exists a diffeomorphism : M
fM
f0
smooth covering map 0 : M
0
such that = .

17

f M be a universal
Proof. Every manifold has a universal covering space, so let : M
f is a smooth manifold with a unique smooth
covering. Proposition 4.40 shows that M
f0 M is another universal
structure such that is a smooth covering map. If 0 : M
f M
f0 such that 0 =
smooth covering, then there is a homeomorphism : M
f, let x = (e) and choose some e0 1 ({x}). Let U be an evenly
. Let e M
e be the component of
covered neighborhood of x with respect to both and 0 , let U
e 0 be the component of ( 0 )1 (U ) containing e0 . Then
1 (U ) containing e, and let U
0
1
|Ue = ( |Ue 0 ) |Ue , so |Ue is a diffeomorphism. This shows that is a bijective local
diffeomorphism and therefore is a diffeomorphism.

Example 51. [Problem 4-1] Use the inclusion map : Hn , Rn to show that Theorem
4.5 does not extend to the case in which M is a manifold with boundary.
Lemma 3.11 shows that d0 is invertible. If Theorem 4.5 holds, then there are neighborhoods U Hn of 0 and a neighborhood V Rn of 0 such that |U V is a
diffeomorphism. Since is a bijection, we must have U = V . This is impossible,
because U cannot be open in Rn .
Theorem 52. [Problem 4-2] Suppose M is a smooth manifold (without boundary), N
is a smooth manifold with boundary, and F : M N is smooth. If p M is a point
such that dFp is nonsingular, then F (p) Int N .
Proof. Let n be the dimension of M , which is the same as the dimension of N . Suppose
F (p) N . Choose smooth charts (U, ) centered at p and (V, ) centered at F (p)
such that (V ) Hn , F (U ) V and (F (p)) = 0. Let Fe = F 1 . Then dFe0
e0 of
is invertible, so the inverse function theorem shows that there are neighborhoods U
e0 Ve0 is a diffeomorphism. But Ve0 is a
0 and Ve0 of 0 open in Rn such that Fe|Ue0 : U
n
neighborhood of 0 open in R and contained in Hn , which is impossible.

Example 53. [Problem 4-4] Let : R T2 be the curve of Example 4.20. The image
set (R) is dense in T2 .
Let (z1 , z2 ) T2 and choose s1 , s2 R so that e2is1 = z1 and e2is2 = z2 . Then
(s1 + n) = (e2is1 , e2is1 e2in ), so it remains to show that


S = e2in : n Z
is dense in S1 whenever is irrational. Let > 0. By Lemma 4.21, there exist integers
n, m such that |n m| < . Let = n m so that
e2i = e2in S
and 6= 0 since is irrational. Any integer power of e2i is also a member of S, and
since was arbitrary, we can approximate any point on S1 by taking powers of e2i .

18



Example 54. We identify S3 with the set (z, w) C2 : |z|2 + |w|2 = 1 . Let p : C
S2 \ {N } be the inverse of the stereographic projection given by
1
p(x + yi) = 2
(2x, 2y, x2 + y 2 1),
x + y2 + 1
or
1
(z + z, i(z z), |z|2 1).
p(z) = 2
|z| + 1
Define q : S3 \ {S1 {0}} C by (z, w) = z/w. Then we have a smooth map
p q : S3 \ {S1 {0}} S2 \ {N } given by


z
1
z
z
z  z 2
(p q)(z, w) =
+ , i

, 1
w w
w
|z/w|2 + 1 w w
= (z w + zw, i(z w zw), z z ww).

It is easy to see that we can extend p q so that it maps S3 into S2 by using the formula
above.
Theorem 55. [Problem 4-5] Let CPn be n-dimensional complex projective space.
(1) The quotient map : Cn+1 \ {0} CPn is a surjective smooth submersion.
(2) CP1 is diffeomorphic to S2 .
Proof. Let (z1 , . . . , zn+1 ) Cn+1 \{0} and assume without loss of generality that zn+1 6=
0. We have a coordinate map given by


w1
wn
[w1 , . . . , wn , wn+1 ] 7
,...,
.
wn+1
wn+1
Then
1
2
wn+1
0

0
w1 wn+1
1
2
0

wn+1

0
w2 wn+1
0

,
( ) (w1 , . . . , wn+1 ) = ..
..
..
..
..

.
.
.
.
.
1
2
0
0
wn+1
wn wn+1
which clearly has full rank when wk = zk for k = 1, . . . , n + 1 (since the first n columns
form an invertible matrix). This proves (1). The map : C2 \ {0} CP1 is a surjective
smooth submersion. Define a map : C2 \ {0} S2 by


(z1 , z2 )
,
(z1 , z2 ) = (p q)
|(z1 , z2 )|
where p q is the map in 54. By Theorem 4.30, descends to a unique smooth map
e : CP1 S2 satisfying e = . This map is a diffeomorphism.


Theorem 56. [Problem 4-6] Let M be a nonempty smooth compact manifold. There
is no smooth submersion F : M Rk for any k > 0.

19

Proof. By Proposition 4.28, F (M ) is open in Rk . Also, F (M ) is compact because M is


compact. Therefore F (M ) is open, closed and bounded. But Rk is connected and M
is nonempty, so F (M ) = Rk . This contradicts the boundedness of F (M ).

Theorem 57. [Problem 4-7] Suppose M and N are smooth manifolds, and : M N
is a surjective smooth submersion. There is no other smooth manifold structure on
e
N that satisfies the conclusion of Theorem 4.29; in other words, assuming that N
represents the same set as N with a possibly different topology and smooth structure,
e P is
and that for every smooth manifold P with or without boundary, a map F : N
e.
smooth if and only if F is smooth, then IdN is a diffeomorphism between N and N
e to N , and e for the identity map from
Proof. Write N for the identity map from N
N
e . Then N is smooth if and only if N = is smooth and e is smooth if and
N to N
N
only if Ne = is smooth. Both of these conditions hold, and since N and Ne are

inverses, IdN = N = Ne is a diffeomorphism.
Theorem 58. [Problem 4-8] Let : R2 R be defined by (x, y) = xy. Then is
surjective and smooth, and for each smooth manifold P , a map F : R P is smooth
if and only if F is smooth; but is not a smooth submersion (cf. Theorem 4.29).
Proof. It is clear that F is smooth if F is smooth. If F is smooth then the
map x 7 (F )(x, 1) is smooth and identical to F , so F is smooth. However, we can
compute


D(x, y) = y x ,
which does not have full rank when (x, y) = (0, 0).

Theorem 59. [Problem 4-9] Let M be a connected smooth manifold and let : E M
be a topological covering map. There is only one smooth structure on E such that is
a smooth covering map (cf. Proposition 4.40).
Proof. Let M be the smooth structure constructed in Proposition 4.40. Suppose there
is another smooth structure M0 for which is a smooth covering map. Let p E and let
(U, )
e be a smooth chart in M0 containing p. Let V be an evenly covered neighborhood
of (p)
e
and let Ve be the component of 1 (V ) containing p so that |Ve : Ve V is
a diffeomorphism (with respect to M0 ). By shrinking V and Ve if necessary, we may
assume that there is a smooth chart (V, ) containing (p). Then |Ve is a smooth
coordinate map in M, and

e ( |Ve )1 =
e (|Ve )1 1
is smooth. This shows that
e and |Ve are smoothly compatible, and since p was
0
arbitrary, M = M .


20

Example 60. [Problem 4-10] Show that the map q : Sn RPn defined in Example
2.13(f) is a smooth covering map.
Since Sn is compact, it is clear that q is proper. For each i = 1, . . . , n + 1, let
Si+ = {(x1 , . . . , xn+1 ) Sn : xi > 0} ,
Si = {(x1 , . . . , xn+1 ) Sn : xi < 0} .
Then q|Si+ and q|Si are diffeomorphisms for i = 1, . . . , n + 1, which shows that q is a
local diffeomorphism. By Proposition 4.46, q is a smooth covering map.
Example 61. [Problem 4-13] Define a map F : S2 R4 by F (x, y, z) = (x2
y 2 , xy, xz, yz). Using the smooth covering map of Example 2.13(f) and Problem 4-10,
show that F descends to a smooth embedding of RP2 into R4 .
Let q : S2 RP2 be the map of Example 60. Since F (x, y, z) = F (x, y, z), the
map F descends to a unique smooth map Fe : RP2 R4 . It is easy to check that Fe1
is smooth and given by
r
r 
r
uv
uw
vw
1
e
,
,
.
F (t, u, v, w) = q
w
v
u
Note that we take uv/w = 0 if w = 0, since w = yz = 0 implies that either u = 0 or
v = 0. This is true for the other coordinates. Therefore Fe is a homeomorphism onto
its image. It remains to show that F is a smooth immersion. Let


: RP2 \ [x, y, z] RP2 : z 6= 0 R2
x y 
,
[x, y, z] 7
z z
be coordinate map. Then
(Fe 1 )(x, y) = Fe[x, y, 1]
= (x2 y 2 , xy, x, y)
so that

2x 2y
y
x
,
D(Fe 1 )(x, y) =
1
0
0
1
which clearly has full rank. We can perform similar computations for the other coordinate maps, showing that dFe[x,y,z] is always surjective.

21

Chapter 5. Submanifolds
Example 62. [Exercise 5.10] Show that spherical coordinates (Example C.38) form a
slice chart for S2 in R3 on any open subset where they are defined.
Spherical coordinates are given by
(, , ) 7 ( sin cos , sin sin , cos ).
For the image to be a subset of S2 , we must have = 1.
Theorem 63. [Exercise 5.24] Suppose M is a smooth manifold with or without boundary, S M is an immersed k-submanifold, : S , M is the inclusion map, and U
is an open subset of Rk . A map X : U M is a smooth local parametrization of S
if and only if there is a smooth coordinate chart (V, ) for S such that X = 1 .
Therefore, every point of S is in the image of some local parametrization.
Proof. If X is a smooth local parametrization of S then X 1 : X(U ) U is a diffeomorphism, so (X(U ), X 1 ) is a smooth chart such that X = (X 1 )1 . Conversely,
suppose that (V, ) is a smooth chart such that X = 1 . Then X((V )) = V is
open in S and X is a diffeomorphism onto V , which shows that X is a smooth local
parametrization.

Theorem 64. [Exercise 5.36] Suppose M is a smooth manifold with or without boundary, S M is an immersed or embedded submanifold, and p S. A vector v Tp M
is in Tp S if and only if there is a smooth curve : J M whose image is contained in
S, and which is also smooth as a map into S, such that 0 J, (0) = p, and 0 (0) = v.
Proof. Let : S , M be the inclusion map. If v Tp S then by Proposition 3.23 there
is a smooth curve 0 : J S such that 0 J, 0 (0) = p and 00 (0) = v. Therefore
= 0 is the desired smooth curve. Conversely, if there is a smooth curve with
the specified properties then 0 : J S given by t 7 (t) is smooth. Therefore

00 (0) Tp S and v = 0 (0) = dp (00 (0)), so v is in Tp S (as a subspace of Tp M ).
Theorem 65. [Exercise 5.40] Suppose S M is a level set of a smooth map : M
N with constant rank. Then Tp S = ker dp for each p S.
Proof. Follows from Theorem 5.12 and Proposition 5.38.

Theorem 66. [Exercise 5.42] Suppose M is a smooth n-dimensional manifold with


boundary, p M , and (xi ) are any smooth boundary coordinates defined on a neighborhood of p. The inward-pointing vectors in Tp M are precisely those with positive
xn -component, the outward-pointing ones are those with negative xn -component, and
the ones tangent to M are those with zero xn -component. Thus, Tp M is the disjoint
union of Tp M , the set of inward-pointing vectors, and the set of outward-pointing
vectors, and v Tp M is inward-pointing if and only if v is outward-pointing.

22

Proof. As in Proposition 3.23.

Theorem 67. [Exercise 5.44] Suppose M is a smooth manifold with boundary, f is a


boundary defining function, and p M . A vector v Tp M is inward-pointing if and
only if vf > 0, outward-pointing if and only if vf < 0, and tangent to M if and only
if vf = 0.
Proof. Let (x1 , . . . , xn ) be smooth coordinates defined on a neighborhood of p. Since f is
a boundaryP
defining function, (f /xi )(p) = 0 for i = 1, . . . , n1 and (f /xn )(p) > 0.
Write v = ni=1 v i (/xi |p ) so that
vf =

n
X
i=1

vi

f
f
(p) = v n n (p);
i
x
x

then the result follows from Theorem 66.

Example 68. [Problem 5-1] Consider the map : R4 R2 defined by


(x, y, s, t) = (x2 + y, x2 + y 2 + s2 + t2 + y).
Show that (0, 1) is a regular value of , and that the level set S = 1 ({(0, 1)}) is
diffeomorphic to S2 .
We compute



2x
1
0 0
D(x, y, s, t) =
.
2x 2y + 1 2s 2t
This matrix clearly has full rank when s 6= 0 or t 6= 0, so suppose that s = t = 0. The
determinant of the submatrix formed by the first two columns is
2x(2y + 1) 2x = 4xy,
which is zero if and only if x = 0 or y = 0. In either case, we cannot have (x, y, s, t) =
(0, 1). This shows that dp is surjective for all p S. Define : R4 R4 by
(x, y, s, t) = (x, x2 + y, s, t);
e = ( 1 )(u, v, s, t) = (v, v + (v u2 )2 + s2 + t2 ).
then is a diffeomorphism and
It is easy to see that


(S) = (u, 0, s, t) R4 : u4 + s2 + t2 = 1 .
This set can be considered as an embedded submanifold of R4 diffeomorphic to S2 , such
that |S is a diffeomorphism. Therefore S is diffeomorphic to (S).
Theorem 69. [Problem 5-2] If M is a smooth n-manifold with boundary, then with the
subspace topology, M is a topological (n1)-dimensional manifold (without boundary),
and has a smooth structure such that it is a properly embedded submanifold of M .

23

Proof. Let x M and choose a smooth boundary chart (U, ) containing x. Then


(M U ) = (x1 , . . . , xn ) (U ) : xn = 0 ,
which shows that M satisfies the local (n 1)-slice condition. The result follows from
Theorem 5.8 and Proposition 5.5.

Theorem 70. [Problem 5-3] Suppose M is a smooth manifold with or without boundary,
and S M is an immersed submanifold. If any of the following holds, then S is
embedded.
(1) S has codimension 0 in M .
(2) The inclusion map S M is proper.
(3) S is compact.
Proof. If (1) holds then Theorem 4.5 shows that the inclusion map : S , M is an
open map. The result follows from Proposition 4.22.

Example 71. [Problem 5-4] Show that the image of the curve : (, ) R2 of
Example 4.19 given by
(t) = (sin 2t, sin t)
is not an embedded submanifold of R2 .
It is well-known that the image of with the subset topology is not a topological
manifold at all. (This can be seen by considering the point (0, 0), around which there
is no coordinate map into R.)
Example 72. [Problem 5-5] Let : R T2 be the curve of Example 4.20. Show that
(R) is not an embedded submanifold of the torus.
Again, (R) with the subset topology is not a topological manifold because it is not
locally Euclidean.
Theorem 73. [Problem 5-6] Suppose M Rn is an embedded m-dimensional submanifold, and let U M T Rn be the set of all unit tangent vectors to M :
U M = {(x, v) T Rn : x M, v Tx M, |v| = 1} .
It is called the unit tangent bundle of M . Prove that U M is an embedded (2m 1)dimensional submanifold of T Rn Rn Rn .
Proof. Let (x, v) U M . Since M is an embedded submanifold of Rn , we can choose a
smooth chart (U, ) for Rn containing x such that


(M U ) = (x1 , . . . , xn ) (U ) : xm+1 = = xn = 0 .

24

Similarly, Sm1 is an embedded submanifold of Rm , so we can choose a smooth chart


(V, ) for Rm containing v such that


(Sm1 V ) = (x1 , . . . , xm ) (V ) : xm = 0 .
e = 1 (U ). By definition, the map
e R2n given by
Write = (x1 , . . . , xn ) and U
e:U


7 (x1 (p), . . . , xn (p), v 1 , . . . , v n )
vi
xi
p

e , we can assume that V ((


e )), where
is a coordinate map for T Rn . By shrinking U
eU
2n
m
: R R is the projection onto the coordinates n + 1, . . . , n + m. Define
(x1 , . . . , xn , v 1 , . . . , v n ) = (x1 , . . . , xn , (v 1 , . . . , v m ), v m+1 , . . . , v n );
e R2n is still a coordinate
then is a diffeomorphism onto its image, and
e:U
map for T Rn . Furthermore,
e)
( )(U
e M U
n
o
e ) : xm+1 = = xn = v m = = v n = 0 ,
= (x1 , . . . , xn , v 1 , . . . , v n ) ( )(
e U
so U M satisfies the local (2m1)-slice condition. By Theorem 5.8, U M is an embedded
(2m 1)-dimensional submanifold of T Rn .

Example 74. [Problem 5-7] Let F : R2 R be defined by F (x, y) = x3 + xy + y 3 .
Which level sets of F are embedded submanifolds of R2 ? For each level set, prove either
that it is or that it is not an embedded submanifold.
We compute


DF (x, y) = 3x2 + y x + 3y 2 ,
which has full rank unless 3x2 + y = 0 and x + 3y 2 = 0. In this case, we have
y = 3x2 = 3(3y 2 )2 = 27y 4
and
0 = y + 27y 4 = y(3y + 1)(9y 2 3y + 1),
so y = 0 or y = 31 . By symmetry, we have x = 0 or x = 13 . From this, it is clear
that DF (x, y) has full rank if and only if (x, y) 6= (0, 0) and (x, y) 6= ( 31 , 13 ). Note
1
that F (0, 0) = 0 and F ( 13 , 13 ) = 27
. We first examine the level set S1 = F 1 ({0}).
The point p = (0, 0) is a saddle point of
 1F , so S1 is a self-intersecting curve at p. Next,
we examine the level set S2 = F 1 ( 27
). Since F has a strict local maximum at
1
1
p = ( 3 , 3 ), the point p is isolated in S2 . In both cases, the level sets fail to be locally
Euclidean. Therefore, we can conclude that F 1 ({c}) is an embedded 1-dimensional
1
submanifold of R2 if and only if c 6= 0 and c 6= 27
.
Theorem 75. Any closed ball in Rn is an n-dimensional manifold with boundary.

25

Proof. Let Bn = B1 (0) be the closed unit ball in Rn , let N = (0, . . . , 0, 1) be the north
pole, and let T = {0}n1 [0, 1] be the vertical line connecting 0 and N . Define
: Bn \ T Hn given by


x1
xn1
1
(x1 , . . . , xn ) =
,...,
, kxk 1 ,
kxk xn
kxk xn
with its inverse given by
1 (u1 , . . . , un ) =

1
(2u1 , . . . , 2un1 , ke
uk2 1)
2
(un + 1)(ke
uk + 1)

where u
e = (u1 , . . . , un1 ). Since both maps are continuous, this proves that Bn \ T is
homeomorphic to Hn . Furthermore, we can repeat the same argument by taking the
vertical line T = {0}n1 [1, 0] instead, giving us Euclidean neighborhoods of every
point in Bn except for 0. But the identity map on the open unit ball Bn is a suitable
coordinate chart around 0, so this proves that Bn is an n-manifold with boundary. 
Theorem 76. [Problem 5-8] Suppose M is a smooth n-manifold and B M is a regular
coordinate ball. Then M \ B is a smooth manifold with boundary, whose boundary is
diffeomorphic to Sn1 .
Proof. We have coordinate balls around every point of M \B, so it remains to show that
we have coordinate half-balls around each point of B. Since B is a regular coordinate
ball, this reduces to showing that Br (0) \ Bs (0) is a smooth manifold with boundary
whenever s < r. But this follows easily from the stereographic projection described in
Theorem 75.

Theorem 77. [Problem 5-9] Let S R2 be the boundary of the square of side 2 centered
at the origin (see Theorem 36). Then S does not have a topology and smooth structure
in which it is an immersed submanifold of R2 .
Proof. Suppose S has a topology and smooth structure for which the inclusion map
: S , R2 is a smooth immersion. By Theorem 4.25, there is a neighborhood U
of (1, 1) such that |U is a smooth embedding. We can then apply the argument of
Theorem 36 to derive a contradiction.

Example 78. [Problem 5-10] For each a R, let Ma be the subset of R2 defined by


Ma = (x, y) : y 2 = x(x 1)(x a) .
For which values of a is Ma an embedded submanifold of R2 ? For which values can Ma
be given a topology and smooth structure making it into an immersed submanifold?
Let F (x, y) = y 2 x(x 1)(x a); then


DF (x, y) = (3x2 (2a + 2)x + a) 2y .

26

Therefore 0 is a regular value of F unless there is a point (x, y) such that y = 0,


3x2 (2a + 2)x + a = 0 and F (x, y) = x(x 1)(x a) = 0. We have the following
cases to consider:
a = 0 and (x, y) = (0, 0),
a = 1 and (x, y) = (1, 0).
When a = 0 the point (0, 0) is a local minimum of F , so (0, 0) is an isolated point of
M0 . Therefore M0 cannot be an embedded or immersed submanifold. When a = 1 the
curve M1 is self-intersecting at (1, 0). By giving M1 an appropriate topology in which
it is disconnected, we can make M1 an immersed submanifold of R2 .
Theorem 79. [Problem 5-12] Suppose E and M are smooth manifolds with boundary,
and : E M is a smooth covering map. The restriction of to each connected
component of E is a smooth covering map onto a component of M .
Proof. Let F be a connected component of E. Let x (F ), let U be an evenly
e be the component
covered neighborhood of x, choose e so that x = (e), and let U
1
e
of (U ) containing e. Then |Ue : U U is a diffeomorphism. It is clear that
e F ) = U M . Since U
e F is an embedded submanifold of U
e and (U
e F ) is an
(U
embedded submanifold of U by Theorem 5.11, |Ue F is a diffeomorphism. This shows
that U M is an evenly covered neighborhood of x, and that the image of |F is open.
Let x
/ (F ), let U be an evenly covered neighborhood of x, and let V = U M . By
shrinking V if necessary, we may assume that V is a connected neighborhood of x in
e be the component
U M . Suppose there is a f F such that y = (f ) V . Let U
e so that x = (e). Let be a path from x
of 1 (U ) containing f and choose e U
e E, so e F since f F
to y in V . Then (|Ue )1 is a path from e to f in U
and F is a connected component of E. This contradicts the fact that x
/ (F ), so
V M \ (F ). This shows that (F ) is both open and closed in M , so it is a
component of M .

Theorem 80. [Problem 5-14] Suppose M is a smooth manifold and S M is an
immersed submanifold. For the given topology on S, there is only one smooth structure
making S into an immersed submanifold.
Proof. Let : S , M be the inclusion map, which is a smooth immersion. Let S 0 denote
S with some other smooth structure such that 0 : S 0 , M is a smooth immersion.
Since S and S 0 have the same topology, the maps : S S 0 and 0 : S 0 S are
both continuous, and Theorem 5.29 shows that the maps are inverses of each other.
Therefore S is diffeomorphic to S 0 .


27

Theorem 81. [Problem 5-16] If M is a smooth manifold and S M is a weakly


embedded submanifold, then S has only one topology and smooth structure with respect
to which it is an immersed submanifold.
Proof. Almost identical to Theorem 80.

Theorem 82. [Problem 5-17] Suppose M is a smooth manifold and S M is a smooth


submanifold.
(1) S is embedded only if every f C (S) has a smooth extension to a neighborhood
of S in M .
(2) S is properly embedded only if every f C (S) has a smooth extension to all
of M .
Proof. First suppose that S is embedded. Let p S and choose a smooth slice chart
(Up , p ) for S in M containing p. Then S Up is closed in Up , so Proposition 2.25 shows
that there is smooth function fp : Up R such that fp |SUp = f |SUp . Let {p }pS be
a partition of unity subordinate to {Up }pS ; then
X
fe(x) =
p (x)fp (x)
pS

is a smooth function defined on U = pS Up such that fe|S = f , taking fp to be zero


on U \ supp fp . For (2), Proposition 5.5 shows that S is closed in M . Therefore we have
a partition of unity {p } {0 } for M subordinate to {Up } {M \ S}, allowing fe to
be defined on all of M .

Theorem 83. [Problem 5-19] Suppose S M is an embedded submanifold and : J
M is a smooth curve whose image happens to lie in S. Then 0 (t) is in the subspace
T(t) S of T(t) M for all t J. This need not be true if S is not embedded.
Proof. Corollary 5.30 shows that is smooth as a map into S. Denote this map by
0 : J S and let : S , M . Then = 0 , so 0 (t) = d(t) (00 (t)) by Proposition
3.24 and therefore 0 (t) T(t) S T(t) M . This need not hold if S is merely an
immersed submanifold. For example, consider a curve that crosses the point of selfintersection in the figure-eight curve of Example 4.19, such that is not continuous
as a map into the image of .

Theorem 84. [Problem 5-21] Suppose M is a smooth manifold and f C (M ).
(1) For each regular value b of f , the sublevel set f 1 ((, b]) is a regular domain
in M .
(2) If a and b are two regular values of f with a < b, then f 1 ([a, b]) is a regular
domain in M .

28

Proof. Let m be the dimension of M . It is clear that f 1 ((, b)) satisfies the local
m-slice condition. Furthermore, there is a boundary slice chart around each point
of f 1 ({b}). We can see this by applying the argument in Corollary 5.14, with the
following modification to Theorem 5.12: for each p f 1 ({b}), the rank theorem shows
that there are smooth charts (U, ) centered at p and (V, ) containing b for which the
coordinate representation fb of f has the form
fb(x1 , . . . , xm ) = x1 .
By modifying fb and appropriately, U becomes a boundary slice chart for f 1 ((, b])
in M . Theorem 5.51 then shows that f 1 ((, b]) is an embedded submanifold with
boundary in M , and it is properly embedded since it is closed in M . Part (2) is
similar.

Theorem 85. [Problem 5-22] If M is a smooth manifold and D M is a regular
domain, then there exists a defining function for D. If D is compact, then f can be
taken to be a smooth exhaustion function for M .
Proof. Let U = {(Up , p )}pD be a collection of interior or boundary slice charts for D
in M . Then U {M \ D} covers M , and we can repeat the argument of Proposition
5.43 by constructing functions {fp }pD {f0 } as follows: set fp = 1 when p Int D,
set fp (x1 , . . . , xn ) = xn when p D, and set f0 = 1 for f0 : M \ D R. If D is
compact then we can modify f by using the argument of Proposition 2.28, taking a
countable open cover of M \ D by precompact open subsets.

Chapter 6. Sards Theorem
Theorem 86. [Exercise 6.7] Let M be a smooth manifold with or without boundary.
Any countable union of sets of measure zero in M has measure zero.
S
Proof. Let A =
n=1 An where each An has measure zero. Let (U, ) be a smooth
chart; we want to show that

[
AU =
An U
n=1

has measure zero. But this follows from the fact that
!

[
[

An U =
(An U )
n=1
n

n=1

has measure zero (in R ) since each (An U ) has measure zero.

29

Theorem 87. [Problem 6-1] Suppose M and N are smooth manifolds with or without
boundary, and F : M N is a smooth map. If dim M < dim N , then F (M ) has
measure zero in N .
Proof. Let n = dim N . Define Fe : M Rk N by (x, y) 7 F (x), where k =
dim N dim M . This map is smooth because it can be written as the composition
(F IdRk ), where : N Rk N is the canonical projection. It suffices to show
that the image of Fe has measure zero in N . Let (U1 U2 , 1 2 ) and (V, ) be smooth
charts for M Rk and N respectively such that U1 M and U2 Rk . Let U = U1 U2
and = 1 2 . Then
(Fe(U ) V ) = (Fe(U Fe1 (V )))
= ( Fe 1 )((U Fe1 (V )))
( Fe 1 )((U ) (1 (F 1 (V )) {0})),
which has measure zero in Rn by Proposition 6.5. Therefore Fe(U ) has measure zero.
But Fe(M Rk ) can be written as a union of countably many sets of this form, so
Fe(M Rk ) = F (M ) has measure zero in N .

Theorem 88. [Problem 6-2] Every smooth n-manifold (without boundary) admits a
smooth immersion into R2n .
Proof. Let M be a smooth n-manifold; by Theorem 6.15, we can assume that M is an
embedded submanifold of R2n+1 . Let U M T R2n+1 be the unit tangent bundle of M
(see Theorem 73) and define G : U M RP2n by (x, v) 7 [v]. This map is smooth

because it is the composition U M R2n+1 \ {0}


RP2n . Since dim U M < dim RP2n ,
Sards theorem shows that the image of G has measure zero in RP2n , and since (R2n )
has measure zero in RP2n , there is a v R2n+1 \ R2n such that [v] is not in the image
of G. Let v : R2n+1 R2n be the projection with kernel Rv; then v |M is smooth.
Furthermore, if p M then v
/ Tp M , so d(v |M )p is injective.

Theorem 89. [Problem 6-3] Let M be a smooth manifold, let B M be a closed
subset, and let : M R be a positive continuous function. There is a smooth
e
function e : M R that is zero on B, positive on M \ B, and satisfies (x)
< (x)
everywhere.
Proof. By Theorem 2.29, there is a smooth nonnegative function f : M R such that
f 1 ({0}) = B. By replacing f with f /(f + 1), we can assume that f = 0 on B and
0 < f 1 on M \ B. By Corollary 6.22, there is a smooth e : M R such that
0 < e(x) < (x) for all x M . Then e = f e is the desired function.


30

Example 90. [Problem 6-7] By considering the map F : R H2 given by F (t) = (t, |t|)
and the subset A = [0, ) R, show that the conclusions of Theorem 6.26 and
Corollary 6.27 can be false when M has nonempty boundary.
F is smooth on the closed subset [0, ), but there is no smooth map G : R H2 such
that G|A = F |A .
Theorem 91. [Problem 6-8] Every proper continuous map between smooth manifolds
is homotopic to a proper smooth map.
Proof. Since the retraction r : U M is proper, it suffices to show that if F is proper
in Theorem 6.21, then Fe is also proper. Let E Rk be compact. Since is continuous,
it attains some maximum (x0 ) on E. Let B be a closed ball containing E such that
for every x E, the closed ball of radius (x) around x is contained in B. Since
|F (x) Fe(x)| < (x), it is clear that Fe1 (E) is a closed subset of F 1 (B). But F is
proper, so F 1 (B) is compact and therefore Fe1 (E) is compact.

Example 92. [Problem 6-9] Let F : R2 R3 be the map
F (x, y) = (ey cos x, ey sin x, ey ).
For which positive numbers r is F transverse to the sphere Sr (0) R3 ? For which
positive numbers r is F 1 (Sr (0)) an embedded submanifold of R2 ?
We compute

y
e sin x ey cos x
DF (x, y) = ey cos x ey sin x .
0
ey
We have kF (x, y)k = r if and only if
r2 = e2y cos2 x + e2y sin2 x + e2y = e2y + e2y ,
i.e.
1
y = log
2


1 2 4
(r r 4) .
2

Thus F 1 (Sr (0)) isalways an embedded submanifold


of R2 , being either empty (r <

2), one line (r = 2) or two lines in R2 (r > 2).However, F is transverse to Sr (0)


if and only if r > 2. Let us examine the case r = 2: we have y = 0, so the image of
dF(x,y) is spanned by the columns of

sin x cos x
DF (x, 0) = cos x sin x .
0
1

31

The corresponding point in S2 (0) is v(x) = (cos x, sin x, 1), so the tangent space consists of all vectors orthogonal to v(x). But v(x) is orthogonal to the columns of DF (x, 0),
so dF(x,y) + Tv(x) S2 (0) is a proper subset of Tv(x) R3 .
Theorem 93. [Problem 6-15] Suppose M and N are smooth manifolds and S M N
is an immersed submanifold. Let M and N denote the projections from M N onto
M and N , respectively. The following are equivalent:
(1) S is the graph of a smooth map f : M N .
(2) M |S is a diffeomorphism from S onto M .
(3) For each p M , the submanifolds S and {p} N intersect transversely in
exactly one point.
If these conditions hold, then S is the graph of the map f : M N defined by f =
N (M |S )1 .
Proof. Let m = dim M and n = dim N . Suppose S = {(x, f (x)) : x M } for some
smooth map f : M N . Then S is the image of the map g : M S given by
x 7 (x, f (x)), so M |S is smooth with inverse g. Conversely, if M |S is a diffeomorphism
then we can take f = N (M |S )1 in (1). This proves that (1) (2). If (1) holds
then d(M )g(p) dgp is always nonzero, so Tp S + Tp ({p} N ) = Tp (M N ). It is also
clear that S ({p} N ) consists of exactly one point. If (3) holds then M |S is a
bijection. For every p M , the submanifolds S and {p} N intersect transversely.
Since dim({p} N ) = n < dim(M N ), we must have dim d(M |S )p (Tp S) m.
Therefore d(M |S )p is invertible for every p M , and M |S is a diffeomorphism. This
proves that (1) (3).

Chapter 7. Lie Groups
Theorem 94. [Exercise 7.2] If G is a smooth manifold with a group structure such
that the map t : G G G given by (g, h) 7 gh1 is smooth, then G is a Lie group.
Proof. The inversion map i(g) = t(e, g) is smooth, and so is the multiplication map
m(g, h) = t(g, i(h)) = t(g, t(e, h)).

Theorem 95. [Problem 7-1] For any Lie group G, the multiplication map m : GG
G is a smooth submersion.
Proof. For each g G, the map g : G G G given by x 7 (g, g 1 x) is a smooth
local section of m since m(g (x)) = gg 1 x = x. Every point (g1 , g2 ) G G is in the
image of g1 , so Theorem 4.26 shows that m is a smooth submersion.

Theorem 96. [Problem 7-2] Let G be a Lie group.

32

(1) Let m : G G G denote the multiplication map. Using Proposition 3.14 to


identify T(e,e) (GG) with Te GTe G, the differential dm(e,e) : Te GTe G Te G
is given by
dm(e,e) (X, Y ) = X + Y.
(2) Let i : G G denote the inversion map. Then die : Te G Te G is given by
die (X) = X.
Proof. We have
dm(e,e) (X, Y ) = dm(e,e) (X, 0) + dm(e,e) (0, Y )
= d(m(1) )e (X) + d(m(2) )e (Y ),
where m(1) : G G is given by x 7 m(x, e) and m(2) : G G is given by y 7 m(e, y).
But m(1) = m(2) = IdG , so dm(e,e) (X, Y ) = X + Y . This proves (1). Let n = m p s
where s : G G G is given by x 7 (x, x) and p : G G G G is given by
(x, y) 7 (x, i(y)); then n is constant, so
0 = dne (X)
= dm(e,e) (dp(e,e) (dse (X)))
= dm(e,e) (dp(e,e) (X, X))
= dm(e,e) (X, die (X))
= X + die (X).
Therefore die (X) = X.

Theorem 97. [Problem 7-3] If G is a smooth manifold with a group structure such
that the multiplication map m : G G G is smooth, then G is a Lie group.
Proof. It suffices to show that the inversion map i : G G is smooth. Define F :
G G G G by (g, h)
7 (g, gh), which is smooth and bijective. We have
dF(e,e) (X, Y ) = (X, X + Y ),
so dF(e,e) is an isomorphism. Write Lg and Rg for the translation maps. If (x, y) GG
then it is easy to check that
F = ((Lxy Ry1 ) IdG ) F ((Ly1 x1 Ry ) Ly1 ),
so dF(x,y) = dF(e,e) where , are isomorphisms. Therefore dF(x,y) is an isomorphism and F is a bijective local diffeomorphism, i.e. a diffeomorphism. Since i(g) is
the second component of F 1 (g, e), the inversion map i is smooth.

Theorem 98. [Problem 7-4] Let det : GL(n, R) R denote the determinant function.
Then
d(det)X (B) = (det X) tr(X 1 B).

33

Proof. Let A M(n, R) write Ai,j for the (i, j) entry of A. Then


d
d
det(In + tA) = (1 + tr(A)t + )
dt t=0
dt t=0
= tr(A).
If X GL(n, R) and B TX GL(n, R) we have
det(X + tB) = det(X) det(In + tX 1 B),
so

d
d(det)X (B) = det(X + tB)
dt t=0

d
= det(X) det(In + tX 1 B)
dt t=0
= det(X) tr(X 1 B).

Theorem 99. [Problem 7-5] For any connected Lie group G, the universal covering
e and G
e0 are simply connected Lie groups
group is unique in the following sense: if G
e G and 0 : G
e0 G that are also Lie group
that admit smooth covering maps : G
e G
e0 such that
homomorphisms, then there exists a Lie group isomorphism : G
0
= .
e and let e0 be the identity in G
e0 . By Theorem 50 there
Proof. Let e be the identity in G
eG
e0 such that 0 = , so it remains to show that is a
is a diffeomorphism : G
group homomorphism. By replacing with (e)1 = L(e)1 , we can assume that
(e) = e0 . (Since 0 ((e)1 (g)) = 0 ((e))1 (g) = (g), we still have 0 = .) If
e then
g, h G
0 ((gh)) = (g)(h) = 0 ((g)(h)),
so (g, h) 7 (gh) and (g, h) 7 (g)(h) are both lifts of (g, h) 7 (gh). Since the
maps all agree at the point (e, e), we have (gh) = (g)(h).

Theorem 100. [Problem 7-6] Suppose G is a Lie group and U is any neighborhood
of the identity. There exists a neighborhood V of the identity such that V U and
gh1 U whenever g, h V .
Proof. Define f (g, h) = gh1 and let W = f 1 (U ). Since (e, e) W , there are neighborhoods W1 , W2 of e such that (e, e) W1 W2 W . Then V = W1 W2 is the
desired neighborhood of the identity.


34

Theorem 101. [Problem 7-7] Let G be a Lie group and let G0 be its identity component.
Then G0 is a normal subgroup of G, and is the only connected open subgroup. Every
connected component of G is diffeomorphic to G0 .
Proof. The subgroup H generated by G0 is a connected open subgroup of G, so H G0
since G0 is a connected component. Therefore H = G0 . For each g G, define the
conjugation map Cg : G G by h 7 ghg 1 . This map is a Lie group isomorphism, so
Cg (G0 ) is a connected open subgroup of G and again we have Cg (G0 ) = G0 . Every left
coset gG0 is the image of G0 under the diffeomorphism Lg , so gG0 is a connected open
subset of G. But G is the union of all such cosets, so the connected components of G
are precisely the cosets of G0 .

Theorem 102. [Problem 7-8] Suppose a connected topological group G acts continuously on a discrete space K. Then the action is trivial.
Proof. Let : G K K be the group action. For any x K the set (G {x}) is
connected and therefore consists of one point. But (e, x) = x, so (g, x) = x for every
g G.

Theorem 103. [Problem 7-9,7-10] The formula
A [x] = [Ax]
defines a smooth, transitive left action of GL(n + 1, R) on RPn . The same formula
defines a smooth, transitive left action of GL(n + 1, C) on CPn .
Proof. The smooth map e : GL(n + 1, R) Rn+1 \ {0} RPn given by (A, x) 7 [Ax]
is constant on each line in Rn+1 since A is linear, so it descends to a smooth map
: GL(n + 1, R) RPn RPn given by (A, [x]) 7 [Ax]. It is easy to see that is a
group action. Furthermore, for any two points v, w Rn+1 there is an invertible linear
map that takes v to w, so is transitive.

Example 104. [Problem 7-11] Considering S2n+1 as the unit sphere in Cn+1 , define an
action of S1 on S2n+1 , called the Hopf action, by
z (w1 , . . . , wn+1 ) = (zw1 , . . . , zwn+1 ).
This action is smooth and its orbits are disjoint unit circles in Cn+1 whose union is
S2n+1 .
Theorem 105. [Problem 7-12] Every Lie group homomorphism has constant rank.
Proof. If F : G H is a Lie group homomorphism, then we can apply Theorem 7.25
with the left translation action on G and the action g h = F (g)h on H.


35

Theorem 106. [Problem 7-13,7-14] For each n 1, U(n) is a properly embedded


n2 -dimensional Lie subgroup of GL(n, C) and SU(n) is a properly embedded (n2 1)dimensional Lie subgroup of U(n).
Proof. We can use the argument in Example 7.27, replacing the transpose operator
with the conjugate transpose operator.

Theorem 107. [Problem 7-15] SO(2), U(1), and S1 are all isomorphic as Lie groups.
Proof. U(1) is obviously identical to S1 . We also have a Lie group isomorphism f :
S1 SO(2) given by

 

Re(z) Im(z)
cos sin
f (z) =
=
,
Im(z) Re(z)
sin cos
where = arg(z).

Theorem 108. [Problem 7-16] SU(2) is diffeomorphic to S3 .




Proof. Identifying S3 with the subspace (z, w) C2 : |z|2 + |w|2 = 1 , we have a diffeomorphism


z w
(z, w) 7
.
w z

Example 109. [Problem 7-17] Determine which of the following Lie groups are compact:
GL(n, R), SL(n, R), GL(n, C), SL(n, C), U(n), SU(n).
Clearly GL(n, R) and GL(n, C) are not compact. SL(n, R) and SL(n, C) are not compact, since the operator norm of

r
r1 SL(n, R)
I
is unbounded as r . However, U(n) is compact since every unitary matrix has
operator norm 1, and SU(n) is compact because it is a properly embedded subgroup of
U(n).
Theorem 110. [Problem 7-18] Suppose G is a Lie group, and N, H G are Lie
subgroups such that N is normal, N H = {e}, and N H = G. Then the map (n, h) 7
nh is a Lie group isomorphism between N o H and G, where : H N N is the
action by conjugation: h (n) = hnh1 . Furthermore, N and H are closed in G.

36

Proof. Let be the map (n, h) 7 nh. If nh = e then n = h1 N H, so n = h = e.


Also, is surjective since N H = G. Therefore is a bijection. We have
((n, h)(n0 , h0 )) = (nhn0 h1 , hh0 )
= nhn0 h1 hh0
= nhn0 h0
= (n, h)(n0 , h0 ),
so is an isomorphism. Since is smooth, it is a Lie group isomorphism.

Theorem 111. [Problem 7-19] Suppose G, N , and H are Lie groups. Then G is isomorphic to a semidirect product N oH if and only if there are Lie group homomorphisms
: G H and : H G such that = IdH and ker
= N.
Proof. Suppose there is an isomorphism f : G N o H. Let : N o H H
be the projection (n, h) 7 h and let : H N o H be the injection h 7 (e, h).
Take = f and = ; then = IdH and ker
= N . Conversely, suppose the
homomorphisms and exist. Let N = ker and H1 = (H). Then N is normal in G
and N H1 = {e}, since x = (h) and x ker implies that h = ((h)) = (x) = e.
For any g G we have (((g))) = (g), so g = n((g)) for some n ker .
Therefore N H1 = G. By Theorem 110, G is a semidirect product.

Example 112. [Problem 7-20] Prove that the following Lie groups are isomorphic to
semidirect products as shown.
(1)
(2)
(3)
(4)

O(n)
= SO(n) o O(1).
U(n)
= SU(n) o U(1).
GL(n, R)
= SL(n, R) o R .

GL(n, C) = SL(n, C) o C .

For (1) to (4), define by M 7 det(M ) and by




z
z 7
In1
where In1 is the (n 1) (n 1) identity matrix, and apply Theorem 111.
Theorem 113. [Problem 7-23] Let H be the Lie group of nonzero quaternions, and
let S H be the set of unit quaternions. Then S is a properly embedded Lie subgroup
of H , isomorphic to SU(2).
Proof. The norm || : H R is a smooth homomorphism, since
q
p
|(a, b)| = (a, b)(
a, b) = |a|2 + |b|2

37

and |pq| = |p| |q|. Therefore ker || = S is a properly embedded Lie subgroup of H .
Furthermore, we have a Lie group isomorphism


a b
(a, b) 7
b a

into SU(2).


Chapter 8. Vector Fields

Theorem 114. [Exercise 8.29] For X, Y X(M ) and f, g C (M ), we have


[f X, gY ] = f g[X, Y ] + (f Xg)Y (gY f )X.
Proof. We compute
[f X, gY ]h = f X(gY h) gY (f Xh)
= gf XY h + Y hf Xg f gY Xh XhgY f
= f g[X, Y ]h + (f Xg)Y h (gY f )Xh.

Theorem 115. [Exercise 8.34] Let A : g h be a Lie algebra homomorphism. Then
ker A and im A are Lie subalgebras.
Proof. It suffices to show that ker A and im A are closed under brackets. If X, Y ker A
then A[X, Y ] = [AX, AY ] = 0, so [X, Y ] ker A. Similarly, if X, Y im A then
X = AX 0 and Y = AY 0 for some X 0 , Y 0 g, so A[X 0 , Y 0 ] = [AX 0 , AY 0 ] = [X, Y ] and
[X, Y ] im A.

Theorem 116. [Exercise 8.35] Suppose g and h are finite-dimensional Lie algebras
and A : g h is a linear map. Then A is a Lie algebra homomorphism if and only if
A[Ei , Ej ] = [AEi , AEj ] for some basis (E1 , . . . , En ) of g.
Proof. One direction is obvious. Suppose that
basis of g and A[Ei , Ej ] =
P (E1 , . . . , En ) is a P
[AEi , AEj ]. Let X, Y g and write X = ni=1 xi Ei and Y = ni=1 yi Ei ; then
" n
#
n
X
X
xi Ei ,
yj Ej
A[X, Y ] = A
i=1

n
X
i=1
n
X
i=1

xi
xi

n
X
j=1
n
X
j=1

j=1

yj A [Ei , Ej ]
yj [AEi , AEj ]

38

"
= A

n
X

xi Ei , A

i=1

n
X

#
yj Ej

j=1

= [AX, AY ].

Theorem 117. [Exercise 8.43] If V is any finite-dimensional real vector space, the
composition of canonical isomorphisms
Lie(GL(V )) TId GL(V ) gl(V )
yields a Lie algebra isomorphism between Lie(GL(V )) and gl(V ).
Proof. Choose a basis B for V and let : GL(V ) GL(n, R) be the associated Lie
group isomorphism. It is also a Lie algebra isomorphism gl(V ) gl(n, R) since it
is linear. By Corollary 8.31 there is a Lie algebra isomorphism : Lie(GL(V ))
Lie(GL(n, R)) induced by the diffeomorphism , and by Proposition 8.41 there is a Lie
algebra isomorphism : Lie(GL(n, R)) gl(n, R). Then 1 is the desired Lie
algebra isomorphism.

Theorem 118. [Problem 8-1] Let M be a smooth manifold with or without boundary,
and let A M be a closed subset. Suppose X is a smooth vector field along A. Given
e on M such
any open subset U containing A, there exists a smooth global vector field X
e
e
that X|A = X and supp X U .
ep
Proof. For each p A, choose a neighborhood Wp of p and a smooth vector field X
on Wp that agrees with X on Wp A. Replacing Wp by Wp U we may assume that
Wp U . The family of sets {Wp : p A} {M \ A} is an open cover of M . Let
{p : p A} {0 } be a smooth partition of unity subordinate to this cover, with
supp p Wp and supp 0 M \ A.
ep is smooth on Wp by Proposition 8.8, and has a smooth
For each p A, the product p X
extension to all of M if we interpret it to be zero on M \supp p . (The extended function
is smooth because the two definitions agree on the open subset Wp \ supp p where they
e : M T M by
overlap.) Thus we can define X
X
ex =
ep |x .
X
p (x)X
pA

Because the collection of supports {supp p } is locally finite, this sum actually has only
a finite number of nonzero terms in a neighborhood of any point of M , and therefore

39

ep |x = Xx for each p such


defines a smooth function. If x A, then 0 (x) = 0 and X
that p (x) 6= 0, so
!
X
X
ex =
X
p (x)Xx = 0 (x) +
p (x) Xx = Xx ,
pA

pA

e is indeed an extension of X. It follows from Lemma 1.13(b) that


so X
[
[
e=
supp p =
supp p U.
supp X
pA

pA


Theorem 119. [Problem 8-2] Let c be a real number, and let f : Rn \ {0} R
be a smooth function that is positively homogeneous of degree c, meaning that
f (x) = c f (x) for all > 0 and x Rn \ {0}. Then V f = cf , where V is the Euler
vector field defined in Example 8.3.
Proof. We want to compute
(V f )(x) = Vx f =

n
X

xi

i=1

f
(x).
xi

Differentiating f (x) = f (x) with respect to on (0, ), we have


cc1 f (x) = Df (x)(x) =

n
X

xi

i=1

f
(x).
xi

Replacing x with 1 x gives


c

c1

f ( x) =

cc1 c f (x) = 1

n
X
i=1
n
X
i=1

cf (x) =

n
X
i=1

xi

xi

f
(x)
xi

xi

f
(x)
xi

f
(x) = (V f )(x).
xi


Theorem 120. [Problem 8-3] Let M be a nonempty positive-dimensional smooth manifold with or without boundary. Then X(M ) is infinite-dimensional.

40

Proof. Let n = dim M . If n 2 then the result is clear from Proposition 8.7 since
there are infinitely many lines in R2 , i.e. RP1 is infinite. Suppose n = 1 and choose
any diffeomorphism : (, 1 + ) U where > 0 and U is open in M . For each
m = 1, 2, . . . , define a vector field Xm along {1/k : k = 1, . . . , m} by setting
(
0,
k < m,
Xm |1/k = d
, k = m.
dt 1/k
em defined on (, 1 + ) by Lemma 8.6. We will
For each m, we have an extension X
em : m = 1, 2, . . . } is linearly independent by induction. Since X
em is
show that {X
em0 } is linearly independent for any m0 . Now suppose
nonzero, the set {X
em1 + + ar X
e mr = 0
a1 X
with m1 < < mr . Then
em1 |1/m + + ar X
emr |1/m = a1 X
em1 |1/m = 0,
a1 X
1
1
1
em1 |1/m 6= 0, and a2 = = ar = 0 by induction. Therefore
so a1 = 0 since X
1
em : m = 1, 2, . . . } is a set of linearly independent vector fields defined on U . We
{ X
em to the closed set ([0, 1])
can extend these vector fields to M by restricting each X
and applying Lemma 8.6. This proves that X(M ) is infinite-dimensional.

Theorem 121. [Problem 8-4] Let M be a smooth manifold with boundary. There
exists a global smooth vector field on M whose restriction to M is everywhere inwardpointing, and one whose restriction to M is everywhere outward-pointing.
Proof. Let n = dim M . Let {(U , )} be a collection of smooth boundary charts whose
domains cover M . For each ,Slet X be the nth coordinate vector field.
P Let { }
be a partition of unity of U = U subordinate to {U }; then X = X is a
smooth vector field defined on U . For any boundary defining function f : M [0, )
we have
X
Xp f =
X |p f > 0

since each X |p is inward-pointing, so Xp is always inward-pointing. By restricting X to


e whose restriction
M and applying Lemma 8.6, we have a global smooth vector field X
e to M is everywhere
to M is everywhere inward-pointing. Also, the restriction of X
outward-pointing.

Theorem 122. [Problem 8-5] Let M be a smooth n-manifold with or without boundary.
(1) If (X1 , . . . , Xk ) is a linearly independent k-tuple of smooth vector fields on an
open subset U M , with 1 k < n, then for each p U there exist smooth

41

vector fields Xk+1 , . . . , Xn in a neighborhood V of p such that (X1 , . . . , Xn ) is a


smooth local frame for M on U V .
(2) If (v1 , . . . , vk ) is a linearly independent k-tuple of vectors in Tp M for some
p M , with 1 k n, then there exists a smooth local frame (Xi ) on a
neighborhood of p such that Xi |p = vi for i = 1, . . . , k.
(3) If (X1 , . . . , Xn ) is a linearly independent n-tuple of smooth vector fields along
e1 , . . . , X
en ) on
a closed subset A M , then there exists a smooth local frame (X
ei |A = Xi for i = 1, . . . , n.
some neighborhood of A such that X
Proof. See Theorem 162.

Theorem 123. [Problem 8-6] Let H be the algebra of quaternions and let S H be
the group of unit quaternions.
(1) If p H is imaginary, then qp is tangent to S at each q S.
(2) Define vector fields X1 , X2 , X3 on H by
X1 |q = qi,

X2 |q = qj,

X3 |q = qk.

These vector fields restrict to a smooth left-invariant global frame on S.


(3) Under the isomorphism (x1 , x2 , x3 , x4 ) x1 1 + x2 i + x3 j + x4 k between R4 and
H, these vector fields have the following coordinate representations:

1
4
3
+
x
+
x

x
,
x1
x2
x3
x4

X2 = x3 1 x4 2 + x1 3 + x2 4 ,
x
x
x
x

X3 = x4 1 + x3 2 x2 3 + x1 4 .
x
x
x
x
X1 = x2

Proof. S is a level set of the norm squared ||2 : H (0, ), which is given by
|(a + bi, c + di)|2 = a2 + b2 + c2 + d2
and has derivative



2a 2b 2c 2d .

If q = (a, b, c, d) then by Theorem 65, Tq S is the kernel of this matrix. If p H is


imaginary then p has the form (ei, f + gi), so
qp = (be cf dg + (ae + cg df )i, af bg + de + (ag + bf ce)i)
and
a(be cf dg) + b(ae + cg df ) + c(af bg + de) + d(ag + bf ce) = 0.

42

Therefore qp Tq S. This proves (1). For (2), let g = (a, b, c, d) and g 0 = (e + f i, g + hi).
We compute

a b c d
b a d c
.
DLg (g 0 ) =
c d
a b
d c b
a
Then

(be + af dg + ch)
a b c d f
b a d c e ae bf cg dh
,
=

c d
a b h de cf + bg + ah
(ce + df + ag bh)
g
d c b
a
which shows that d(Lg )g0 (X1 |g0 ) = X1 |gg0 , i.e. X1 is left-invariant. Similar calculations
show that X2 and X3 are left-invariant.

Example 124. [Problem 8-9] Show by finding a counterexample that Proposition 8.19
is false if we replace the assumption that F is a diffeomorphism by the weaker assumption that it is smooth and bijective.
Consider the smooth bijection : [0, 1) S1 given by s 7 e2is and consider the
smooth vector field X given by x 7 (1 2x) d/dt|x . There is no way of defining an
F -related smooth vector field Y on S1 , since the sign of Y1 is ambiguous.
Example 125. [Problem 8-10] Let M be the open submanifold of R2 where both x
and y are positive, and let F : M M be the map F (x, y) = (xy, y/x). Show that F
is a diffeomorphism, and compute F X and F Y , where

X=x
+y , Y =y .
x
y
x
The inverse


u
F (u, v) =
, uv
v
is smooth, so F is a diffeomorphism. We compute


y
x
DF (x, y) =
y/x2 1/x
1

so that

r

p


uv
u/v
p
p
DF (F (u, v)) =
.
v v/u
v/u
Therefore the coordinates of F X are given by
p

 p
  
uv
u/v
2u
u/v
p
p

=
,
0
uv
v v/u
v/u
1

43

and the coordinates of F Y are given by


p
   


uv
u/v
uv
uv
p
p
=
.
0
v 2
v v/u
v/u
So
F X = 2u

,
u

F Y = uv

v2 .
u
v

Example 126. [Problem 8-11] For each of the following vector fields on the plane,
compute its coordinate representation in polar coordinates on the right half-plane
{(x, y) : x > 0}.

+y .
x
y

(2) Y = x
y .
x
y

(3) Z = (x2 + y 2 ) .
x

(1) X = x

Let M be the right half-plane.


We have a diffeomorphism F : M (0, ) (, )
p
2
2
given by F (x, y) = ( x + y , arctan(y/x)), with inverse F 1 (r, ) = (r cos , r sin ).
Then
 p

p
x/ x2 + y 2 y/ x2 + y 2
DF (x, y) =
,
y/(x2 + y 2 ) x/(x2 + y 2 )
so


cos
sin
1
DF (F (r, )) =
.
r1 sin r1 cos
We have


cos
sin
1
1
r sin r cos


 

r r cos 2 r2 cos
r cos r cos r2
.
=
r sin r sin 0
0 sin 2 r sin

Therefore
X=r

,
r

sin 2 ,
r

Z = r2 cos r sin .
r

Y = r cos 2

Example 127. [Problem 8-12] Let F : R2 RP2 be the smooth map F (x, y) = [x, y, 1],
and let X X(R2 ) be defined by X = x/y y/x. Prove that there is a vector
field Y X(RP2 ) that is F -related to X, and compute its coordinate representation in

44

terms of each of the charts defined in Example 1.5. Let : R3 \ {0} RP2 be the
e : R3 \ {0} T RP2 by
quotient map and define X
!



e(x,y,z) = d x


y
.
X
y (x,y,z)
x (x,y,z)
e is constant on the fibers of , it descends to a vector field Y : RP2 T RP2 .
Since X
ei R3 \ {0}, Ui RP2 and i : Ui Rn be as in Example 1.5:
Let U
i [x1 , x2 , x3 ] = (x1 /xi , x2 /xi , x3 /xi ),
1
2
3
1
i1
1
, 1, ui+1 , . . . , u3 ].
i (u , u , u ) = [u , . . . , u

The coordinate representations of d and Y with respect to 3 are given by


 1 2

u u
v1
u1 v 3 v 2
u2 v 3
1
2
3 1 2 3
d(u , u , u , v , v , v ) =
, , 1, 3 3 2 , 3 3 2 , 0
u3 u3
u
(u ) u
(u )
and


1
2
3
1
Y (u , u , u ) = d u
y

(u1 ,u2 ,1)



2
u
x

!
(u1 ,u2 ,1)

= (u1 , u2 , 1, u2 , u1 , 0).
On this chart, the coordinate representation of dF is
dF (u1 , u2 , v 1 , v 2 ) = (u1 , u2 , 1, v 1 , v 2 , 0),
so for all (x, y) R2 ,
dF(x,y) (X(x,y) ) = dF

!




x
y
y (x,y,z)
x (x,y,z)

= (x, y, 1, y, x, 0)
= Y (x, y, 1).
The computations for 1 and 2 are similar. This shows that Y is F -related to X.
Theorem 128. [Problem 8-13] There is a smooth vector field on S2 that vanishes at
exactly one point.
Proof. Let N = (0, 0, 1) be the north pole and let : S2 \{N } R2 be the stereographic
projection given by
(x1 , x2 , x3 ) = (x1 , x2 )/(1 x3 ),
1 (u1 , u2 ) = (2u1 , 2u2 , |u|2 1)/(|u|2 + 1).

45

Let X be the 1st coordinate vector field on R2 and define a vector field Y on S2 by
(
0,
p = N,
Y (p) =
1
(( ) X)p p 6= N.
This vector field vanishes at exactly one point. Let S = (0, 0, 1) be the south pole
and let : S2 \ {S} R2 be the stereographic projection given by
(x1 , x2 , x3 ) = (x1 , x2 )/(1 + x3 ),
1 (u1 , u2 ) = (2u1 , 2u2 , 1 |u|2 )/(|u|2 + 1).
Then
( 1 )(u1 , u2 ) = (2u1 , 2u2 )/(2 |u|2 ),
and the coordinate representation of Y for (u1 , u2 ) 6= (0, 0) with respect to is given
by
Y (u1 , u2 ) = d( 1 )( 1 )(u1 ,u2 ) X( 1 )(u1 ,u2 )
!


d

= d( 1 )( 1 )(u1 ,u2 )
dx ( 1 )(u1 ,u2 )


1 2
2 2
4u1 u2
4u1
1 1
2 2(1 (u ) + (u ) )
= (u , u ),
,
,
,
(1 + |u|2 )2
(1 + |u|2 )2 (1 + |u|2 )2
which shows that Y is smooth on S2 .

Theorem 129. [Problem 8-14] Let M be a smooth manifold with or without boundary,
let N be a smooth manifold, and let f : M N be a smooth map. Define F : M
M N by F (x) = (x, f (x)). For every X X(M ), there is a smooth vector field on
M N that is F -related to X.
Proof. Let m = dim M and n = dim N . By Lemma 8.6, it suffices to show that there is a
smooth vector field along F (M ) that is F -related to X. By Proposition 5.4, F (M ) is an
embedded m-submanifold of M N . Let p F (M ), let (U, ) be a slice chart for F (M )
e = (U ) Rm+n . By shrinking U , we may assume
containing p, let pe = (p), and let U
e is an open cube containing pe, and that (F (M ) U ) = {(a, b) Rm+n : b = 0}.
that U
e with U
e Rm+n . Define a vector field Y on U
e by assigning Y(a,b) the
We identify TpeU
coefficients of d(X(1 (a,0)) ), where : M N M is the canonical projection. Then
Y is smooth, and (1 ) Y is a smooth vector field defined on U that agrees with X on
F (M ) U .

Theorem 130. [Problem 8-15] Suppose M is a smooth manifold and S M is an
embedded submanifold with or without boundary. Given X X(S), there is a smooth
vector field Y on a neighborhood of S in M such that X = Y |S . Every such vector field
extends to all of M if S is properly embedded.

46

Proof. As in Theorem 82.

Example 131. [Problem 8-16] For each of the following pairs of vector fields X, Y
defined on R3 , compute the Lie bracket [X, Y ].

2xy 2 ; Y =
.
z
y
y

(2) X = x
y ; Y =y
z .
y
x
z
y

y ; Y =x
+y .
(3) X = x
y
x
y
x

(1) X = y

For (1), we have


X 2
= 2y 2 ,
x

X 2
= 4xy,
y

so
[X, Y ] = 4xy

X 3
= 1,
y

.
y z

For (2), we have


X 1
= 1,
y

X 2
= 1,
x

so
[X, Y ] = z

Y 2
= 1,
z

Y 3
= 1,
y

x .
x
z

For (3), we have


X 1
= 1,
y

X 2
= 1,
x

Y 1
= 1,
y

[X, Y ] = 2x

2y .
x
y

so

Y 2
= 1,
x

Theorem 132. [Problem 8-17] Let M and N be smooth manifolds. Given vector fields
X X(M ) and Y X(N ), we can define a vector field X Y on M N by
(X Y )(p,q) = (Xp , Yq ),
where we think of the right-hand side as an element of Tp M Tq N , which is naturally
identified with Tp,q (M N ) as in Proposition 3.14. Then X Y is smooth if X and Y
are smooth, and [X1 Y1 , X2 Y2 ] = [X1 , X2 ] [Y1 , Y2 ].
Proof. The smoothness of X Y follows easily from Proposition 8.1. If f C (M N )
then
[X1 Y1 , X2 Y2 ](p,q) f = (X1 Y1 )(p,q) (X2 Y2 )f (X2 Y2 )(p,q) (X1 Y1 )f

47

= (X1 |p X2 fq , Y1 |q Y2 fp ) (X2 |p X1 fq , Y2 |q Y1 fp )
= ([X1 , X2 ]fq , [Y1 , Y2 ]fp )
= ([X1 , X2 ] [Y1 , Y2 ])f,
where fq C (M ) is the map p 7 f (p, q) and fp C (N ) is the map q 7 f (p, q). 
Theorem 133. [Problem 8-18] Suppose F : M N is a smooth submersion, where M
and N are positive-dimensional smooth manifolds. Given X X(M ) and Y X(N ),
we say that X is a lift of Y if X and Y are F -related. A vector field V X(M ) is
said to be vertical if V is everywhere tangent to the fibers of F (or, equivalently, if V
is F -related to the zero vector field on N ).
(1) If dim M = dim N , then every smooth vector field on N has a unique lift.
(2) If dim M 6= dim N , then every smooth vector field on N has a lift, but it is not
unique.
(3) Assume in addition that F is surjective. Given X X(M ), X is a lift of a
smooth vector field on N if and only if dFp (Xp ) = dFq (Xq ) whenever F (p) =
F (q). If this is the case, then X is a lift of a unique smooth vector field.
Proof. Let m = dim M and n = dim N . If m = n then F is a local diffeomorphism,
and if Y X(N ) then we can define a smooth vector field X X(M ) by
Xp = (dFp )1 (YF (p) ).
This is clearly the only vector field that satisfies dFp (Xp ) = YF (p) , and smoothness
follows from Theorem 41 and the fact that dF is a local diffeomorphism. This proves
(1). Now suppose Y X(N ) and m 6= n, i.e. m > n. Let p M , let q = F (p), and
choose smooth coordinates (xi ) centered at p and (y i ) centered at q in which F has
the coordinate representation (x1 , . . . , xm ) = (x1 , . . . , xn ). If is a sufficiently small
number, the coordinate cube


Cp = x : xi < for i = 1, . . . , m
is a neighborhood of p whose image under F is the cube


Cp0 = y : y i < for i = 1, . . . , n .
Define a smooth vector field Xp on Cp by setting Xpi (x) = Y i (F (x)) for i = 1, . . . , n
and Xpi (x) = 0 for i = n + 1, . . . m. Then dFx (Xp |x ) = YF (x) for all x Cp . Choose a
partition of unity {p } subordinate to the open cover {Cp }. Then
X
X=
p Xp
pM

48

is a smooth vector field on M (taking p Xp = 0 on M \ supp p ), and


!
X
dFx (Xx ) = dFx
p (x)Xp |x
pM

p (x)dFx (Xp |x )

pM

p (x)YF (x)

pM

= YF (x)
for all x M . This proves (2). For (3), F is a quotient map. If X is a lift of some
Y X(N ) then for all p, q M ,
dFp (Xp ) = YF (p) = YF (q) = dFq (Xq ).
Conversely, if dFp (Xp ) = dFq (Xq ) whenever F (p) = F (q) then dF X is constant on the
fibers of F , so it descends to a smooth vector field Y on N satisfying dF X = Y F ,
i.e. dFp (Xp ) = YF (p) for all p M .

Theorem 134. [Problem 8-20] Let A X(R3 ) be the subspace spanned by {X, Y, Z},
where

z , Y =z
x , Z =x
y .
X=y
z
y
x
z
y
x
3
3
Then A is a Lie subalgebra of X(R ) which is isomorphic to R with the cross product.
Proof. Since {X, Y, Z} is linearly independent, it is a basis for A. Simple computations
similar to those in Example 131 show that A is closed under brackets. Define an
isomorphism : A R3 by setting X = (1, 0, 0), Y = (0, 1, 0), Z = (0, 0, 1), and
extending linearly. It is easy to check that is a Lie algebra isomorphism. For example,
[Z, X] = Y = [(0, 0, 1), (1, 0, 0)] = [Z, X].

Theorem 135. [Problem 8-21] Up to isomorphism, there are exactly one 1-dimensional
Lie algebra and two 2-dimensional Lie algebras. All three algebras are isomorphic to
Lie subalgebras of gl(2, R).
Proof. Any 1-dimensional Lie algebra is abelian, and is isomorphic to the subalgebra
{rI2 : r R} of gl(2, R). Let A be a 2-dimensional Lie algebra and let {x, y} be a basis
for A. If A is abelian then it is isomorphic to the subalgebra



r1 0
, r1 , r2 R
0 r2

49

of gl(2, R). Otherwise, we have [x, y] = ax + by for some a, b R, not both zero.
Assume without loss of generality that b 6= 0. (If b = 0, then we can swap x and y.)
Define an homomorphism : A gl(2, R) by setting




1
0
ab1
0
x =
, y =
0 1+b
1
ab1 (1 + b)
and extending linearly. Then is an isomorphism onto its image, and it is easy to
verify that [x, y] = [x, y].

Theorem 136. [Problem 8-22] Let A be any algebra over R. A derivation of A is a
linear map D : A A satisfying D(xy) = (Dx)y + x(Dy) for all x, y A. If D1 and
D2 are derivations of A, then [D1 , D2 ] = D1 D2 D2 D1 is also a derivation. The
set of derivations of A is a Lie algebra with this bracket operation.
Proof. Identical to Lemma 8.25.

Theorem 137. [Problem 8-23]


(1) Given Lie algebras g and h, the direct sum g h is a Lie algebra with the bracket
defined by
[(X, Y ), (X, Y 0 )] = ([X, X 0 ], [Y, Y 0 ]).
(2) Suppose G and H are Lie groups. Then Lie(G H) is isomorphic to Lie(G)
Lie(H).
Proof. We have isomorphisms : Lie(G H) Te (G H), 1 : Lie(G) Te (G) and
2 : Lie(H) Te (H). Thus we have an isomorphism
0 : Lie(G) Lie(H) Te (G) Te (H)
= Te (G H),
and it remains to show that 1 0 respects brackets. Let (X1 , X2 ), (Y1 , Y2 ) Lie(G)
Lie(H); then
(1 0 )[(X1 , X2 ), (Y1 , Y2 )](g,h) = ((1 0 )([X1 , Y1 ], [X2 , Y2 ]))(g,h)
= (1 ([X1 , Y1 ]e , [X2 , Y2 ]e ))(g,h)
= d(L(g,h) )(e,e) ([X1 , Y1 ] [X2 , Y2 ])(e,e)
= d(L(g,h) )(e,e) [X1 X2 , Y1 Y2 ](g,h)
= [X1 X2 , Y1 Y2 ](g,h)
= [(1 0 )(X1 , X2 ), (1 0 )(Y1 , Y2 )](g,h)
since [X1 X2 , Y1 Y2 ] is left-invariant (see Theorem 132).

Theorem 138. [Problem 8-24] Suppose G is a Lie group and g is its Lie algebra. A
vector field X X(G) is said to be right-invariant if it is invariant under all right
translations.

50

(1) The set g of right-invariant vector fields on G is a Lie subalgebra of X(G).


(2) Let i : G G denote the inversion map i(g) = g 1 . The pushforward i :
X(G) X(G) restricts to a Lie algebra isomorphism from g to g.
Proof. Part (1) follows from Corollary 8.31. If X g then for all g, g 0 G we have
d(Rg )g0 ((i X)g0 ) = (d(Rg )g0 dii1 (g0 ) )(Xi1 (g0 ) )
= d(Rg i)(g0 )1 (X(g0 )1 )
= d(i Lg1 )(g0 )1 (X(g0 )1 )
= di(g0 g)1 (d(Lg1 )(g0 )1 X(g0 )1 )
= di(g0 g)1 (X(g0 g)1 )
= (i X)g0 g .
Therefore i (g) g. A similar calculation shows that i (g) g, so (i )1 = i and i is
an isomorphism. Corollary 8.31 shows that i is a Lie algebra isomorphism.

Theorem 139. [Problem 8-25] If G is an abelian Lie group, then Lie(G) is abelian.
Proof. If G is abelian then the inversion map i : G G is a Lie group isomorphism.
The induced Lie algebra isomorphism is given by
(i X)g = d(Lg )e (die (Xe ))
= d(Lg )e (Xe )
= d(Lg )e Xe
= Xg ,
so i X = X for all X Lie(G). Then
[X, Y ] = [X, Y ] = [i X, i Y ] = i [X, Y ] = [X, Y ],
so [X, Y ] = 0 for all X, Y Lie(G). This proves that Lie(G) is abelian.

Theorem 140. [Problem 8-26] Suppose F : G H is a Lie group homomorphism.


The kernel of F : Lie(G) Lie(H) is the Lie algebra of ker F (under the identification
of the Lie algebra of a subgroup with a Lie subalgebra as in Theorem 8.46).
Proof. Consider X for some X ker F , where : ker F , G is the inclusion map.
Then F X = (F ) X = 0 since F = 0. This shows that (Lie(ker F )) ker F .
Conversely, suppose F Y = 0 for some Y Lie(G). By definition we have dFe (Ye ) = 0,
so Ye ker(dFe ) = Te (ker F ) by Theorem 65. By Theorem 8.46, this shows that
ker F (Lie(ker F )).


51

Theorem 141. [Problem 8-27] Let G and H be Lie groups, and suppose F : G
H is a Lie group homomorphism that is also a local diffeomorphism. The induced
homomorphism F : Lie(G) Lie(H) is an isomorphism of Lie algebras.
Proof. Since F is a local diffeomorphism, dFe is bijective. If F X = 0 then dFe (Xe ) = 0,
so Xe = 0 and X = 0. Suppose Y Lie(H). Let : Lie(G) Te G be the canonical
isomorphism. Then
(F 1 (dFe )1 Ye )e = dFe ((1 (dFe )1 Ye )e )
= dFe ((dFe )1 Ye )
= Ye ,
so F (1 (dFe )1 Ye ) = Y . This proves that F is a bijection.

Theorem 142. [Problem 8-28] Considering det : GL(n, R) R as a Lie group homomorphism, its induced Lie algebra homomorphism is tr : gl(n, R) R.
Proof. This follows immediately from Theorem 98.

Example 143. [Problem 8-29] Theorem 8.46 implies that the Lie algebra of any Lie
subgroup of GL(n, R) is canonically isomorphic to a subalgebra of gl(n, R), with a
similar statement for Lie subgroups of GL(n, C). Under this isomorphism, show that
Lie(SL(n, R))
= sl(n, R),

Lie(SO(n)) = o(n),
Lie(SL(n, C))
= sl(n, C),

Lie(U(n)) = u(n),
Lie(SU(n))
= su(n),
where
sl(n, R) = {A gl(n, R) : tr A = 0},
o(n) = {A gl(n, R) : AT + A = 0},
sl(n, C) = {A gl(n, C) : tr A = 0},
u(n) = {A gl(n, C) : A + A = 0},
su(n) = u(n) sl(n, C).
We have SL(n, R) = ker det, so Lie(SL(n, R))
= ker det = ker tr = sl(n, R). The same
computation holds for SL(n, C). The others follow by considering the kernel of the map
A 7 AT A (or A 7 A A in the complex case).

52

Example 144. [Problem 8-30] Show by giving an explicit isomorphism that su(2) and
o(3) are isomorphic Lie algebras, and that both are isomorphic to R3 with the cross
product.
We have an isomorphism from R3 to o(3) given by

a
0
c b
b 7 c 0
a .
c
b a 0
We also have an isomorphism from R3 to su(2) given by



a
1
ai
b
+
ci
b 7
.
b + ci ai
2
c
Theorem 145. [Problem 8-31] Let g be a Lie algebra. A linear subspace h g is called
an ideal in g if [X, Y ] h whenever X h and Y g.
(1) If h is an ideal in g, then the quotient space g/h has a unique Lie algebra
structure such that the projection : g g/h is a Lie algebra homomorphism.
(2) A subspace h g is an ideal if and only if it is the kernel of a Lie algebra
homomorphism.
Proof. Define a bracket on g/h by [X + h, Y + h] = [X, Y ] + h. If X 0 + h = X + h and
Y 0 + h = Y + h then
[X 0 , Y 0 ] = [X 0 + X X 0 , Y 0 ] = [X, Y 0 + Y Y 0 ] = [X, Y ]
since X X 0 h and Y Y 0 h, which shows that the bracket is well-defined. It is
clearly the unique bracket making a Lie algebra homomorphism. If h g is an ideal
then it is the kernel of the projection : g g/h. Conversely, if f is a Lie algebra
homomorphism then ker f is an ideal since X ker f implies that
f [X, Y ] = [f X, f Y ] = 0.

Chapter 9. Integral Curves and Flows
Theorem 146. [Exercise 9.37] Suppose v Rn and W is a smooth vector field on an
open subset of Rn . The directional derivative

d
Dv W (p) = Wp+tv
dt t=0
is equal to (LV W )p , where V is the vector field V = v i /xi with constant coefficients
in standard coordinates.

53

Proof. The flow of V is given by (t, x) = x + tv. Therefore



d
(LV W )p = d(t )t (p) (Wt (p) )
dt t=0

d
= d(t )p+tv (Wp+tv )
dt t=0

d
= Wp+tv
dt t=0
since t (x) = x tv is a translation of x.

Theorem 147. [Problem 9-1] Suppose M is a smooth manifold, X X(M ), and is


a maximal integral curve of X.
(1) We say that is periodic if there is a number T > 0 such that (t + T ) = (t)
for all t R. Exactly one of the following holds:
(a) is constant.
(b) is injective.
(c) is periodic and nonconstant.
(2) If is periodic and nonconstant, then there exists a unique positive number T
(called the period of ) such that (t) = (t0 ) if and only if t t0 = kT for
some k Z.
(3) The image of is an immersed submanifold of M , diffeomorphic to R, S1 , or
R0 .
Proof. If is constant then (b) and (c) cannot hold, so we assume that is nonconstant.
If is not injective then (t0 ) = (t1 ) for some points t0 < t1 , and is defined on at
least (t0 , t1 + ) for some > 0. Let T = t1 t0 . Then and t 7 (t1 + t) are both
integral curves of X starting at (t0 ), so Theorem 9.12 shows that must be defined
on at least (t0 , t1 + T + ). By induction, t0 + kT is in the domain of for all k Z,
so is defined on R and has period T . This proves (1). For (2), let A be the set of all
T > 0 such that (t + T ) = (t) for all t R, which is nonempty since is periodic.
If we can show that A is closed, then T0 = inf A A is the period of . But if T
/A
then (t + T ) 6= (t) for some t R, so
1 (M \ {(t)}) t = {s t : (s) 6= (t)}
is a neighborhood of T contained in R\A. For (3), if is constant then the image of is
diffeomorphic to R0 . Otherwise, Proposition 9.21 shows that is a smooth immersion.
If is injective then Proposition 5.18 shows that the image of is diffeomorphic to
R, and if is periodic then it descends to a smooth injective immersion from S1 to M
(using a suitable smooth covering of S1 by R), which is a diffeomorphism onto its image
by Proposition 5.18.


54

Theorem 148. [Problem 9-2] Suppose M is a smooth manifold, S M is an immersed


submanifold, and V is a smooth vector field on M that is tangent to S.
(1) For any integral curve of V such that (t0 ) S, there exists > 0 such that
((t0 , t0 + )) S.
(2) If S is properly embedded, then every integral curve that intersects S is contained
in S.
(3) (2) need not hold if S is not closed.
Proof. Let x = (t0 ) S. By Proposition 8.23, there is a smooth vector field V |S on S
that is -related to V . Let : I S be a maximal integral curve of V |S starting at x,
where I contains (, ) for some > 0. Then
( )0 (0) = dx (0 (0)) = dx ((V |S )x ) = Vx ,
which shows that is an integral curve of V starting at x. By Theorem 9.12, is
defined on J = (t0 , t0 + ) and (t) = ( )(t t0 ) S for t J. This proves (1).
Now suppose that S is properly embedded and let be an integral curve that intersects
S, i.e. (t0 ) S for some t0 . Let be defined as above. By Theorem 9.12, is defined
on at least I + t0 = (a, b) and (t) S for all t (a, b). Suppose can be extended to
an open interval J larger than (a, b). Since S is closed, (b) S by continuity. Then
(1) shows that ((b , b + )) S for some > 0, which contradicts the maximality
of . This proves (2). For (3), let S be the open ball of radius 1 in R and let V be the
1st coordinate vector field on R. Then : R R R given by (t, x) = x + t is an
integral curve of V that intersects S, but is not contained in S.

Example 149. [Problem 9-3] Compute the flow of each of the following vector fields
on R2 :

(1) V = y
+
.
x y

(2) W = x
+ 2y .
x
y

(3) X = x
y .
x
y

(4) Y = y
+x .
x
y
Let (t) = (x(t), y(t)) be a curve. We have the differential equations
x0 (t) = y(t), y 0 (t) = 1,
x0 (t) = x(t), y 0 (t) = 2y(t),
x0 (t) = x(t), y 0 (t) = y(t)
x0 (t) = y(t), y 0 (t) = x(t).

55

Therefore the flows (in this order) are




1 2
t (x, y) = x + ty + t , y + t ,
2
t
2t
t (x, y) = (xe , ye ),
t (x, y) = (xet , yet ),

1 t
t (x, y) =
e (x + y) + et (x y), et (x + y) et (x y) .
2
Theorem 150. [Problem 9-4] For any integer n 1, define a flow on the odddimensional sphere S2n1 Cn by (t, z) = eit z. The infinitesimal generator of
is a smooth nonvanishing vector field on S2n1 .
Proof. This is obvious.

Theorem 151. [Problem 9-5] Suppose M is a smooth, compact manifold that admits
a nowhere vanishing smooth vector field. There exists a smooth map F : M M that
is homotopic to the identity and has no fixed points.
Proof. Let V be a nowhere vanishing smooth vector field on M . Since M is compact,
Corollary 9.17 shows that there is a global flow : R M M of V . By Theorem
9.22, each p has a neighborhood Up in which V has the coordinate representation
/s1 . Choosing a sufficiently small neighborhood Vp of p, there is some Tp > 0 such
that t (x) = x + (t, 0, . . . , 0) in local coordinates for all 0 t Tp and x Vp .
Since {Vp : p M } is an open cover of M , there is a finite subcover {Vp1 , . . . , Vpn }. Let
T = min {Tp1 , . . . , Tpn }. Then T has no fixed points, and the map H : M I M
given by (x, t) 7 (tT, x) is a homotopy from the identity to T .

Theorem 152. [Problem 9-6] Suppose M is a smooth manifold and V X(M ). If
: J M is a maximal integral curve of V whose domain J has a finite least upper
bound b, then for any t0 J, ([t0 , b)) is not contained in any compact subset of M .
Proof. Let be the flow of V . Let {bn } be an increasing sequence contained in [t0 , b)
and converging to b. Since ([t0 , b)) is contained in a compact set E M , there is a
subsequence {(bnk )} of {(bn )} that converges to a point p in E. Choose some > 0
and a neighborhood U of p such that is defined on (2, 2) U , and choose an
integer m such that bm (b , b) and (bm ) U . Define 1 : [t0 , b + ) M by
(
(t),
t0 t < b,
1 (t) =
((bm ))

(t bm ) b t < b + .
For all t (bm , b) we have
((bm )) (t bm ) = tbm ((bm )) = (t),

56

so 1 is smooth because and t 7 ((bm )) (t bm ) agree where they overlap. But 1


extends at b, which contradicts the maximality of .

Theorem 153. [Problem 9-7] Let M be a connected smooth manifold. The group of
diffeomorphisms of M acts transitively on M : that is, for any p, q M , there is a
diffeomorphism F : M M such that F (p) = q.
Proof. First assume that M = Bn . By applying Lemma 8.6 to the map x 7 q p
defined on the line segment between p and q, we have a compactly supported smooth
vector field on Bn whose flow satisfies 1 (p) = q. Theorem 9.16 shows that 1 is a
diffeomorphism on Bn , which proves the result for M = Bn . The general case follows
from Theorem 29 and the fact that every point of M is contained in a coordinate
ball.

Theorem 154. [Problem 9-8] Let M be a smooth manifold and let S M be a compact
embedded submanifold. Suppose V X(M ) is a smooth vector field that is nowhere
tangent to S. There exists > 0 such that the flow of V restricts to a smooth embedding
: (, ) S M .
Proof. Let : D M be the flow of V and let = O where O = (R S) D. Let
: S R be as in Theorem 9.20. Since S is compact, attains a minimum on S, and
: (, )S M is a smooth immersion. Let = /2. Then [, ]S is compact,
so : [, ] S M is a smooth embedding. Therefore : (, ) S M is also
a smooth embedding.

Theorem 155. [Problem 9-9] Suppose M is a smooth manifold and S M is an
embedded hypersurface (not necessarily compact). Suppose further that there is a smooth
vector field V defined on a neighborhood of S and nowhere tangent to S. Then S has a
neighborhood in M diffeomorphic to (1, 1) S, under a diffeomorphism that restricts
to the obvious identification {0} S S.
Proof. Using Theorem 9.20, we have that |O is a diffeomorphism onto an open submanifold of M . So it remains to show that O = {(t, p) O : |t| < (p)} is diffeomorphic to (1, 1) S. Let : (1, 1) S O be given by (t, p) 7 (t(p), p); then is
smooth, and its inverse 1 (t, p) = (t/(p), p) is also smooth.

Example 156. [Problem 9-10] For each vector field in Example 149, find smooth
coordinates in a neighborhood of (1, 0) for which the given vector field is a coordinate
vector field.
We only consider the first vector field. We have V(1,0) = /y, so we can take S to be
the x-axis, parametrized by X(s) = (s, 0). Therefore


1 2
(t, s) = t (s, 0) = s + t , t ,
2

57

and


1 2
(x, y) = x y , y ;
2


1 y
1
D (x, y) =
.
0 1
1

In these coordinates, we have

+
x y



= s + s +
s
s t

= .
t

V =y

Chapter 10. Vector Bundles


Theorem 157. [Exercise 10.1] Suppose E is a smooth vector bundle over M . The
projection map : E M is a surjective smooth submersion.
Proof. Let v E and let : 1 (U ) U Rk be a smooth local trivialization of E
such that (v) U . Then U = , so dv = d(U )(v) dv is surjective since dv
and d(U )(v) are both surjective.

Theorem 158. [Exercise 10.9] The zero section of every vector bundle : E M
is continuous, and the zero section of every smooth vector bundle is smooth.
Proof. Let p M and let : 1 (U ) U Rk be a local trivialization of E such that
p U . We have ( )(x) = (x, 0) for all x U , so |U is continuous and |U is
continuous since is a homeomorphism. But p was arbitrary, so is continuous. A
similar argument holds if is a smooth vector bundle.

Theorem 159. [Exercise 10.11] Let : E M be a smooth vector bundle.
(1) If , (E) and f, g C (M ), then f + g (E).
(2) (E) is a module over the ring C (M ).
Proof. Let : 1 (U ) U Rk be a local trivialization of E; then
( (f + g ))(x) = (x, f (x)Rk ((x)) + g(x)Rk ( (x))) ,
so f + g is smooth on U . Therefore f + g is smooth.

58

Theorem 160. [Exercise 10.13] Let : E M be a smooth vector bundle over a


smooth manifold M with or without boundary. Suppose A is a closed subset of M ,
and : A E is a section of E|A that is smooth in the sense that extends to a
smooth local section of E in a neighborhood of each point. For each open subset U M
containing A, there exists a global smooth section
e (E) such that
e|A = and
supp
e U.
Proof. For each p A, choose a neighborhood Wp of p and a local section
ep : Wp E
of E that agrees with on Wp A. Replacing Wp by Wp U we may assume that
Wp U . The family of sets {Wp : p A} {M \ A} is an open cover of M . Let
{p : p A} {0 } be a smooth partition of unity subordinate to this cover, with
supp p Wp and supp 0 M \ A.
For each p A, the product p
ep is smooth on Wp by Theorem 159, and has a smooth
extension to all of M if we interpret it to be zero on M \ supp p . (The extended
function is smooth because the two definitions agree on the open subset Wp \ supp p
where they overlap.) Thus we can define
e : M E by
X
p (x)e
p (x).

e(x) =
pA

Because the collection of supports {supp p } is locally finite, this sum actually has only
a finite number of nonzero terms in a neighborhood of any point of M , and therefore
defines a smooth function. If x A, then 0 (x) = 0 and
ep (x) = (x) for each p such
that p (x) 6= 0, so
!
X
X
p (x) (x) = (x),
p (x)(x) = 0 (x) +

e(x) =
pA

pA

so
e is indeed an extension of . It follows from Lemma 1.13(b) that
[
[
supp
e=
supp p =
supp p U.
pA

pA


Theorem 161. [Exercise 10.14] Let : E M be a smooth vector bundle. Every
element of E is in the image of a smooth global section.
Proof. Let v E and let p = (v). The assignment p 7 v is a section of E|{p} , which
is easily seen to be smooth in the sense of Theorem 160.

Theorem 162. [Exercise 10.16] Suppose : E M is a smooth vector bundle of rank
k.

59

(1) If (1 , . . . , m ) is a linearly independent m-tuple of smooth local sections of E


over an open subset U M , with 1 m < k, then for each p U there exist
smooth sections m+1 , . . . , k defined on some neighborhood V of p such that
(1 , . . . , k ) is a smooth local frame for E over U V .
(2) If (v1 , . . . , vm ) is a linearly independent m-tuple of elements of Ep for some
p M , with 1 m k, then there exists a smooth local frame (i ) for E over
some neighborhood of p such that i (p) = vi for i = 1, . . . , m.
(3) If A M is a closed subset and (1 , . . . , k ) is a linearly independent k-tuple
of sections of E|A that are smooth in the sense described in Theorem 160, then
there exists a smooth local frame (1 , . . . , k ) for E over some neighborhood of
A such that i |A = i for i = 1, . . . , k.
Proof. Choose a smooth local trivialization : 1 (V ) V Rk such that p V . By
shrinking V , we may assume there is a coordinate map : V Rn and that V U ;
let (xi ) be the coordinates. Let vi = (Rk i )(p) for i = 1, . . . , m, and complete
{v1 , . . . , vm } to a basis {v1 , . . . , vk } of Rk . For each i = m + 1, . . . , k, let i : V E
be the local section of E defined by i (x) = 1 (x, vi ). Let W be the subspace of Rk
spanned by {vm+1 , . . . , vk }; then F = 1 (V (Rn \ W )) is open, so Vi = i1 (F ) is a
neighborhood of p for each i = 1, . . . , m. Let Ve = V V1 Vm . Then {1 , . . . , k }
is a smooth local frame for E on Ve since (Rk i )(x)
/ W for all x Ve and
i = 1, . . . , k. This proves (1). Part (2) follows by choosing constant coefficient local
sections i such that i (p) = vi on a sufficiently small neighborhood of p. For (3), apply
Theorem 160 to each i and restrict to a sufficiently small neighborhood of A.

Theorem 163. [Exercise 10.27] A smooth rank-k vector bundle over M is smoothly
trivial if and only if it is smoothly isomorphic over M to the product bundle M Rk .
Proof. If : E M is smoothly trivial then there is a trivialization : E M Rk .
This map satisfies M = , so E and M Rk are smoothly isomorphic over M . The
converse is similar.

Theorem 164. [Exercise 10.31] Given a smooth vector bundle : E M and a
smooth subbundle D E, the inclusion map : D , E is a smooth bundle homomorphism over M .
Proof. This is simply the condition = |D .

Theorem 165. [Problem 10-1] The Mobius bundle E is not trivial.


Proof. E is not homeomorphic to S1 R. Removing the center circle S1 {0} from
S1 R leaves a disconnected space, but removing the center circle from E leaves a
connected space.


60

Theorem 166. [Problem 10-2] Let E be a vector bundle over a topological space M .
The projection map : E M is a homotopy equivalence.
Proof. Let : M E be the zero section. Then = IdM , and H : E I E
defined by H(v, t) = t IdE is a homotopy from to IdE . This shows that is a
homotopy equivalence.

Theorem 167. [Problem 10-3] Let VB denote the category whose objects are smooth
vector bundles and whose morphisms are smooth bundle homomorphisms, and let Diff
denote the category whose objects are smooth manifolds and whose morphisms are
smooth maps. The assignment M 7 T M , F 7 dF defines a covariant functor from
Diff to VB, called the tangent functor.
Proof. This follows immediately from Corollary 3.22.

Theorem 168. [Problem 10-4] The map : U V GL(k, R) in Lemma 10.5 is


smooth.
Proof. We know that : (U V ) Rk Rk given by (p, v) = (p)v is smooth.
Let {e1 , . . . , ek } be the standard basis for Rk . The (i, j) entry of (p) is i ( (p)ej ) =
i ((p, ej )), where i : Rk R is the projection onto the ith coordinate. This map is
smooth as a function of p, so is smooth.

Theorem 169. [Problem 10-5] Let : E M be a smooth vector bundle of rank k over
a smooth manifold M with or without boundary. Suppose that {U }A is an open cover
of M , and for each A we are given a smooth local trivialization : 1 (U )
U Rk of E. For each , A such that U U 6= , let : U U GL(k, R)
be the transition function defined by (10.3). The following identity is satisfied for all
, , A:
(p) (p) = (p),
p U U U .
Proof. This follows from the fact that
(p, (p)v) = ( 1
)(p, v)
1
= ( 1
)(p, v)

= (p, (p) (p)v).



Theorem 170. [Problem 10-6] Let M be a smooth manifold with or without boundary,
and let {U }A be an open cover of M . Suppose for each , A we are given a
smooth map : U U GL(k, R) such that
(p) (p) = (p),

p U U U

61

is satisfied for all , , A. Then there is a smooth rank-k vector bundle E M


with smooth local trivializations : 1 (U ) U Rk whose transition functions
are the given maps .
`
Proof. Note that = IdRk for all A. Let F = A U Rk and let i : U Rk
F be the canonical injection for each A. Define an equivalence relation on F by
declaring that (p, v) (p0 , v 0 ) if and only if p = p0 and (p)v 0 = v. This relation is
symmetric because
(p) (p) = (p) = IdRk ,
0
so (p)v = v implies that v 0 = (p)v. Similarly, the relation is transitive because
(p)v 0 = v and (p)v 00 = v 0 implies that
(p)v 00 = (p) (p)v 00 = (p)v 0 = v.

Let E = F/ and let q : F E be the quotient map. Let Ep = q {p} Rk for each
p M ; we want to give Ep a vector space structure. Define m
e : R q 1 (Ep ) Ep
by m(r, (p, v) ) = q((p, rv) ). If (p, v) (p, v 0 ) then (p)v 0 = v and (p)rv 0 =
r (p)v 0 = rv. Therefore m
e descends to a continuous map m : R Ep Ep , which is
scalar multiplication in Ep . A similar construction shows that vector addition can be
defined on Ep .
Let
e : F M be given by (p, v) 7 p; then
e descends to a continuous map : E M
satisfying q =
e. For each A, define : 1 (U ) U Rk as the unique
bijection satisfying q i = IdU Rk . We now check conditions (ii) and (iii) in
Lemma 10.6. For (ii), it is clear that |Ep is linear, so it is an isomorphism. For (iii),
we have
( 1
)(p, v) = (q((p, v) ))
= (q((p, (p)v) ))
= (p, (p)v).
Lemma 10.6 now shows that : E M is a smooth rank-k vector bundle, with
{(U , )} as smooth local trivializations.

Example 171. [Problem 10-7] Compute the transition function for T S2 associated
with the two local trivializations determined by stereographic coordinates.
See Theorem 128.
Theorem 172. [Problem 10-8] Let Vec1 be the category whose objects are finite-dimensional
real vector spaces and whose morphisms are linear isomorphisms. If F is a covariant
functor from Vec1 to itself, for each finite-dimensional vector space V we get a map
F : GL(V ) GL(F(V )) sending each isomorphism A : V V to the induced isomorphism F(A) : F(V ) F(V ). We say F is a smooth functor if this map is
smooth for every V . Given a smooth vector bundle : E M and a smooth functor

62

F : Vec1 Vec1 , there is a smooth vector bundle


e : F(E) M whose fiber at each
point p M is F(Ep ).
`
Proof. Let k be the rank of . Define F(E) = pM F(Ep ) and let
e : F(E) M
be the projection. Let {Up }pM be an open cover of M where each Up contains p and
has an associated local trivialization p : 1 (Up ) Up Rk . Choose an arbitrary
isomorphism : F(Rk )
= Rn , where n is the dimension of F(Rk ). For each p M ,
ep :
ex =
define
e1 (Up ) Up Rn as follows: for each x Up and v E
e1 ({x}), set
e p (v) = (x, ( F(p |Ex ))(v)) where

F(p |Ex ) : F(Ex ) F({x} Rk )


= F(Rk )
is an isomorphism. For each p, q M with Up Uq 6= , we have
ep
e 1 )(x, v) = (x, ( F(p |Ex ) F(q |Ex )1 1 )(v))
(
q
= (x, ( F(p |Ex (q |Ex )1 ) 1 )(v))
= (x, pq (x)v)
for some smooth pq since F is a smooth functor. Applying Lemma 10.6 completes the
proof.

Theorem 173. [Problem 10-9] Suppose M is a smooth manifold, E M is a smooth
vector bundle, and S M is an embedded submanifold with or without boundary. For
any smooth section of the restricted bundle E|S S, there exist a neighborhood U of
S in M and a smooth section
e of E|U such that =
e|S . If E has positive rank, then
every smooth section of E|S extends smoothly to all of M if and only if S is properly
embedded.
Proof. As in Theorem 82.

Theorem 174. [Problem 10-11] Suppose E and E 0 are smooth vector bundles over a
smooth manifold M with or without boundary, and F : E E 0 is a bijective smooth
bundle homomorphism over M . Then F is a smooth bundle isomorphism.
Proof. Let k be the rank of E and let k 0 be the rank of E 0 ; let : E M and
0 : E 0 M be the projections. We want to show that F 1 is smooth. Let v E
and let : 1 (U ) U Rk be a smooth local trivialization of E satisfying (v) U
0
and U = . Let 0 : ( 0 )1 (U 0 ) U 0 Rk be a smooth local trivialization of E 0
satisfying 0 (F (v)) U 0 and U 0 0 = 0 . Consider the smooth map f = 0 F 1
with its inverse f 1 = F 1 (0 )1 defined on a sufficiently small neighborhood
e 0 Rk0 of 0 (F (v)). For each x U the map f |{x}Rk is an isomorphism whose inverse
U
is f 1 |{y(x)}Rk0 , where y(x) = 0 (F ((x))) and : M E is the zero section. Since

63

e 0 Rk0 . Therefore F 1
operator inversion is smooth and y is smooth, f 1 is smooth on U
is smooth on a neighborhood of F (v), and since v was arbitrary, F 1 is smooth.

e M be two smooth rank-k
Theorem 175. [Problem 10-12] Let : E M and
e:E
vector bundles over a smooth manifold M with or without boundary. Suppose {U }A
e admit smooth local trivializations over
is an open cover of M such that both E and E
each U . Let { } and {e
} denote the transition functions determined by the given
e respectively. Then E and E
e are smoothly isomorphic
local trivializations of E and E,
over M if and only if for each A there exists a smooth map : U GL(k, R)
such that
(*)

e (p) = (p) (p) (p)1 ,

p U U .

e Let A and let


Proof. Suppose there is a smooth bundle isomorphism F : E E.
1
k
1
k
e :
: (U ) U R and
e (U ) U R be the associated smooth local
e F =
e F : 1 (U ) U Rk
e F = , the map
trivializations. Since U
is a smooth local trivialization of E. Therefore
e F 1
(
)(p, v) = (p, (p)v)
for some smooth linear map : U GL(k, R). Now let , A. We have
e
e 1 )(p, v)
(p, e (p)v) = (

1
1
e F 1
e 1 )(p, v)
= (

= (p, (p) (p) (p)1 v),


so e (p) = (p) (p) (p)1 . Conversely, suppose that (*) holds. For each A,
e 1 , where (p, v) =
define a map F : 1 (U )
e1 (U ) by F =

(p, (p)v). If , A and U U 6= then


e 1
F =

e 1
e
e 1 1
=

e 1
=

= F
on 1 (U ) 1 (U ) by (*), so the gluing lemma shows that there is a smooth map
e such that F |1 (U ) = F for all A. The restriction of F to each fiber
F :EE

is clearly linear. On any 1 (U ) we have


e 1

eF =
e

= U
= U

64

= ,
so
e F = on E. By Theorem 174, F is a smooth bundle isomorphism.

Theorem 176. [Problem 10-14] Suppose M is a smooth manifold with or without


boundary, and S M is an immersed submanifold with or without boundary. Identifying Tp S as a subspace of Tp M for each p S in the usual way, T S is a smooth
subbundle of T M |S .
Proof.SLet : S , M be the inclusion map. For each p S, let Dp = dp (Tp S). Let
D = pS Dp = T S T M |S . Let p S, let V be a neighborhood of p in S that is an
embedded submanifold of M , and let (U, ) be a slice chart for V containing p. Write
= (x1 , . . . , xn ) and let k be the dimension of S. Then V U is neighborhood of p in
S, and the maps i : V U T M |S given by i (q) = dq (/xi |q ) for i = 1, . . . , k
satisfy the condition in Lemma 10.32.

Theorem 177. [Problem 10-17] Suppose M Rn is an immersed submanifold. The
ambient tangent bundle T Rn |M is isomorphic to the Whitney sum T M N M , where
N M M is the normal bundle.
Proof. Obvious.

Theorem 178. [Problem 10-19] Suppose : E M is a fiber bundle with fiber F .


(1)
(2)
(3)
(4)

is an open quotient map.


If the bundle is smooth, then is a smooth submersion.
is a proper map if and only if F is compact.
E is compact if and only if both M and F are compact.

Proof. For (1), let V be open in E. Let x (V ) and let : 1 (U ) U F be


a local trivialization of E such that x U . Choose any f F and a neighborhood
Ve 1 (U ) of 1 (x, f ). Then (Ve ) is open in U F since is a homeomorphism,
and U ((Ve )) = (Ve ) is a neighborhood of x contained in (V ). This shows that
is open, and is a quotient map since it is surjective. For (2), let v E and let
: 1 (U ) U F be a smooth local trivialization of E such that (v) U . Then
U = , so dv = d(U )(v) dv is surjective since dv and d(U )(v) are both
surjective. This shows that is a smooth submersion. For (3), suppose that is
proper. If x M then 1 ({x}) {x} F F is compact. Conversely, suppose that
F is compact and A M is compact. For each p A, choose a local trivialization
: 1 (Up ) Up F such that p Up , and choose a precompact neighborhood Vp
of p such that Vp Up . Since A is compact, the open cover {Vp } has a finite subcover

65

{Vp1 , . . . , Vpk }. Then 1 (A) is a closed subset of


k
[
i=1

(Vpi )
=
1

k
[

Vpi F,

i=1

which is compact. Therefore (A) is compact. For (4), if E is compact then M =


(E) is compact and F is compact by (3) since is proper. Conversely, if M and F
are compact then is proper by (3), so E = 1 (M ) is compact.

Chapter 11. The Cotangent Bundle
Theorem 179. [Exercise 11.10] Suppose M is a smooth manifold and E M is a
smooth vector bundle over M . Define the dual
bundle to E to be the bundle E M
`
whose total space is the disjoint union E = pM Ep , where Ep is the dual space to Ep ,
with the obvious projection. Then E M is a smooth vector bundle, whose transition
functions are given by (p) = ( (p)1 )T for any transition function : U GL(k, R)
of E.
Proof. Since

(p)
(p) = ( (p)1 )T ( (p)1 )T

= (( (p) (p))1 )T

=
(p),

we can apply Theorem 170.

Theorem 180. [Exercise 11.12] Let M be a smooth manifold with or without boundary,
and let : M T M be a rough covector field.
(1) is smooth.
(2) In every smooth coordinate chart, the component functions of are smooth.
(3) Each point of M is contained in some coordinate chart in which has smooth
component functions.
(4) For every smooth vector field X X(M ), the function (X) is smooth on M .
(5) For every open subset U M and every smooth vector field X on U , the function
(X) : U R is smooth on U .
Proof. (1) (2) (3) is clear. If (3) holds for a coordinate chart (U, (xi )) and the
natural coordinates (xi , i ) on 1 (U ) T M then the coordinate representation of
on U is

b (x) = (x1 , . . . , xn , 1 (x), . . . , n (x)),


which is smooth if 1 , . . . , n are smooth. This proves (3) (1). This also implies
(3) (4). Suppose that (4) holds, U M is open and X is a smooth vector field on U .

66

For each p U , choose a smooth bump function that is equal to 1 on a neighborhood


e = X, extended to be zero on M \ supp . Then
of p and supported in U . Define X
e is smooth and is equal to (X) in a neighborhood of p. So (X) is smooth in a
(X)
neighborhood of every point in U , which proves (4) (5). If (5) holds then (2) holds,
since i = (/xi ) for any smooth chart (U, (xi )).

Theorem 181. [Exercise 11.16] Let M be a smooth manifold with or without boundary,
and let be a rough covector field on M . If (i ) is a smooth coframe on an open subset
U M , then is smooth on U if and only if its component functions with respect to
(i ) are smooth.
Proof. This follows from Proposition 10.22.

Example 182. [Exercise 11.17] Let f (x, y) = x2 on R2 , and let X be the vector field

.
x
Compute the coordinate expression for X in polar coordinates (on some open subset
on which they are defined) and show that it is not equal to
X = grad f = 2x

f
f
+
.
r r
Set x = r cos and y = r sin ; then f (r, ) = r2 cos2 , so
f
f

+
= 2r cos2 2r2 sin cos .
r r
r

The derivative of the transition map (r, ) = (r cos , r sin ) is




cos r sin
.
sin r cos
Therefore




f
f
X = 2r cos cos + sin
6=
+
.
r

r r

Theorem 183. [Exercise 11.21] Let M be a smooth manifold with or without boundary,
and let f, g C (M ).
(1)
(2)
(3)
(4)

If a and b are constants, then d(af + bg) = a df + b dg.


d(f g) = f dg + g df .
d(f /g) = (g df f dg)/g 2 on the set where g 6= 0.
If J R is an interval containing the image of f , and h : J R is a smooth
function, then d(h f ) = (h0 f ) df .
(5) If f is constant, then df = 0.

67

Proof. For (1), we have


d(af + bg)p (v) = v(af + bg) = avf + bvg = (a dfp + b dgp )(v).
For (2), we have
d(f g)p (v) = v(f g) = f (p)vg + g(p)vf = (f dgp + g dfp )(v).
For (3), if p M and (U, (xi )) is a smooth chart containing p then on U ,
d(f /g)x =

n
X
(f /g)
i=1

xi

(x) dxi |x


n 
f
1 X
g
g(x) i (x) f (x) i (x) dxi |x
=
g(x)2 i=1
x
x
= (g(x) dfx f (x) dgx )/g(x)2 .
For (4), we have
d(h f )p (v) = dhf (p) (dfp (v)) = h0 (f (p))dfp (v).
For (5), if (U, (xi )) is a smooth chart and fb is the coordinate representation of f on U
then Dfb = 0, so fb is constant and f is constant on U . The result follows from Theorem
29.

Theorem 184. [Exercise 11.24] For a smooth real-valued function f : M R, p M
is a critical point of f if and only if dfp = 0.
Proof. Obvious.

Theorem 185. [Exercise 11.35] Let M be a smooth manifold with or without boundary.
Suppose : [a, b] M is a piecewise smooth curve segment, and , 1 , 2 X (M ).
(1) For any c1 , c2 R,

(c1 1 + c2 2 ) = c1

1 + c2

(2) If is a constant map, then = 0.


(3) If 1 = |[a,c] and 2 = |[c,b] with a < c < b, then

=
+
.

2 .

(4) If F : M N is any smooth map and X (N ), then

.
F =

68

Proof. We can assume that is smooth rather than piecewise smooth. For (1),

(c1 1 + c2 2 ) =
(c1 1 + c2 2 )

[a,b]

= c1
1 + c2
2
[a,b]
[a,b]

= c1 1 + c2 2

since is linear. For (2), if is constant then d = 0 and = 0. For (3),

[a,b]


+
=
[c,b]
[a,c]

=
+
.
1

For (4),

F =

(F ) =

F =
[a,b]

.
F

[a,b]


Theorem 186. [Exercise 11.41] A smooth covector field is conservative
if and only

if its line integrals are path-independent, in the sense that = e whenever and

e are piecewise smooth curve segments with the same starting and ending points.
Proof. Suppose that is conservative, is defined on [a, b], and
e is defined on [c, d].
Define
e : [a, b + d c] M by
(
(t),
0 t b,
(
e)(t) =

e(b + d t), b t b + d c.
Then
e is piecewise smooth, so

0=
e

by Theorem 185. The converse is obvious since any piecewise smooth closed curve
segment has the same starting and ending points as a constant curve segment.

Theorem 187. [Problem 11-1]
(1) Suppose V and W are finite-dimensional vector spaces and A : V W is any
linear map. The following diagram commutes:

69

(A )

where V and W denote the natural isomorphisms.


(2) There does not exist a way to assign to each finite-dimensional vector space V
an isomorphism V : V V such that for every linear map A : V W , the
following diagram commutes:
V

V
V

W
W

Proof. For (1), we have


((A ) (V v)) = (V v)(A ) = (A )v = (Av) = (W (Av)).
For (2), suppose that there are such isomorphisms V . Take V = W = R. The diagram
commutes when A = 0, so
R = 0 R 0 = 0.
This contradicts the fact that R is an isomorphism.

Theorem 188. [Problem 11-3] Let Vec1 be the category of finite-dimensional vector
spaces and linear isomorphisms as in Theorem 172. Define a functor F : Vec1 Vec1
by setting F(V ) = V for a vector space V , and F(A) = (A1 ) for an isomorphism
A. Then F is a smooth covariant functor, and for every M , F(T M ) and T M are
canonically smoothly isomorphic vector bundles.
Proof. F is covariant because
((A B)1 ) = (B 1 A1 ) = (A1 ) (B 1 ) ,
and it is smooth because matrix inversion and transposition are smooth. So Theorem
172 shows that F(T M ) exists, and we can define a smooth vector bundle isomorphism
: F(T M ) T M by setting |Tp M = IdTp M for each p M .

Theorem 189. [Problem 11-4] Let M be a smooth manifold with or without boundary
and let p be a point of M . Let Jp denote the subspace of C (M ) consisting of smooth

70

functions that vanish at p, and let Jp2 be the subspace of Jp spanned by functions of the
form f g for some f, g Jp .
(1) f Jp2 if and only if in any smooth local coordinates, its first-order Taylor
polynomial at p is zero.
(2) Define a map : Jp Tp M by setting (f ) = dfp . The restriction of to Jp2
is zero, and descends to a vector space isomorphism from Jp /Jp2 to Tp M .
P
Proof. If f Jp2 then f = ki=1 i gi hi for some i R and gi , hi Jp . If in smooth
local coordinates we have Df1 (p) = Df2 (p) = 0, then
D(gi hi )(p)u = gi (p)[Dhi (p)u] + [Dgi (p)u]hi (p) = 0,
so the first-order Taylor polynomial of f is zero by linearity. Conversely, if Df (p) = 0
then Theorem C.15 shows that locally we can write
1
n
X
2f
i
i
j
j
(1 t) i j (p + t(x p)) dt,
f (x) =
(x p )(x p )
x x
0
i,j=1
which is an element of Jp2 since xi pi and xj pj times the integral are elements of
P
Jp . Therefore f = ki=1 i gi hi for some i R and gi , hi Jp in a neighborhood U
of p. Choose a precompact neighborhood V of p with V U and choose a partition
of unity {0 , 1 } subordinate to the open cover U, M \ V . We take gi = hi = 0 on
M \ supp 0 . Then 1 , f Jp and
f=

k
X

i (0 gi )hi + 1 f,

i=1

which proves (1). Part (2) follows immediately.

Theorem 190. [Problem 11-5] For any smooth manifold M , T M is a trivial vector
bundle if and only if T M is trivial.
Proof. If T M is trivial then it admits a smooth global frame; Lemma 11.14 shows that
its dual coframe is a smooth global coframe for M , so T M is trivial. The converse is
similar.

Theorem 191. [Problem 11-6] Suppose M is a smooth n-manifold, p M , and
y 1 , . . . , y k are smooth real-valued functions defined on a neighborhood of p in M .
(1) If k = n and (dy 1 |p , . . . , dy n |p ) is a basis for Tp M then (y 1 , . . . , y n ) are smooth
coordinates for M in some neighborhood of p.
(2) If (dy 1 |p , . . . , dy k |p ) is a linearly independent k-tuple of covectors and k < n,
then there are smooth functions y k+1 , . . . , y n such that (y 1 , . . . , y n ) are smooth
coordinates for M in a neighborhood of p.

71

(3) If (dy 1 |p , . . . , dy k |p ) span Tp M , there are indices i1 , . . . , in such that (y i1 , . . . , y in )


are smooth coordinates for M in a neighborhood of p.
Proof. Part (1) follows from the inverse function theorem. For (2), complete (dy 1 |p , . . . , dy k |p )
to a basis (dy 1 |p , . . . , dy k |p , k+1 , . . . , n ), choose smooth functions y i such that dy i |p =
i for i = k + 1, . . . , n, and apply (1). For (3), choose indices i1 , . . . , in such that
dy i1 |p , . . . , dy in |p is a basis for Tp M and apply (1).

Example 192. [Problem 11-7] In the following problems, M and N are smooth manifolds, F : M N is a smooth map, and X (N ). Compute F in each case.
(1) M = N = R2 ; F (s, t) = (st, et ); = x dy y dx.
(2) M = R2 and N = R3 ; F (, ) = ((cos + 2) cos , (cos + 2) sin , sin );
= z 2 dx.

(3) M = {(s, t) R2 : s2 + t2 < 1} and N = R3 \ {0}; F (s, t) = (s, t, 1 s2 t2 );


= (1 x2 y 2 ) dz.
For (1),
dx = t ds + s dt,

dy = et dt,

so
= st(et dt) et (t ds + s dt) = tet ds + set (t 1) dt.
For (2),
dx = (cos + 2) sin d sin cos d,
so
= sin2 ((cos + 2) sin d sin cos d).
For (3),
dx = ds,
dy = dt,
dz =
so

s
t
ds +
dt,
2
2
1s t
1 s2 t2

= s 1 s2 t2 ds t 1 s2 t2 dt.

Theorem 193. [Problem 11-8]


(1) Suppose F : M N is a diffeomorphism, and let dF : T N T M be the
map whose restriction to each cotangent space Tq N is equal to dFF 1 (q) . Then
dF is a smooth bundle homomorphism.

72

(2) Let Diff 1 be the category whose objects are smooth manifolds, but whose only
morphisms are diffeomorphisms; and let VB be the category whose objects are
smooth vector bundles and whose morphisms are smooth bundle homomorphisms.
The assignment M 7 T M , F 7 dF defines a contravariant functor from Diff 1
to VB, called the cotangent functor.
Proof. It is clear that dF is a smooth bundle homomorphism covering F 1 . Since

d(F G)p = (dFG(p) dGp ) = dGp dFG(p)

we have d(F G) = dG dF . Also, d(IdM ) = IdT M . This shows that M 7 T M ,


F 7 dF defines a contravariant functor.

Example 194. [Problem 11-9] Let f : R3 R be the function f (x, y, z) = x2 + y 2 + z 2 ,
and let F : R2 R3 be the following map (the inverse of the stereographic projection):


2u
2v
u2 + v 2 1
,
,
F (u, v) =
.
u2 + v 2 + 1 u2 + v 2 + 1 u2 + v 2 + 1
Compute F df and d(f F ) separately, and verify that they are equal.
We have
df = 2x dx + 2y dy + 2z dz
and
4uv
2(u2 + v 2 + 1) 4u2
du
+
dv,
(u2 + v 2 + 1)2
(u2 + v 2 + 1)2
4uv
2(u2 + v 2 + 1) 4v 2
dy = 2
du
+
dv,
(u + v 2 + 1)2
(u2 + v 2 + 1)2
4u
4v
dz = 2
du + 2
dv,
2
2
(u + v + 1)
(u + v 2 + 1)2

dx =

so
df = 0.
Alternatively we have f F = 1, so d(f F ) = 0.
Example 195. [Problem 11-10] In each of the cases below, M is a smooth manifold
and f : M R is a smooth function. Compute the coordinate representation for df ,
and determine the set of all points p M at which dfp = 0.
(1) M = {(x, y) R2 : x > 0}; f (x, y) = x/(x2 + y 2 ). Use standard coordinates
(x, y).
(2) M and f are as in (1); this time use polar coordinates (r, ).
(3) M = S2 R3 ; f (p) = z(p) (the z-coordinate of p as a point in R3 ). Use north
and south stereographic coordinates.
(4) M = Rn ; f (x) = |x|2 . Use standard coordinates.

73

For (1),
x2 + y 2
2xy
dx + 2
dy,
2
2
2
(x + y )
(x + y 2 )2
which is zero when x2 = y 2 and xy = 0, i.e. never on M . For (2) we have
df =

f (r, ) =

cos
,
r

so
cos
sin
d,
dr
2
r
r
which is zero when cos = sin = 0, i.e. never on M . For (3) we have
df =

u2 + v 2 1
f (u, v) = 2
u + v2 + 1
on S2 \ N using the north stereographic projection, so
df =

(u2

4v
4u
du + 2
dv,
2
2
+ v + 1)
(u + v 2 + 1)2

which is zero when u = v = 0, i.e. when p is the south pole (0, 0, 1). A similar
calculation using the south stereographic projection shows that dfp = 0 when p is the
north pole (0, 0, 1). For (4) we have
df = 2x1 dx1 + + 2xn dxn ,
which is zero when p = 0.
Theorem 196. [Problem 11-11] Let M be a smooth manifold, and C M be an
embedded submanifold. Let f C (M ), and suppose p C is a point at which f
attains a local maximum or minimum value among points in C. Given a smooth local
defining function : U Rk for C on a neighborhood U of p in M , there are real
numbers 1 , . . . , k (called Lagrange multipliers) such that
dfp = 1 d1 |p + + k dk |p .
Proof. Let n be the dimension of M ; then C has dimension n k. Let (V, ) be a
smooth slice chart for C in M centered at p such that V U , let fe = f 1 , let
e = 1 , and let pe = (p). It suffices to show that there are real numbers 1 , . . . , k

such that
e 1 (p) + + k D
e k (p).
Dfe(p) = 1 D
Since 1 (0) V C, this follows from the method of Lagrange multipliers on Rn . 
Theorem 197. [Problem 11-12] Any two points in a connected smooth manifold can
be joined by a smooth curve segment.

74

Proof. Let M be a connected smooth n-manifold. Define an equivalence relation on


M by setting p q if and only if there is a smooth curve segment from p to q. This
relation is clearly reflexive and symmetric; if we can show that it is transitive, then the
result follows from Theorem 29. Suppose p q and q r, let : [1, 0] M be a
smooth curve segment from p to q, and let 0 : [0, 1] M be a smooth curve segment
from q to r. Let (U, ) be a smooth chart centered at q; by shrinking U , we may
assume that (U ) = Bn . Choose 0 < < 1 so that ((, 0]) U and 0 ([0, )) U .
Let
e = : (, 0] U and
e0 = 0 : [0, ) U . Let ` : (, ) Bn be
given by t 7 (t/, 0, . . . , 0). By Proposition 2.25, there is a smooth bump function
: (, ) R for [/4, /4] supported in (/2, /2). Define
e : (, ) Bn by
t 7 (t)`(t) + (1 (t))(e
(t) +
e0 (t)),
taking
e = 0 on (0, ) and
e0 = 0 on (, 0). It is easy to check that
e is smooth on
1
(, 0), (/4, /4) and (0, ), so
e is smooth. Let =
e; then , |[1,/2) , and
0 |(/2,1] agree on their overlaps, so we have a smooth curve segment defined on [1, 1]
from p to r.

Theorem 198. [Problem 11-13] The length of a smooth curve segment : [a, b] Rn
is defined to be the value of the (ordinary) integral
b
| 0 (t)| dt.
L() =
a

There is no smooth covector field X (Rn ) with the property that

= L()

for every smooth curve .


Proof. Let denote the curve t 7 (b t + a). Then L() = L(), but

= = L().


Example 199. [Problem 11-14] Consider the following two covector fields on R3 :
4z dx
2y dy
2x dz
= 2
+ 2
+ 2
,
2
(x + 1)
y +1 x +1
2y dy
2 dz
4xz dx
+ 2
= 2
+ 2
.
2
(x + 1)
y +1 x +1
(1) Set up and evaluate the line integral of each covector field along the straight
line segment from (0, 0, 0) to (1, 1, 1).
(2) Determine whether either of these covector fields is exact.

75

(3) For each one that is exact, find a potential function and use it to recompute the
line integral.
Let : [0, 1] R3 be given by t 7 (t, t, t). Then


1
4t
2t
2t
2
+
dt
=
+
(t + 1)2 t2 + 1 t2 + 1
0

1
4t3
=
dt
2 2
0 (1 + t )
= 2 log 2 1,
and

2t
2
4t2
+ 2
+ 2
2
2
(t + 1)
t +1 t +1


dt

2(t3 t2 + t + 1)
dt
(1 + t2 )2
0
= log 2 + 1.
=

It is easy to check that

1 6=
3 ,
z
x
so is not closed and not exact. It is similarly easy to check that is closed, so
Theorem 11.49 shows that is exact. Suppose f is a potential for ; it must satisfy
f
4xz
f
2y
f
2
= 2
,
=
,
=
.
x
(x + 1)2
y
y2 + 1
z
x2 + 1
Integrating the first equation, we have
f (x, y, z) =

2z
+ C1 (y, z).
x2 + 1

Then
C1
2y
= 2
,
y
y +1
so
C1 (y, z) = log(y 2 + 1) + C2 (z).
Finally we have
2
C2
2
+
= 2
,
+1
z
x +1
so C2 is constant. It is now easy to check that for any C R,
2z
f (x, y, z) = 2
+ log(y 2 + 1) + C
x +1
x2

76

is a potential for . Therefore

2
= df = + log(2) 0 = log 2 + 1.
2

Theorem 200. [Problem 11-15] Let X be a smooth vector field on an open subset
U Rn . Given a piecewise smooth
curve segment : [a, b] U , define the line
integral of X over , denoted by X ds, as

b
X ds =
X(t) 0 (t) dt,

where the dot on the right-hand side denotes the Euclidean dot product between tangent
vectors at (t), identified with elements of Rn . A conservative vector field is one
whose line integral around every piecewise smooth closed curve is zero.
(1) X is conservative if and only if there exists a smooth function f C (U ) such
that X = grad f .
(2) Suppose n = 3. If X is conservative then curl X = 0, where






X 1 X 3
X 2 X 1

X 3 X 2

.
curl X =
2
3
1
3
1
2
1
2
x
x
x
x
x
x
x
x
x3
(3) If U R3 is star-shaped, then X is conservative on U if and only if curl X = 0.
Proof. For any X X(U ), define a smooth covector field by x (v) = Xx v; then

0
X ds =
(t) ( (t)) dt = .

Part (1) follows immediately from Theorem 11.42, and part (2) follows from Proposition
11.44. Part (3) follows from Theorem 11.49.

Theorem 201. [Problem 11-16] Let M be a compact manifold of positive dimension.
Every exact covector field on M vanishes at least at two points in each component of
M.
Proof. Let = df be an exact covector field. Let U be a component of M . Since U
is compact, f attains a maximum at some x U and a minimum at some y U . If
x = y then f is constant on U , so df = 0 on U . Otherwise, df vanishes at the distinct
points x and y.

Theorem 202. [Problem 11-17] Let Tn = S1 S1 Cn denote the n-torus. For
each j = 1, . . . , n, let j : [0, 1] Tn be the curve segment
j (t) = (1, . . . , e2it , . . . , 1).

A closed covector field on Tn is exact if and only if j = 0 for j = 1, . . . , n.

77
1

Proof. Let n : Rn Tn be the smooth covering map n (x1 , . . . , xn ) = (e2ix , . . . , e2ix ).


n
Note that n is a local diffeomorphism, so Corollary 11.46 shows that
( ) is closed
n
and Theorem 11.49 shows that ( ) is exact. If is exact then j = 0 since each

j is a closed curve segment. Conversely, suppose that j = 0 for j = 1, . . . , n. Let


: [0, 1] Tn be a piecewise smooth closed curve segment and let
e : [0, 1] Rn be
n
a lift of such that =
e; Theorem 41 shows that
e is smooth. Write
e(0) =
1
n
1
n
(x , . . . , x ) and
e(1) = (x +m1 , . . . , x +mn ) for integers m1 , . . . , mn . Assume without
loss of generality that x1 = = xn = 0. For each i = 1, . . . , n, let i : [0, 1] Rn be
the straight line path from (m1 , . . . , mi1 , 0, . . . , 0) to (m1 , . . . , mi , . . . , 0). Let be the
concatenation of 1 , . . . , n so that (0) = 0 and (1) = (m1 , . . . , mn ). Then Theorem
186 shows that

n e

= (n )

= (n )

n
(n ) ,
=
( ) + +
1
n
=
+ +

n 1

n n

=0
since each n i is the concatenation of zero or more copies of i .

Theorem 203. [Problem 11-18] Suppose C and D are categories, and F, G are (covariant or contravariant) functors from C to D. A natural transformation from F to
G is a rule that assigns to each object X Ob(C) a morphism X HomD (F(X), G(X))
in such a way that for every pair of objects X, Y Ob(C) and every morphism f
HomC (X, Y ), the following diagram commutes:

F(X)

F(f )

X
G(X)

F(Y )
Y

G(f )

G(Y ).

78

(If either F or G is contravariant, the corresponding horizontal arrow should be reversed.)


(1) Let VecR denote the category of real vector spaces and linear maps, and let D be
the contravariant functor from VecR to itself that sends each vector space to its
dual space and each linear map to its dual map. The assignment V 7 V , where
V : V V is the map defined by V (v) = (v), is a natural transformation
from the identity functor of VecR to D D.
(2) There does not exist a natural transformation from the identity functor of VecR
to D.
(3) Let Diff 1 be the category of smooth manifolds and diffeomorphisms and VB the
category of smooth vector bundles and smooth bundle homomorphisms, and let
T, T : Diff 1 VB be the tangent and cotangent functors, respectively. There
does not exist a natural transformation from T to T .
(4) Let X : Diff 1 VecR be the covariant functor given by M 7 X(M ), F 7 F ;
and let X X : Diff 1 VecR be the covariant functor given by M 7 X(M )
X(M ), F 7 F F . The Lie bracket is a natural transformation from X X
to X.
Proof. Theorem 187 proves (1) and (2). If there is a natural transformation from T to
T , then taking the one-point space {} produces a natural transformation from the
identity functor of VecR to D, contradicting (2). This proves (3). Corollary 8.31 proves
(4).

Chapter 12. Tensors
Theorem 204. [Exercise 12.18] T k T M , T k T M , and T (k,l) T M have natural structures
as smooth vector bundles over M .
Proof. Let n = dim M . Let (U, ) be a smooth chart for M with coordinate functions
(xi ). Define : 1 (U ) U Rn(k+l) by
!








k
aij11,...,i

dxj1 p dxjl p
,...,jl


i
i
1
k
x p
x p
n,...,n
= (p, (a1,...,1
1,...,1 , . . . , an...,n )).

e , )
e :
Suppose (U
e is another smooth chart with coordinate functions (e
xi ), and let
e) U
e Rn be defined analogously. It follows from (11.6) and (11.7) that
1 (U



xi

e
xj
j
=
(p)
,
de
x
|
=
(p) dxi |p ,
p
e
xj
e
xj
xi
xi
p

79

where x
e=
e 1 and x =
e1 = x
e1 , and p denotes either a point in M or its
e ) we have
coordinate representation as appropriate. Therefore on 1 (U U
e 1 )(p, (e
(
a1,...,1
an,...,n
1,...,1 , . . . , e
n...,n ))
  1
x
x1
e
xj 1
e
xjl
k
= p,
(p)

(p)
(p)

(p)e
aij11,...,i
,...,jl ,
e
xi 1
e
xi k
x1
x1

xn
e
xj 1
xn
e
xj l
i1 ,...,ik
. . . , i1 (p) i (p) n (p) n (p)e
.
aj1 ,...,jl
e
x
e
xk
x
x
It follows from the vector bundle chart lemma that T M has a smooth structure making
it into a smooth vector bundle for which the maps are smooth local trivializations. 
Theorem 205. [Proposition 12.25] Suppose F : M N and G : N P are smooth
maps, A and B are covariant tensor fields on N , and f is a real-valued function on N .
(1)
(2)
(3)
(4)
(5)
(6)

F (f B) = (f F )F B.
F (A B) = F A F B.
F (A + B) = F A + F B.
F B is a (continuous) tensor field, and is smooth if B is smooth.
(G F ) B = F (G B).
(IdN ) B = B.

Proof. For (1),


(F (f B))p = dFp ((f B)F (p) ) = dFp (f (F (p))BF (p) ) = (f F )(p)(F B)p
since dFp is linear. For (2),
(F (A B))p (v1 , . . . , vk+l ) = (A B)p (dFp (v1 ), . . . , dFp (vk+l ))
= Ap (dFp (v1 ), . . . , dFp (vk ))Bp (dFp (vk+1 ), . . . , dFp (vk+l ))
= (F A)p (v1 , . . . , vk )(F B)p (vk+1 , . . . , vk+l ).
Part (3) follows from the fact that dFp is linear. For (4), let p M be arbitrary, and
choose smooth coordinates (y j ) for N in a neighborhood V of F (p). Let U = F 1 (V ),
which is a neighborhood of p. Writing B in coordinates as
B = Bj1 ,...,jk dy j1 dy jk
for continuous functions Bj1 ,...,jk on V , we have
F B = F (Bj1 ,...,jk dy j1 dy jk )
= (Bj1 ,...,jk F )F (dy j1 dy jk )
= (Bj1 ,...,jk F )F dy j1 F dy jk
= (Bj1 ,...,jk F )d(y j1 F ) d(y jk F ),

80

which is continuous, and is smooth if B is smooth. Part (5) follows from the fact that
d(G F )p ()(v1 , . . . , vk ) = (dGF (p) (dFp (v1 )), . . . , dGF (p) (dFp (vk )))
= dGF (p) ()(dFp (v1 ), . . . , dFp (vk ))
= dFp (dGF (p) ())(v1 , . . . , vk ).
Part (6) is obvious.

Example 206. [Problem 12-1] Give an example of finite-dimensional vector spaces V


and W and a specific element V W that cannot be expressed as v w for v V
and w W .
Take V = W = R2 , let e1 = (1, 0) and e2 = (0, 1), and let = e1 e1 + e2 e2 . Suppose
= v w for v, w R2 , and write v = v i ei and w = wj ej . Then = v i wj ei ej , so
v 1 w1 = 1,

v 1 w2 = 0,

v 2 w1 = 0,

v 2 w2 = 1.

This implies that w1 6= 0, so v 2 = 0, contradicting v 2 w2 = 1.


Theorem 207. [Problem 12-2] For any finite-dimensional real vector space V , there
are canonical isomorphisms R V
=V
= V R.
Proof. We can define a multilinear map f : R V V by (r, v) 7 rv, which induces a
linear map fe : R V V satisfying fe(r v) = rv. It is easy to check that the linear
map v 7 1 v is an inverse to fe, so fe is an isomorphism. A similar argument shows
that V

= V R.
Theorem 208. [Problem 12-3] Let V and W be finite-dimensional real vector spaces.
The tensor product space V W is uniquely determined up to canonical isomorphism
by its characteristic property. More precisely, suppose
e : V W Z is a bilinear map
into a vector space Z with the following property: for any bilinear map B : V W Y ,
e : Z Y such that the following diagram commutes:
there is a unique linear map B
V W

Y.

e
B

Z
Then there is a unique isomorphism : V W Z such that
e = , where
: V W V W is the canonical projection.
Proof. Since
e is bilinear, there exists a unique linear map : V W Z such that

e = . Similarly, there exists a unique linear map : Z V W such that

81

=
e. We have
e = =
e, so = IdZ by uniqueness. Similarly,
=
e = implies that = IdV W by uniqueness. Therefore is an
isomorphism.

Theorem 209. [Problem 12-4] Let V1 , . . . , Vk and W be finite-dimensional real vector
spaces. There is a canonical isomorphism
V V W
= L(V1 , . . . , Vk ; W ).
1

Proof. Proposition 12.10 shows that


V1 Vk
= L(V1 , . . . , Vk ; R),
so it remains to show that
L(V1 , . . . , Vk ; R) W
= L(V1 , . . . , Vk ; W ).
The bilinear map : L(V1 , . . . , Vk ; R) W L(V1 , . . . , Vk ; W ) defined by
(f, w)(v1 , . . . , vk ) = f (v1 , . . . , vk )w
induces a linear map
e : L(V1 , . . . , Vk ; R) W L(V1 , . . . , Vk ; W ) satisfying
(f
e w)(v1 , . . . , vk ) = f (v1 , . . . , vk )w.
It is easy to check that
e takes a basis of L(V1 , . . . , Vk ; R)W to a basis of L(V1 , . . . , Vk ; W ),
so
e is an isomorphism.

Theorem 210. [Problem 12-5] Let V be an n-dimensional real vector space. Then


n+k1
k

dim (V ) =
.
k
Proof. Let {1 , . . . , n } be a basis for V ; we want to show that


B = i1 ik : 1 i1 ik n
is a basis for k (V ), from which the result follows immediately. It is clear that B is
linearly independent. Let k (V ) and write
= ai1 ,...,ik i1 ik .
We have
= Sym = ai1 ,...,ik Sym i1 ik
= ai1 ,...,ik i1 ik ,
which is in the span of B after reordering the indices i1 , . . . , ik in each term i1 ik
to be in ascending order. This shows that B is a basis for k (V ).

Theorem 211. [Problem 12-6]

82

(1) Let be a covariant k-tensor on a finite-dimensional real vector space V . Then


Sym is the unique symmetric k-tensor satisfying
(Sym )(v, . . . , v) = (v, . . . , v)
for all v V .
(2) The symmetric product is associative: for all symmetric tensors , , ,
() = ().
1

(3) Let , . . . , be covectors on a finite-dimensional vector space. Their symmetric product satisfies
1 X (1)
1 k =

(k) .
k! S
k

Proof. Let {E1 , . . . , Ek } be a basis for V . For (1),


1 X
(v, . . . , v) = (v, . . . , v).
(Sym )(v, . . . , v) =
k! S
k

The uniqueness of Sym follows from the fact that




(Ei1 + + Ein )k : 1 n k, 1 i1 < < in k
spans V k . For (2), we have
(Sym(Sym ) )(v, . . . , v) = (Sym )(v, . . . , v)(v, . . . , v)
= (v, . . . , v)(v, . . . , v)(v, . . . , v)
= (v, . . . , v)(Sym )(v, . . . , v)
= (Sym (Sym ))(v, . . . , v),
so the result follows from (1). (Note that the tuples (v, . . . , v) may have different
numbers of elements.) Part (3) is obvious.

Example 212. [Problem 12-7] Let (e1 , e2 , e3 ) be the standard dual basis for (R3 ) . The
tensor e1 e2 e3 is not equal to a sum of an alternating tensor and a symmetric tensor.
Let (e1 , e2 , e3 ) be the standard basis for R3 . Suppose that
e1 e2 e3 = +
where is alternating and is symmetric. We have
1 = (e1 e2 e3 )(e1 , e2 , e3 ) = (e1 , e2 , e3 ) + (e1 , e2 , e3 )
but
0 = (e1 e2 e3 )(e2 , e3 , e1 ) = (e2 , e3 , e1 ) + (e2 , e3 , e1 )
= (e1 , e2 , e3 ) + (e1 , e2 , e3 ),

83

which is a contradiction.
Chapter 13. Riemannian Metrics
Theorem 213. [Exercise 13.21] Let (M, g) be a Riemannian n-manifold with or without boundary. For any immersed k-dimensional submanifold S M with or without
boundary, the normal bundle N S is a smooth rank-(n k) subbundle of T M |S . For
each p S, there is a smooth frame for N S on a neighborhood of p that is orthonormal
with respect to g.
Proof. Let p M be arbitrary, and let (X1 , . . . , Xk ) be a smooth local frame for T S
over some neighborhood V of p in M . Because immersed submanifolds are locally
embedded, by shrinking V if necessary, we may assume that it is a single slice in some
coordinate ball or half-ball U Rn . Since V is closed in U , Theorem 122(c) shows
e1 , . . . , X
en ) for T M |S over
that we can complete (X1 , . . . , Xk ) to a smooth local frame (X
U , and then Proposition 13.6 yields a smooth orthonormal frame (Ej ) over U such
that span(E1 |p , . . . , Ek |p ) = span(X1 |p , . . . , Xk |p ) = Tp S for each p U . It follows
that (Ek+1 , . . . , En ) restricts to a smooth orthonormal frame for N S over V . Thus N S
satisfies the local frame criterion in Lemma 10.32, and is therefore a smooth subbundle
of T M |S .

f, ge) are Riemannian manifolds
Theorem 214. [Exercise 13.24] Suppose (M, g) and (M
f is a local isometry. Then Lge(F ) = Lg ()
with or without boundary, and F : M M
for every piecewise smooth curve segment in M .
Proof. We have

Lge(F ) =

|(F )0 (t)|ge dt

=
a



dF(t) ( 0 (t)) dt
g
e

bq
=
ge(F )(t) (dF(t) ( 0 (t)), dF(t) ( 0 (t))) dt
a
bp
=
(F ge)( 0 (t), 0 (t)) dt
a
bp
=
g( 0 (t), 0 (t)) dt
a

= Lg ().


84

f, ge) are connected Riemannian


Theorem 215. [Exercise 13.27] Suppose (M, g) and (M
f is a Riemannian isometry. Then
manifolds and F : M M
dge(F (p), F (q)) = dg (p, q)
for all p, q M .
Proof. If is a piecewise smooth curve segment from p to q, then F is a piecewise
smooth curve segment from F (p) to F (q), and Theorem 214 shows that Lg () = Lge(F
). Similarly, if
e is a piecewise smooth curve segment from F (p) to F (q) then F 1
e
is a piecewise smooth curve segment from p to q and Lge(e
) = Lg (F 1
e).

Theorem 216. [Problem 13-1] If (M, g) is a Riemannian
n-manifold with
n
o or without
boundary, let U M T M be the subset U M = (x, v) T M : |v|g = 1 , called the
unit tangent bundle of M . Then U M is a smooth fiber bundle over M with model
fiber Sn1 .
Proof. Define ` : T M R by (x, v) 7 g(x)(v, v) and for fixed x M , define
`x : Tx M R by v 7 g(x)(v, v). In any smooth local coordinates around x we
have D`(v)(u) = 2g(x)(v, u) and in particular D`(v)(v) = 2g(x)(v, v) > 0 for all v 6= 0.
Therefore Corollary 5.14 shows that U M = `1 ({1}) is a properly embedded submanifold of T M . Let : T M M be the canonical projection. Let : 1 (U ) U Rn
be a smooth local trivialization. Define
: 1 (U ) U M U Sn1


(Rn )(x, v)
(x, v) 7 x,
k(Rn )(x, v)k
and
: U Sn1 1 (U ) U M
(x, v) 7

1 (x, v)
.
|1 (x, v)|g

Then and are smooth and inverses of each other, so is a local trivialization of
U M over M .

Theorem 217. [Problem 13-2] Suppose that E is a smooth vector bundle over a smooth
manifold M with or without boundary, and V E is an open subset with the property
that for each p M , the intersection of V with the fiber Ep is convex and nonempty.
By a section of V , we mean a (local or global) section of E whose image lies in V .
(1) There exists a smooth global section of V .

85

(2) Suppose : A V is a smooth section of V defined on a closed subset A M .


(This means that extends to a smooth section of V in a neighborhood of each
point of A.) There exists a smooth global section
e of V whose restriction to A
is equal to . If V contains the image of the zero section of E, then
e can be
chosen to be supported in any predetermined neighborhood of A.
Proof. For (1), let p M and choose a local trivialization : 1 (U ) U Rn such
that p U . Choose any v 1 ({p}) and choose neighborhoods Up of p and Fp of v 0 =
(Rn )(v) such that Up Fp ( 1 (U ) V ). Define p : Up V by x 7 1 (x, v 0 ).
The family of sets {Up : p M } is an open cover of M . Let {p : p M } be a smooth
partition of unity subordinate to this cover, with supp p Up . For each p M , the
product p p is smooth on Wp by Theorem 159, and has a smooth extension to all of M
if we interpret it to be zero on M \ supp p . (The extended function is smooth because
the two definitions agree on the open subset Wp \ supp p where they overlap.) Thus
we can define : M V by
X
(x) =
p (x)p (x).
pM

Because the collection of supports {supp p } is locally finite, this sum actually has only
a finite number of nonzero terms in a neighborhood
of any point of M , and therefore
P
defines a smooth function. The fact that pM p = 1 implies that (x) is a convex
combination of vectors in Ex for every x M , so the image of does indeed lie in V .
For (2), let : M V be a smooth global section of V . If V contains the imaage of
the zero section of E, then let U be some neighborhood of A and replace with the
zero section of E. For each p A, choose a neighborhood Wp of p and a local section

ep : Wp V of V that agrees with on Wp A. If U was chosen, then replacing Wp by


Wp U we may assume that Wp U . The family of sets {Wp : p A} {M \ A} is an
open cover of M . Let {p : p A} {0 } be a smooth partition of unity subordinate
to this cover, with supp p Wp and supp 0 M \ A.
For each p A, the product p
ep is smooth on Wp and has a smooth extension to all
of M if we interpret it to be zero on M \ supp p . Thus we can define
e : M V by
X

e(x) = 0 (x) (x) +


p (x)e
p (x).
pA

If x A, then 0 (x) = 0 and


ep (x) = (x) for each p such that p (x) 6= 0, so
!
X
X

e(x) =
p (x)(x) = 0 (x) +
p (x) (x) = (x),
pA

pA

P
so
e is indeed an extension of . The fact that 0 + pA p = 1 implies that
e(x) is
a convex combination of vectors in Ex for every x M , so the image of
e does indeed

86

lie in V . If U was chosen, then it follows from Lemma 1.13(b) that


[
[
supp p =
supp p U.
supp
e=
pA

pA


Theorem 218. Suppose (M, g) is a Riemannian manifold, and X is a vector field on
M . Given any positive continuous function : M R, there exists a smooth vector
e X(M ) that is -close to X. If X is smooth on a closed subset A M ,
field X
e
e
then X can
be chosen to be equal to X on A. (Two vector fields X, X are -close if


ex < (x) for all x M .
X x X
g

Proof. If X is smooth on the closed subset A, then by Lemma 8.6, there is a smooth
vector field X0 X(M ) that agrees with X on A. Let
U0 = {y M : |X0 |y Xy |g < (y)}.
Then U0 is an open subset containing A. (If there is no such set A, we just take
U0 = A = and F0 = 0.)
We will show that there are countably many points {xi }
i=1 in M \ A and smooth

coordinate balls Ui of xi in M \ A such that {Ui }i=1 is an open cover of M \ A and


(*)

|Xy Xi |y |g < (y)

for all y Ui , where Xi X(Ui ) is a constant coefficient vector field such that Xi |xi =
Xxi . To see this, for any x M \ A, let Ux be a smooth coordinate ball around x
contained in M \ A and small enough that


1
1
(y) > (x) and Xy X (x) |y g < (x)
2
2
for all y Ux , where X (x) X(Ux ) is a constant coefficient vector field such that
X (x) |x = Xx . (Such a neighborhood exists by continuity of and X.) Then if y Ux ,
we have


Xy X (x) |y < 1 (x) < (y).
g
2
The collection {Ux : x M \ A} is an open cover of M \ A. Choosing a countable
(xi )
subcover {Uxi }
, we have (*).
i=1 and setting Ui = Uxi and Xi = X
Let {0 , i } be a smooth partition of unity subordinate to the cover {U0 , Ui } of M , and
e X(M ) by
define X
X
ey = 0 (y)X0 |y +
X
i (y)Xi |y .
i1

87

e is smooth, and is equal to X on A. For any y M , the fact that


Then
clearly X
P
i0 i = 1 implies that

!




X
X
e



i (y)Xi |y 0 (y) +
i (y) Xy
Xy Xy = 0 (y)X0 |y +


g
i1
i1
g
X
0 (y) |X0 |y Xy |g +
i (y) |Xi |y Xy |g
i1

< 0 (y)(y) +

i (y)(y) = (y).

i1


Lemma 219. Let V be a real inner product space and let kk be the norm on V . Let
{u1 , . . . , uk } be an orthonormal set of vectors in V with kui k = 1 for i = 1, . . . , k. Let
v V with kvk = 1.
(1) We have
hu1 , vi2 + + huk , vi2 1,
and equality holds if and only if the set {v, u1 , . . . , uk } is linearly dependent.
(2) If w V and
kv wk2 < 1 hu1 , vi2 huk , vi2 ,
then {w, u1 , . . . , uk } are linearly independent.
Proof. For (1), complete {u1 , . . . , uk } to an orthonormal basis {u1 , . . . , un } of V . Write
v = a1 u1 + + an un where a1 , . . . , an R. Then
hu1 , vi2 + + huk , vi2 = hu1 , a1 u1 i2 + + huk , ak uk i2
= a21 + + a2k
= 1 a2k+1 a2n
since kvk = 1. Clearly the last expression is equal to 1 if and only if ak+1 , . . . , an = 0,
which is true if and only if v is in the span of {u1 , . . . , uk }. For (2), suppose w =
a1 u1 + + ak uk where a1 , . . . , ak R. Then

2
k
k


X
X


0 > v
ai u i 1 +
hui , vi2


i=1
i=1
2
* k
+ k
k
X

X
X


= 1 2 v,
ai ui +
ai u i 1 +
hui , vi2


i=1

i=1

i=1

88

k
X

(a2i 2ai hui , vi + hui , vi2 )

i=1

k
X

(ai hui , vi)2 ,

i=1

which is a contradiction.

Theorem 220. [Problem 13-3] Let M be a smooth manifold.


(1) If there exists a global nonvanishing vector field on M , then there exists a global
smooth nonvanishing vector field.
(2) If there exists a linearly independent k-tuple of vector fields on M , then there
exists such a k-tuple of smooth vector fields.
Proof. Let g be a Riemannian metric on M . For (1), let X be a global nonvanishing
e X(M ) such
vector field on M . By Theorem 218, there is a smooth vector field X
that
ex |g < |Xx |g
|Xx X
ex |g > 0 for all x M , so X
e is nonvanishing. For
for all x M . This implies that |X
(2), let (X1 , . . . , Xk ) be a linearly independent k-tuple of vector fields on M . Using the
Gram-Schmidt process, we may assume that (X1 , . . . , Xk ) is an orthonormal k-tuple of
vector fields. Assume that X1 , . . . , Xj1 are already smooth. Define
s
X
(x) = 1
hXj |x , Xi |x i2g ;
i6=j

then : M R is a positive continuous function by Lemma 219. By Theorem 218,


ej X(M ) such that
there is a smooth vector field X
X
ej |x )|2 < 1
|(Xj |x X
hXj |x , Xi |x i2g
g
i6=j

ej , . . . , Xk ) is linearly indefor all x M . Then Lemma 219 shows that (X1 , . . . , X


ej ) to obtain an orthonormal
pendent. Apply the Gram-Schmidt process to (X1 , . . . , X
k-tuple of vector fields such that the first j vector fields are smooth. Repeating this
procedure k times produces an orthonormal k-tuple of smooth vector fields.


Example 221. [Problem 13-4] Let g denote the round metric on Sn . Compute the

coordinate representation of g in stereographic coordinates.


We have
(x1 , . . . , xn+1 ) =

(2u1 , . . . , 2un , |u|2 1)


,
|u|2 + 1

89

so
dxj =

2(|u|2 2uj + 1) j X 4ui uj


du
dui
2
2
(|u|2 + 1)2
(|u|
+
1)
i6=j

for each j = 1, . . . , n and


dxn+1 =

n
X
i=1

4ui
dui .
2
2
(|u| + 1)

Therefore

g = (dx1 )2 + + (dxn+1 )2
4
=
((du1 )2 + + (dun )2 ).
2
2
(|u| + 1)
Theorem 222. [Problem 13-5] Suppose (M, g) is a Riemannian manifold. A smooth
curve : J M is said to be a unit-speed curve if | 0 (t)|g = 1. Every smooth curve
with nowhere-vanishing velocity has a unit-speed reparametrization.
Proof. We can assume that J is open. Choose any t0 J and define ` : J R by
t
`(t) =
| 0 (x)|g dx.
t0

Then x 7 | 0 (x)|g is smooth since v 7 |v|g is smooth on {(x, v) T M : v 6= 0} and


0 (x) 6= 0 for all x J. Therefore ` is smooth, and since `0 (t) 6= 0 for all t J, the
inverse function theorem shows that `1 : `(J) J is smooth. Define 1 : `(J) M
by 1 = `1 . We have
10 (x) = (`1 )0 (x) 0 (`1 (x))
0 (`1 (x))
`0 (`1 (x))
0 (`1 (x))
= 0 1
,
| (` (x))|g

so 1 is a unit-speed reparametrization of .

Theorem 223. [Problem 13-6] Every Riemannian 1-manifold is flat.


Proof. Let (M, g) be a Riemannian 1-manifold. Suppose x M and (U, ) is a smooth
chart containing x. By shrinking U , we can assume that U is connected. Therefore 1 :
(U ) U is a smooth curve, and Theorem 222 shows that there is a diffeomorphism
: E (U ) such that 1 is a unit-speed curve. We want to show that 1 :

90

E U is an isometry. If v, w Tp E are nonzero vectors then v = r d/dt|p and


w = s d/dt|p for some r, s R, so

!


d
d
,
((1 ) gp )(v, w) = rs((1 ) gp )

dt p dt p

! 2


d


= rs d(1 )p

dt p
g
1
2
0
= rs ( ) (p)
g

= rs
= g (1 )(p) (v, w).

Theorem 224. [Problem 13-7] A product of flat metrics is flat.
Proof. If (R1 , g1 ), . . . , (Rk , gk ) are Riemannian manifolds, then
(R1 Rk , g1 gk )
is also a Riemannian manifold. Let (x1 , . . . , xk ) R1 Rk . For each i = 1, . . . , k,
let Fi : Ui Vi be an isometry such that xi Ui . Then F1 Fk is an isometry
from the neighborhood U1 Uk to the open set V1 Vk , since
(F1 Fk ) (g g) = F1 g Fk g
= g1 gk .

Corollary 225. [Problem 13-8] Let Tn = S1 S1 Cn , and let g be the metric
on Tn induced from the Euclidean metric on Cn (identified with R2n ). Then g is flat.
Theorem 226. [Problem 13-10] The shortest path between two points in Euclidean
space is a straight line segment. More precisely, for x, y Rn , let : [0, 1] Rn be the
curve segment (t) = x + t(y x). Any other piecewise smooth curve segment
e from
x to y satisfies Lg (e
) > Lg () unless
e is a reparametrization of .
Proof. First note that if
e : [a, b] Rn then
b

b


0
0
|e
(t)| dt

e (t)dt = |e
(b)
e(a)| .
a

By rotating Rn , we can assume that x, y X, where X = R {0}n1 is the x1 axis. Let


e : [a, b] Rn be a piecewise smooth curve segment from x to y such that

91

Lg (e
) = |y x|. Suppose that
e(c)
/ X for some c (a, b). Then
b
c
0
|e
0 (t)| dt
|e
(t)| dt +
Lg (e
) =
a

|e
(c)
e(a)| + |e
(b)
e(c)|
> |y x| = Lg (),
which is a contradiction. Therefore
e([a, b]) X. Let : Rn X be the canonical
projection. A similar argument shows that
e is a diffeomorphism from [a, b] to the
line segment from (x) to (y).

Example 227. [Problem 13-11] Let M = R2 \ {0}, and let g be the restriction to M
of the Euclidean metric g. Show that there are points p, q M for which there is no
piecewise smooth curve segment from p to q in M with Lg () = dg (p, q).
Let p = (1, 0) and q = (1, 0). For each 0 < r < 1, let
p0 = (r, 0), q 0 = (r, 0),
p00 = (r, r), q 00 = (r, r)
and define r : [0, 5] M by

(1 t)p + tp0 ,

0
00

(2 t)p + (t 1)p ,
r (t) = (3 t)p00 + (t 2)q 00 ,

(4 t)q 00 + (t 3)q 0 ,

(5 t)q 0 + (t 4)q,

0 t < 1,
1 t < 2,
2 t < 3,
3 t < 4,
4 t 5.

We have
Lg (r ) = 1 r + 4r + 1 r = 2 + 2r
2
as r 0. Therefore dg (p, q) 2. But Theorem 226 shows that dg (p, q) 2, so
dg (p, q) = 2. There is no piecewise smooth curve segment from p to q such that
Lg () = 2, due to Theorem 226.
Theorem 228. [Problem 13-12] Consider Rn as a Riemannian manifold with the Euclidean metric g.
(1) Suppose U, V Rn are connected open sets, , : U V are Riemannian
isometries, and for some p U they satisfy (p) = (p) and dp = dp . Then
= .
(2) The set of maps from Rn to itself given by the action of E(n) on Rn described
in Example 7.32 is the full group of Riemannian isometries of (Rn , g).

92

Proof. By considering 1 : V V , it suffices to show that for any connected


open set U Rn and any Riemannian isometry f : U U satisfying f (p) = p and
dfp = IdTp U for some p U , we have f = IdU . If : I U is a smooth curve
then Lg () = Lg (f ), so Theorem 226 implies that f takes straight line segments to
straight line segments (of the same length). Let
E = {x U : f (x) = x, dfx = IdTx U } ;
then E is nonempty. Let x E and let B = Br (x) be an open ball of radius r around
x such that B U . For any v Sn1 , let : [1, 1] U be the curve t 7 x + trv
and let L = ([1, 1]). Then f is a straight line segment of length 2r such that
(f )(0) = x. But d(f )0 = dfx d0 = d0 , so (f )([1, 1]) = L. Using the fact
that
dg (x, y) = dg (f (x), f (y)) = dg (x, f (y))
for all y U , we conclude that f = . But v was arbitrary, so f |B is the identity map
on B(x). This shows that B E, and therefore E is open. It is clear from continuity
that E is closed. Since U is connected, we must have E = U . This proves (1).
Let : Rn Rn be a Riemannian isometry. Then (x)
e
= D(0)1 ((x) (0))
defines a Riemannian isometry satisfying D(0)
e
= I and (0)
e
= 0, so (1) shows that

e = IdRn . Therefore (x) = (0) + D(0)x is an affine transformation.



Theorem 229. [Problem 13-15] Let (M, g) be a Riemannian manifold, and let gb be
the product metric on M R determined by g and the Euclidean metric on R. Let
X = 0 d/dt be the product vector field on M R determined by the zero vector field
on M and the standard coordinate vector field d/dt on R. Then X is a Killing vector
field for (M R, gb).
Proof. The flow of X is given by ((x, r), t) = (x, r + t).

Theorem 230. [Problem 13-16] If g = f (t)dt2 is a Riemannian metric on R, then g


is complete if and only if both of the following improper integrals diverge:
p
0 p
(*)
f (t) dt,
f (t) dt.

Proof. If x, y R and : [x, y] R is the identity map then


y
y
yp
0
dg (x, y) =
| (t)|g dt =
|1|g dt =
f (t) dt.
x

p
Note that f (t) > 0 for all t R. If 0
f (t) dt converges, then {xn } = {1, 2, . . . }
is a Cauchy sequence. Suppose
that
g
is
complete;
then n x for some x R.
np
If x 0 then dg (n, x) = x f (t) dt converges to a positive number as n ,
which is a contradiction. A similar contradiction arises if x < 0. Therefore g is not

93

complete. Conversely, suppose that both integrals in (*) diverge and let {xn } be a
Cauchy sequence. Then {xn } is contained in a set K R that is bounded with respect
to the metric induced by g. Since the integrals in (*) diverge, K must also be bounded
with respect to the Euclidean metric. By replacing K with a closed and bounded
interval containing K, we may assume that K is compact. There are positive constants
c, C such that
p
p
p
c f (t) C and
f (t)/C 1 f (t)/c
for all t K. Let > 0. Since {xn } is Cauchy, there exists an integer N such that
xn p
c <
f (t) dt < c
xm

for all m, n N . Then


c
1
< <
C
C

xn

xm

p
1 xn p
c
f (t) dt |xn xm |
f (t) dt <
= ,
c xm
c

so in the Euclidean metric, {xn } is Cauchy and xn x for some x K. We have



x
p


0

f
(t)
dt


xn

as n , so xn x in the metric induced by g.

Theorem 231. [Problem 13-18] Suppose (M, g) is a connected Riemannian manifold,


S M is a connected embedded submanifold, and ge is the induced Riemannian metric
on S.
(1) dge(p, q) dg (p, q) for p, q S.
(2) If (M, g) is complete and S is properly embedded, then (S, ge) is complete.
(3) Every connected smooth manifold admits a complete Riemannian metric.
Proof. Let : S , M be the inclusion map. If : [a, b] S is a piecewise smooth
curve segment from p to q in S then is a piecewise smooth curve segment from
p to q in M and Lge() = Lg ( ) dg (p, q). Since this holds for all , we have
dge(p, q) dg (p, q). This proves (1). Let {xn } be a Cauchy sequence in S and let > 0.
There exists an integer N such that
dg (xm , xn ) dge(xm , xn ) <
for all m, n N , so {xn } is a Cauchy sequence in M and xn x for some x M . But
S is closed in M , so x S. This proves (2). Let N be a connected smooth manifold.
Theorem 6.15 shows that there is a proper smooth embedding j : N , R2n+1 . Since
(R2n+1 , g) is complete, (N, j g) is complete by (2). This proves (3).


94

Example 232. [Problem 13-19] The following example shows that the converse of
Theorem 231(b) does not hold. Define F : R R2 by F (t) = ((et +1) cos t, (et +1) sin t).
Show that F is an embedding that is not proper, yet R is complete in the metric induced
from the Euclidean metric on R2 .
We have



et (cos t sin t) sin t
DF (t) = t
,
e (cos t + sin t) + cos t

which is never zero. Since F is an open map, it is a smooth embedding. But F 1 B 2 (0)
is not compact, where B 2 (0) is the closed ball of radius 2 around (0, 0). The metric on
R induced from the Euclidean metric on R2 is given by
f g = d((et + 1) cos t)2 + d((et + 1) sin t)2
= (2e2t + 2et + 1)dt2 .
But

2e2t

2et

+ 1 dt and

2e2t + 2et + 1 dt

diverge, so Theorem 230 shows that f g is complete.


Theorem 233. [Problem 13-21] Let (M, g) be a Riemannian manifold, let f C (M ),
and let p M be a regular point of f .
(1) Among all unit vectors v Tp M , the directional derivative vf is greatest when
v points in the same direction as grad f |p , and the length of grad f |p is equal to
the value of the directional derivative in that direction.
(2) grad f |p is normal to the level set of f through p.
Proof. We have hgrad f |p , vi = vf , so the Cauchy-Schwarz inequality shows that
| hgrad f |p , vig |
is maximized when v = grad f |p . If S is the level set of f through p then Tp S = ker dfp
by Proposition 5.38. Clearly hgrad f |p , vi = vf = dfp (v) = 0 for every v ker dfp .

Theorem 234. [Problem 13-22] For any smooth manifold M with or without boundary,
the vector bundles T M and T M are smoothly isomorphic over M .
Proof. Choose any Riemannian metric g on M and let gb : T M T M be the smooth
bundle homomorphism defined at each p M by gb(v)(w) = gp (v, w). Then gb is injective
(and therefore bijective) at each point, so Theorem 174 shows that gb is a smooth bundle
isomorphism.


95

Example 235. [Problem 13-23] Is there a smooth covector field on S2 that vanishes at
exactly one point?
Yes, due to Theorem 128 and Theorem 234.

Chapter 14. Differential Forms


Theorem 236. [Exercise 14.4] Let be a covariant tensor on a finite-dimensional
vector space.
(1) Alt is alternating.
(2) Alt = if and only if is alternating.
Proof. If Sk then
1 X
(sgn )(v( )(1) , . . . , v( )(k) )
k! S
k
1 X
sgn( )(v( )(1) , . . . , v( )(k) )
= sgn( )
k! S
k
X
1
(sgn )(v(1) , . . . , v(k) )
= sgn( )
k! S

(Alt )(v (1) , . . . , v (k) ) =

= sgn( )(Alt )(v1 , . . . , vk ),


which proves (1). If is alternating then
1 X
(sgn )(v(1) , . . . , v(k) )
k! S
k
1 X
=
(sgn )2 (v1 , . . . , vk )
k! S

(Alt )(v1 , . . . , vk ) =

= (v1 , . . . , vk ).
Conversely, if Alt = then is alternating since Alt is alternating. This proves
(2).

Theorem 237. [Exercise 14.12] The wedge product is the unique associative, bilinear,
and anticommutative map k (V ) l (V ) k+l (V ) satisfying
i 1 i k = I
for any basis (i ) for V and any multi-index I = (i1 , . . . , ik ).

96

e be such a map. Let k (V ) and l (V ). By Lemma 14.10,


Proof. Let
!
!
0
0
X
X
e =
e

I I
J J
=
=
=
=

I
0
0
XX
I
J
0
0
XX
I
J
0
0
XX
I
J
0
0
XX

e J
I J I
e ej1
e
e ejl
e
e e ik
I J i1
I J IJ
I J I J

J
0
X

I I

0
X

!
J J

= .

Theorem 238. [Exercise 14.14] k T Mis a smooth subbundle of T k T M , and therefore is a smooth vector bundle of rank nk over M .
Proof. This follows from Proposition 14.8 and Lemma 10.32.

Theorem 239. [Exercise 14.28] The diagram below commutes:

C (R3 )

grad

Id
0 (R3 )

X(R3 )

curl

1 (R3 )

div

2 (R3 )

C (R3 )

[
d

X(R3 )

3 (R3 ).

Therefore curl grad = 0 and div curl = 0 on R3 . The analogues of the left-hand and
right-hand square commute when R3 is replaced by Rn for any n.

97

Proof. In Rn , we have
n
X
f
([ grad)(f ) = [
xi xi
i=1

n
X
f i
=
dx = df
i
x
i=1

and
(d )(X) = d(Xy(dx1 dxn ))
n
X
ci dxn
=d
(1)i1 dxi (X)dx1 dx
i=1
n
X

ci dxn
(1)i1 X i dx1 dx

=d

i=1

n
X

i1

(1)

i=1
n
X

n
X
X i
j=1

xj

ci dxn
dxj dx1 dx

X i 1
dx dxn
i
x
i=1
!
n
X
X i
=
xi
i=1
=

= ( div)(X).
In R3 we have



X 1 X 3
X 3 X 2

( curl)(X) =
2
3
1
3
1
x
x
x
x
x
x2



X 2 X 1

x1
x2 x3




X 2 X 1
X 3 X 1
1
2
=

dx dx +

dx1 dx3
x1
x2
x1
x3


X 3 X 2
+

dx2 dx3
x2
x3

= d X 1 dx1 + X 2 dx2 + X 3 dx3


= (d [)(X).

Theorem 240. [Problem 14-1] Covectors 1 , . . . , k V are linearly dependent if and
only if 1 k = 0.

98

Proof. Let n = dim V . Suppose that 1 , . . . , k are linearly dependent and assume
without loss of generality that a1 1 + + ak k = 0 with a1 6= 0. Then 1 =
2
k
a1
1 (a2 + + ak ), so
2
k
2
k
1 k = a1
1 (a2 + + ak ) = 0.



Conversely, suppose that 1 , . . . , k are linearly independent. We can extend 1 , . . . , k
to a basis { 1 , . . . , n } of n (V ). Then
1 n = ( 1 k ) ( k+1 n ) 6= 0,
so 1 k 6= 0.

Theorem 241. [Problem 14-5] Let M be a smooth n-manifold with or without boundary, and let ( 1 , . . . , k ) be an ordered k-tuple of smooth 1-forms on an open subset
U M such that ( 1 |p , . . . , k |p ) is linearly independent for each p U . Given smooth
1-forms 1 , . . . , k on U such that
k
X

i i = 0,

i=1
i

each can be written as a linear combination of 1 , . . . , k with smooth coefficients.


Proof. First note that each i is in the span of 1 , . . . , k , since
i 1 k = (1)i+1 1 i i k
X
j j k
= (1)i 1
j6=i

= 0.
Therefore we have i = rij j for some coefficients rij : U R. But the span of
1 , . . . , k forms a smooth subbundle of T M , so Proposition 10.22 shows that the
functions rij are smooth.

Example 242. [Problem 14-6] Define a 2-form on R3 by
= x dy dz + y dz dx + z dx dy.
(1) Compute in spherical coordinates (, , ) defined by
(x, y, z) = ( sin cos , sin sin , cos ).
(2) Compute d in both Cartesian and spherical coordinates and verify that both
expressions represent the same 3-form.
(3) Compute the pullback S2 to S2 , using coordinates (, ) on the open subset
where these coordinates are defined.
(4) Show that S2 is nowhere zero.

99

We have
dx = sin cos d + cos cos d sin sin d,
dy = sin sin d + cos sin d + sin cos d,
dz = cos d sin d,
so
= 3 sin d d.
Also,
d = 3dx dy dz
and
d = 32 sin d d d.
The pullback of to S2 is given by
S2 = sin d d,
which is defined for (, ) (0, )(0, 2). This expression is never zero since sin > 0
for (0, ).

Chapter 15. Orientations


Theorem 243. [Exercise 15.4] Suppose M is an oriented smooth n-manifold with or
without boundary, and n 1. Every local frame with connected domain is either positively oriented or negatively oriented.
Proof. Let (Ei ) be a local frame with a connected domain U . Let D be the points of
ei ) be an oriented local
U at which (Ei ) is positively oriented. Let p D and let (E
ej for continuous
frame defined on a neighborhood V U of p. Writing Ei = Bij E
component functions Bij : V R, we have det(Bij (p)) > 0. Since det is continuous,
there is a neighborhood of p on which Ei is oriented. This shows that D is open. A
similar argument shows that U \ D is open, so D is either empty or equal to U by the
connectedness of U .

Theorem 244. [Exercise 15.8] Suppose M1 , . . . , Mk are orientable smooth manifolds.
There is a unique orientation on M1 Mk , called the product orientation,
with the following property: if for each i = 1, . . . , k, i is an orientation form for the
given orientation on Mi , then 1 1 k k is an orientation form for the product
orientation.

100

Proof. Let 1 , . . . , k be orientation forms for M1 , . . . , Mk and let O be the orientation


determined by 1 1 k k . Suppose 1 , . . . , k are orientation forms for M1 , . . . , Mk .
Then i = fi i for some strictly positive continuous functions fi : Mi R, so
1 1 k k = (f1 1 ) (fk k )1 k .
Since (f1 1 ) (fk k ) is strictly positive, 1 1 k k determines the same
orientation as 1 k .

Theorem 245. [Exercise 15.10] Let M be a connected, orientable, smooth manifold
with or without boundary. Then M has exactly two orientations. If two orientations of
M agree at one point, they are equal.
Proof. If dim M = 0 then M consists of a single point, so the result is clear. Suppose
O, O0 are two orientations of M that agree at a single point p. Let D = {x M : Ox = Ox0 };
this set is nonempty since p D. Let x D and let (Ei ) be a local frame defined in
a connected neighborhood of x that is positively oriented with respect to O0 . Applying Theorem 243 to (M, O) shows that (Ei ) is positively oriented with respect to O.
Therefore D is open. A similar argument shows that M \ D is open, so D = M by the
connectedness of M .
Since M is orientable, there is an orientation O for M . Let be an orientation form
for O; then determines an orientation O0 for M with Op 6= Op0 for all p M . If
O00 is any orientation and p is any point in M then either Op00 = Op or Op00 = Op0 , so
O00 = O or O00 = O0 .

Theorem 246. [Exercise 15.12] Suppose M is an oriented smooth manifold with or
without boundary, and D M is a smooth codimension-0 submanifold with or without
boundary. The orientation of M restricts to an orientation of D. If is an orientation
form for M , then D is an orientation form for D.
Proof. It suffices to check that D is nonvanishing. But this is clear since d(D )p is an
isomorphism for every p M .

Theorem 247. [Exercise 15.13] Suppose M and N are oriented positive-dimensional
smooth manifolds with or without boundary, and F : M N is a local diffeomorphism.
The following are equivalent:
(1) F is orientation-preserving.
(2) With respect to any oriented smooth charts for M and N , the Jacobian matrix
of F has positive determinant.
(3) For any positively oriented orientation form for N , the form F is positively
oriented for M .

101

Proof. If (U, (xi )) and (V, (y i )) are oriented smooth charts for M and N respectively,
then dFp takes the oriented basis (/xi |p ) to an oriented basis
!
F j

(p) j .
xi
y p
Since (/y i |p ) is also an oriented basis, we have det(F j /xi (p)) > 0. This proves
(1) (2). Suppose that (2) holds. By shrinking U and V , we can assume that they
are connected sets and that F |U : U V is a diffeomorphism. Then
= f dy 1 dy n
on V for some positive continuous function f , so by Proposition 14.20 we have
F = (f F )(det DF )dx1 dxn .
Since det DF > 0 by (2), F is positively oriented on U . This proves (2) (3).
Finally, suppose that (3) holds and let (Ei ) be an oriented basis of Tp M . If is a
positively oriented orientation form for N then
(F )p (E1 , . . . , En ) > 0 F (p) (dFp (E1 ), . . . , dFp (En )) > 0,
so (dFp (Ei )) is an oriented basis of TF (p) N . This proves (3) (1).

Theorem 248. [Exercise 15.14] A composition of orientation-preserving maps is orientationpreserving.


Proof. Let F : M N and G : N P be orientation-preserving maps. If (Ei ) is an
oriented basis of Tp M then (dFp (Ei )) is an oriented basis of TF (p) N and (d(G F )p (Ei ))
is an oriented basis of T(GF )(p) P .

Theorem 249. [Exercise 15.16] Suppose F : M N and G : N P are local
diffeomorphisms and O is an orientation on P . Then (G F ) O = F (G O).
Proof. This is clear from the fact that (G F ) = F G for any orientation form
.

Theorem 250. [Exercise 15.20] Every Lie group has precisely two left-invariant orientations, corresponding to the two orientations of its Lie algebra.
Proof. Let G be a Lie group. Corollary 8.39 shows that there are at least two leftinvariant orientations. If O is a left-invariant orientation on G then it is completely
determined by Oe , so there are exactly two left-invariant orientations.

f, ge) are positive-dimensional
Theorem 251. [Exercise 15.30] Suppose (M, g) and (M
f is a local isometry.
Riemannian manifolds with or without boundary, and F : M M

Then F ge = g .

102

Proof. Let p M and let U be a neighborhood of p for which F |U is an isometry.


If (Ei ) is a local oriented orthonormal frame for M then (dF (Ei )) is a local oriented
f on a neighborhood of F (p) since
orthonormal frame for M
hdF (Ei ), dF (Ej )ige = hEi , Ej ig = ij .
Therefore
(F ge)(E1 , . . . , En ) = ge(dF (E1 ), . . . , dF (En )) = 1,
which shows that F ge is the Riemannian volume form on M .

Theorem 252. [Problem 15-1] Suppose M is a smooth manifold that is the union of
two orientable open submanifolds with connected intersection. Then M is orientable.
Proof. Denote the two open submanifolds by N and P . We can assume that there is
some p N P , for otherwise the result is trivial. Choose an orientation O for N , and
choose an orientation O0 for P such that Op = Op0 . By Theorem 245, O and O0 agree
on N P , and it is easy to check that the orientation given by
(
Ox , x N,
Ox00 =
Ox0 , x P
is well-defined and continuous.

Theorem 253. [Problem 15-2] Suppose M and N are oriented smooth manifolds with
or without boundary, and F : M N is a local diffeomorphism. If M is connected,
then F is either orientation-preserving or orientation-reversing.
Proof. If M and N are 0-manifolds, then M and N both consist of a single point and
the result is trivial. Let D be the set of points x for which dFx takes oriented bases of
Tx M to oriented bases of TF (x) N . Let be a positively oriented orientation form for
N . Let x D and let (Ei ) be an oriented local frame defined on a neighborhood U of
x. Then (F )x (E1 |x , . . . , En |x ) > 0, so by continuity there is a neighborhood V U
of x on which (F )(E1 , . . . , En ) > 0. Therefore V D by Theorem 247, and D is
open. A similar argument shows that M \ D is open, so D = or D = M since M is
connected.

Theorem 254. [Problem 15-3] Suppose n 1, and let : Sn Sn be the antipodal
map: (x) = x. Then is orientation-preserving if and only if n is odd.
Proof. The outward unit normal vector field along Sn is N = xi /xi , and Corollary
15.34 shows that the Riemannian volume form ge of Sn is
!
n+1
X
ci dxn+1 .
ge = n (N y(dx1 dxn+1 )) = n
(1)i1 xi dx1 dx
S

i=1

103

We have

ge =

Sn

= (1)

n+1
X
\i ) (dxn+1 )
(1)i1 (xi ) (dx1 ) (dx

i=1
n+1

ge,

so F is orientation-preserving if and only if n is odd.

Theorem 255. [Problem 15-5] Let M be a smooth manifold with or without boundary.
The total spaces of T M and T M are orientable.
Proof. Let : T M M be the projection. Let (U, ) and (V, ) be smooth charts
e be the corresponding charts for T M . The
for M , and let ( 1 (U ), )
e and ( 1 (V ), )
transition map is given by


e
xn
e
x1
j
j
1
1
n 1
n
1
n
e
(
e )(x , . . . , x , v , . . . , v ) = x
e (x), . . . , x
e (x), j (x)v , . . . , j (x)v .
x
x
The Jacobian matrix of this map at (x1 , . . . , xn , v 1 , . . . , v n ) is

 exi
(x)
0
j
x
,
e
xi

(x)
xj
which clearly has positive determinant. The result follows from Proposition 15.6. A
similar computation holds for T M .

Theorem 256. [Problem 15-7] Suppose M is an oriented Riemannian manifold with
or without boundary, and S M is an oriented smooth hypersurface with or without
boundary. There is a unique smooth unit normal vector field along S that determines
the given orientation of S.
Proof. Let n = dim M , let be an orientation form for M , and let O be the orientation
on S. Let N S be the normal bundle of S (see Theorem 213), which is of rank 1. Let Z
be the set of all zero vectors in N S. Define a map f : N S \ Z {1, +1} as follows:
if v Np S then choose an oriented (with respect to O) basis (Ei ) for Tp S and define
f (v) =

(v, E1 , . . . , En1 )
.
|(v, E1 , . . . , En1 )|

This map is well-defined because (Ei ) is oriented, and is continuous because there is
an oriented local frame defined on a neighborhood of every p. Let P = f 1 ({+1}). It
is easy to see that P is convex and open in N S, so Theorem 217 shows that there is
e = X/ |X| is the desired smooth unit
a smooth global section X : S P . Then X
g
normal vector field along S.


104

Theorem 257. [Problem 15-8] Suppose M is an orientable Riemannian manifold, and


S M is an immersed or embedded submanifold with or without boundary.
(1) If S has trivial normal bundle, then S is orientable.
(2) If S is an orientable hypersurface, then S has trivial normal bundle.
Proof. If S has trivial normal bundle then there is a global frame for N S, and Proposition 15.21 shows that S is orientable. If S is an orientable hypersurface, then Theorem
256 gives a global frame for N S (since N S is of rank 1).

Theorem 258. [Problem 15-9] Let S be an oriented, embedded, 2-dimensional submanifold with boundary in R3 , and let C = S with the induced orientation. By Theorem
256, there is a unique smooth unit normal vector field N on S that determines the orientation. Let T be the oriented unit tangent vector field on C, and let V be the unique
unit vector field tangent to S along C that is orthogonal to T and inward-pointing. Then
(Tp , Vp , Np ) is an oriented orthonormal basis for R3 at each p C.
Proof. It is clear that (Tp , Vp , Np ) is an orthonormal basis, so it remains to check that
(Tp , Vp , Np ) is oriented. This is true if and only if (Np , Tp , Vp ) is oriented, and by the
definition of the induced orientation, this is true if and only if (Tp , Vp ) is an oriented
basis for Tp S. This is true since (Vp , Tp ) is an oriented basis for Tp S.

Theorem 259. [Problem 15-10] Let M be a connected nonorientable smooth manifold
c M be its orientation covering. If X is
with or without boundary, and let
b : M
any oriented smooth manifold with or without boundary, and F : X M is any local
diffeomorphism, then there exists a unique orientation-preserving local diffeomorphism
c such that
Fb : X M
b Fb = F .
Proof. Define Fb(x) = (F (x), OF (x) ), where OF (x) is the orientation of TF (x) M deterbO be an element
mined by F and the orientation of X. It is clear that
b Fb = F . Let U
1 b
b
of the basis defined in Proposition 15.40. If U is connected, then F (UO ) is either U or
bO ) is open for any open set U , and Fb is continuous. By Theoempty. Therefore Fb1 (U
rem 41, Fb is smooth, and it is easy to check that it is orientation-preserving and a local
diffeomorphism. Uniqueness follows from the fact that Fb is orientation-preserving. 
Theorem 260. [Problem 15-11] Let M be a nonorientable connected smooth manifold
c M be its orientation covering. If M
f
with or without boundary, and let
b : M
is an oriented smooth manifold with or without boundary that admits a two-sheeted
f M , then there exists a unique orientation-preserving
smooth covering map
e : M
fM
c such that
diffeomorphism : M
b=
e.

105

Proof. By Theorem 259, there is a unique orientation-preserving local diffeomorphism


fM
c such that
:M
b =
e. Similarly, there is a unique orientation-preserving
c
f
local diffeomorphism : M M such that
e =
b. Then
b =
e =
b, so
= IdM
by
uniqueness.
Similarly,

=
Id
.
Therefore

is
a
diffeomorphism.

c
f
M
Theorem 261. [Problem 15-22] Every orientation-reversing diffeomorphism of R has
a fixed point.
Proof. Let f : R R be such a diffeomorphism; then f 0 (x) < 0 for all x R. Let
g(x) = f (x) x; then g 0 (x) = f 0 (x) 1 < 1 for all x R. If g(0) = 0, then we are
done. If g(0) > 0 then by the mean value theorem, there exists a point t (0, g(0))
such that
g(g(0)) g(0)
= g 0 (t) < 1,
g(0)
i.e. g(g(0)) < 0, and the intermediate value theorem shows that g(x) = 0 for some
x (0, g(0)). Similarly, if g(0) < 0 then there exists a point t (g(0), 0) such that
g(g(0)) g(0)
= g 0 (t) < 1,
g(0)
i.e. g(g(0)) > 0, and the intermediate value theorem shows that g(x) = 0 for some
x (g(0), 0).


Chapter 16. Integration on Manifolds


Theorem 262. [Exercise 16.29] Suppose (M, g) is an oriented Riemannian manifold
and f : M R is continuous and compactly supported. Then





f dVg
|f | dVg .

M

Proof. If f is supported in the domain of a single oriented smooth chart (U, ), then



q



1
n

=

f
dV
f
(x)
det(g
)
dx

dx
g
ij



M
(U )

|f (x)| det(gij ) dx1 dxn


(U )

=
|f | dVg .
M

106

For the general case, cover supp f by finitely many domains of positively or negatively
oriented smooth charts {Ui } and choose a subordinate smooth partition of unity {i }.
Then



X




i f dVg
f dVg =



M
M
i

i |f | dVg
M

|f | dVg .

=
M


Example 263. [Problem 16-2] Let T2 = S1 S1 R4 denote the 2-torus, defined as the
set of points (w, x, y, z) such that w2 + x2 = y 2 + z 2 = 1, with the product orientation
determined by the standard orientation on S1 . Compute T2 , where is the following
2-form on R4 :
= xyz dw dy.
Let D = (0, 2) (0, 2) and define an orientation-preserving diffeomorphism F : D
T2 by
F (, ) = (cos , sin , cos , sin )
so that
dw = sin d, dx = cos d,
dy = sin d, dz = cos d.
Then

sin cos sin ( sin d) ( sin d)

T2

T2

sin2 cos sin2 d d


2
T 2
  2

2
2
=
sin d
cos sin d

=0
= 0.
Theorem 264. [Problem 16-3] Suppose E and M are smooth n-manifolds with or
without boundary, and : E M is a smooth k-sheeted covering map or generalized
covering map.

107

(1) If E and M are oriented and is orientation-preserving, then E = k M


supported n-form on M .
for any compactly

(2) E = k M whenever is a compactly supported density on M .


Proof. Let {Ui } be a finite open cover of supp by evenly covered sets, and let {i } be a
(1)
(k)
subordinate smooth
of unity. For each i, let Ui , . . . , Ui be the components
o
n partition
(j)
be a smooth partition of unity subordinate to the open cover
of 1 (Ui ). Let bi
o
n
(j)
of supp . Applying Proposition 16.6(d), we have
Ui

k
XX

j=1

(j)
Ui

j=1

(j)
Ui

k
XX
i

Ui

k
XX
i

j=1

k
XX

Ui

XX
k
E

j=1

i0

j 0 =1

XX
k
E

i0

(i |U (j) )
i

k
XX
(j)

(i |U (j) )

i0

(j 0 )
bi0

(j 0 )
bi0 (i |U (j) )
i

j 0 =1

k
XX
(i |U (j) )
i

j=1

(j 0 )
bi0

j 0 =1

=
E

This proves (1). Part (2) is similar.

Theorem 265. [Problem 16-4] If M is an oriented compact smooth manifold with


boundary, then there does not exist a retraction of M onto its boundary.
Proof. Suppose that there exists such a retraction r : M M . By Theorem 6.26, we
can assume that r is smooth. Let : M , M be the inclusion map. Let be an
orientation form for M . By Theorem 16.11 and Proposition 14.26,

r =
d(r ) =
r (d) = 0
M

108

since is a top-degree form. By definition,


r =
r =
M

(r ) =

,
M

so

= 0.
M

This contradicts Proposition 16.6(c).

Theorem 266. [Problem 16-5] Suppose M and N are oriented, compact, connected,
smooth manifolds, and F, G : M N are homotopic diffeomorphisms. Then F and G
are either both orientation-preserving or both orientation-reversing.
Proof. Theorem 253 shows that F and G are individually orientation-preserving or
orientation-reversing. By Theorem 6.29, there is a smooth homotopy H : M I N
from F to G. Let be a positively oriented orientation form for N . By Theorem 16.11,

H (d) = 0.
d(H ) =
H =
But

M I

M I

(M I)

H =
(M I)

G ,

so

G .

F =
M

The result follows from Proposition 16.6(d).

Theorem 267. [Problem 16-6] The following are equivalent:


(1) There exists a nowhere-vanishing vector field on Sn .
(2) There exists a continuous map V : Sn Sn satisfying V (x) x (with respect
to the Euclidean dot product on Rn+1 ) for all x Sn .
(3) The antipodal map : Sn Sn is homotopic to IdSn .
(4) The antipodal map : Sn Sn is orientation-preserving.
(5) n is odd.
Therefore, there exists a nowhere-vanishing vector field on Sn if and only if n is odd.
Proof. Suppose X is a nowhere-vanishing vector field on Sn . Then taking V = X/ |X|
proves (1) (2). Suppose (2) holds. We can define a homotopy H1 : Sn I Sn from
to V by
(1 t)(x) + tV (x)
H1 (x, t) =
,
|(1 t)(x) + tV (x)|

109

where the denominator is nonzero since (1 t)(x) + tV (x) = 0 contradicts the fact
that V (x) x. Similarly, we can define a homotopy H2 from V to IdSn by
(1 t)V (x) + tx
.
|(1 t)V (x) + tx|

H2 (x, t) =

This proves (2) (3). Theorem 266 proves (3) (4), and Theorem 254 proves
(4) (5). Finally, Theorem 150 proves (5) (1).

Theorem 268. [Problem 16-9] Let be the (n 1)-form on Rn \ {0} defined by
n

= |x|

n
X

ci dxn .
(1)i1 xi dx1 dx

i=1

(1) Sn1 is the Riemannian volume form of Sn1 with respect to the round metric
and the standard orientation.
(2) is closed but not exact on Rn \ {0}.
Proof. The unit normal vector field for Sn1 with the standard orientation is given by
x 7 x, so (1) follows from the formula in Lemma 14.13. For (2), we have
 i 
n
X
x
i1
ci dxn
d =
(1) d
dx1 dx
n
|x|
i=1
!
n
X
|x|n n(xi )2 |x|n2
dx1 dxn
=
2n
|x|
i=1

n 
X
1
n(xi )2
dx1 dxn
=
n
n+2
|x|
|x|
i=1
= 0.
If is exact then

=0
Sn1

by Corollary 16.13, which is a contradiction.

Example 269. [Problem 16-10] Let D denote the torus of revolution in R3 obtained
by revolving the circle (r 2)2 + z 2 = 1 around the z-axis, with its induced Riemannian
metric and with the orientation determined by the outward unit normal.
(1) Compute the surface area of D.
(2) Compute the integral over D of the function f (x, y, z) = z 2 + 1.
(3) Compute the integral over D of the 2-form = z dx dy.

110

Let : D , R3 be the inclusion map. Let U = (0, 2) (0, 2) and define an


orientation-preserving diffeomorphism F : U D by
F (, ) = ((2 + cos ) cos , (2 + cos ) sin , sin ).
We have
dx = sin cos d (2 + cos ) sin d,
dy = sin sin d + (2 + cos ) cos d,
dz = cos d
and
F g = dx2 + dy 2 + dz 2
= sin2 cos2 d2 + (2 + cos )2 sin2 d2
+ 2 sin sin cos (2 + cos ) d d
+ sin2 sin2 d2 + (2 + cos )2 cos2 d2
2 sin sin cos (2 + cos ) d d
+ cos2 d2
= d2 + (2 + cos )2 d2 .
For (1), we have

p
(2 + cos )2

g =
D

(,)U
2

(2 + cos ) d

= 2
0

= 8 2 .
For (2), we have

(sin2 + 1)

f g =
D

p
(2 + cos )2

(,)U
2

(sin2 + 1)(2 + cos ) d

= 2
0
2

= 12 .
For (3), we have
dx dy = ( sin cos d (2 + cos ) sin d)
( sin sin d + (2 + cos ) cos d)
= (2 + cos ) sin d d

111

so that

(2 + cos ) sin2 d d
D
2
(2 + cos ) sin2 d
= 2

z dx dy =
D

0
2

= 4 .
Theorem 270. [Problem 16-11] Let (M, g) be a Riemannian n-manifold with or without
boundary. In any smooth local coordinates (xi ) we have



1
 ip
i

div X
=
X
det
g
,
xi
det g xi
where det g = det(gkl ) is the determinant of the component matrix of g in these coordinates.
Proof. We have
g =

p
det gdx1 dxn

by Proposition 15.31, so

 X
n
p
i
i1 i
ci dxn .
det g dx1 dx
X
=
(1)
X
i
x
i=1
Then
 


 ip
i
d X
X
=
det
g
dx1 dxn
xi
xi
and


div X
xi
i


  
i
=
d X
xi

1
 ip
X
det
g
.
=
det g xi
1


Theorem 271. [Problem 16-12] Let (M, g) be a compact Riemannian manifold with
boundary, let ge denote the induced Riemannian metric on M , and let N be the outward
unit normal vector field along M .
(1) The divergence operator satisfies the following product rule for f C (M ),
X X(M ):
div(f X) = f div X + hgrad f, Xig .

112

(2) We have the following integration by parts formula:

hgrad f, Xig dVg =


f hX, N ig dVge
(f div X) dVg .
M

Proof. Theorem 270 shows that in coordinates, we have



1
 ip
div(f X) =
det
g
f
X
det g xi


 f
p
1
 ip
i
=
f i X det g + i X det g
x
x
det g


p

f
1
X i det g + i X i
= f
i
x
det g x
= f div X + hgrad f, Xig .
Therefore

hgrad f, Xig dVg =

div(f X) dVg

=
M

f div X dVg

f div X dVg
f hX, N ig dVge
M

by Theorem 16.32.

Theorem 272. [Problem 16-13] Let (M, g) be a Riemannian n-manifold with or without
boundary. The linear operator 4 : C (M ) C (M ) defined by 4u = div(grad u)
is called the (geometric) Laplacian. The Laplacian is given in any smooth local
coordinates by


p
1
u

ij
4u =
det g j .
g
x
det g xi
n
On R with the Euclidean metric and standard coordinates,
n
X
2u
4u =
.
(xi )2
i=1

Proof. Obvious from Theorem 270.

Theorem 273. [Problem 16-14] Let (M, g) be a Riemannian n-manifold with or without
boundary. A function u C (M ) is said to be harmonic if 4u = 0.
(1) If M is compact, then

u4v dVg =
hgrad u, grad vig dVg
M

uN v dVge,

113

(u4v v4u) dVg =


M

(vN u uN v)dVge,

where N and ge are as in Theorem 271. These are known as Greens identities.
(2) If M is compact and connected and M = , the only harmonic functions on
M are the constants.
(3) If M is compact and connected, M 6= , and u, v are harmonic functions on
M whose restrictions to M agree, then u = v.
Proof. (1) follows by putting f = u and X = grad v in Theorem 271. For (2), if u is
harmonic then

2
|grad u|g dVg =
u4u dVg = 0
M

by part (1). Therefore grad u = 0, and u is constant since M is connected. For (3), we
have

2
(u v)N (u v) dVge = 0
|grad(u v)|g dVg =
M

by part (1). Therefore grad(u v) = 0, and u v is constant since M is connected.


Since u and v agree on M , we have u = v.

Theorem 274. [Problem 16-15] Let (M, g) be a compact connected Riemannian manifold without boundary, and let 4 be its geometric Laplacian. A real number is called
an eigenvalue of 4 if there exists a smooth real-valued function u on M , not identically
zero, such that 4u = u. In this case, u is called an eigenfunction corresponding to
.
(1) 0 is an eigenvalue of 4, and all other eigenvalues are strictly positive.
(2) If u and v are eigenfunctions corresponding to distinct eigenvalues, then M uv dVg =
0.
Proof. Any constant function is an eigenfunction with eigenvalue 0. If 4u = u and u
is not constant then Theorem 273 shows that

2
0<
|grad u|g dVg =
u4u dVg =
u2 dVg ,
M

so > 0. Suppose 4u = u and 4v = v where 6= . By Theorem 273,

0=
(u4v v4u) dVg = ( )
uv dVg ,
M

so

uv dVg = 0
M

since 6= 0.

114

Theorem 275. [Problem 16-16] Let M be a compact connected Riemannian n-manifold


with nonempty boundary. A number R is called a Dirichlet eigenvalue for M
if there exists a smooth real-valued function u on M , not identically zero, such that
4u = u and u|M = 0. Similarly, is called a Neumann eigenvalue if there exists
such a u satisfying 4u = u and N u|M = 0, where N is the outward unit normal.
(1) Every Dirichlet eigenvalue is strictly positive.
(2) 0 is a Neumann eigenvalue, and all other Neumann eigenvalues are strictly
positive.
Proof. As in Theorem 274.

Theorem 276. [Problem 16-17] Suppose M is a compact connected Riemannian nmanifold with nonempty
boundary. A function u C (M ) is harmonic if and only

if it minimizes M |grad u|2g dVg among all smooth functions with the same boundary
values.
Proof. Suppose f C (M ) vanishes on M . For all , we have

2
2
hgrad u, grad f ig dVg
|grad u|g dVg + 2
|grad(u + f )|g dVg =
M
M
M

2
|grad f |2g dVg .
+
M

Applying Theorem 271, we have

2
2
(f 4u) dVg
|grad u|g dVg + 2
|grad(u + f )|g dVg =
M
M
M

2
+
|grad f |2g dVg .
M

If u is harmonic then

2
2
|grad(u + f )|g dVg =
|grad u|g dVg +
|grad f |2g dVg ,
M

|grad u|2g
M

and it is clear that u minimizes


dVg among all smooth functions with the

same boundary values. Conversely, suppose that u minimizes M |grad u|2g dVg . Then

|grad(u + f )|2g dVg


M

has a global minimum at = 0, and its derivative (with respect to )

2
(f 4u) dVg + 2
|grad f |2g dVg
M

115

must be zero at = 0. That is,

(f 4u) dVg = 0
M

for all f C (M ) such that f |M = 0. Therefore 4u = 0.

Theorem 277. [Problem 16-18] Let (M, g) be an oriented Riemannian n-manifold.


(1) For each k = 1, . . . , n, g determines a unique inner product on k (Tp M ) (denoted by h, ig , just like the inner product on Tp M ) satisfying


1


k , 1 k g = det ( i )# , ( j )# g
whenever 1 , . . . , k , 1 , . . . , k are covectors at p.
(2) The Riemannian volume form dVg is the unique positively oriented n-form that
has unit norm with respect to this inner product.
(3) For each k = 0, . . . , n, there is a unique smooth bundle homomorphism :
k T M nk T M satisfying
= h, ig dVg
for all smooth k-forms , . (For k = 0, interpret the inner product as ordinary
multiplication.) This map is called the Hodge star operator.
(4) : 0 T M n T M is given by f = f dVg .
(5) = (1)k(nk) if is a k-form.
Proof. For (1), any such inner product must satisfy



i1 # j1 #
( ) , ( ) g (i1 )# , (jk )# g

I J

..
..
..
, g = det

.
.
.

i # j #

i # j #
1
k
k
k
( ) , ( ) g ( ) , ( ) g
i1 j1

g
g i1 jk
..
...
= det ...
.
ik j1
ik jk
g
g
(*)

= JI

for all increasing multi-indices I and J, whenever (i ) is the coframe dual to a local
orthonormal frame (Ei ). (Here (g ij ) is the inverse of gij (p), which is the matrix of g
in these coordinates.) This proves that the inner product is uniquely determined. To
prove existence, we will define h, ig by (*) and extend bilinearly. To check that the

116

definition is independent of the choice of orthonormal frame, let (e


i ) be another coframe
ei ). Then
dual to a local orthonormal frame (E
I

e =

0
X

and e =

0
X

M M

where L = eI (El1 , . . . , Elk ) and M = eJ (Em1 , . . . , Emk ), so


0 X
0
X

I J


e , e g =
L M L , M g

=
=
=

L M
0 X
0
X

L
L M M

L M
0
X

L L

L
0
X

eI (El1 , . . . , Elk )e
J (El1 , . . . , Elk ).

ej for some orthonormal matrix (Aj ), we have


Since Ei = Aji E
i

e , e

0 X
X
L

Apl11 Aplkk PI

0 X
X

J
Aql11 Aqlkk Q

Q
i

(sgn )Al1(1) Alk(k)

(sgn )Al1 (1) Alk (k)

Sk

L Sk

=???????????????
For (2), if is any positively oriented n-form then p = c(dVg )p for some c > 0, and
hp , p ig = c2 hdVg , dVg ig


= c2 1 n , 1 n g
= c2
whenever (i ) is the coframe dual to a local orthonormal frame, by Proposition 15.29.
If has unit norm then it is clear that c = 1.

Example 278. [Problem 16-19] Consider Rn as a Riemannian manifold with the Euclidean metric and the standard orientation.
(1) Calculate dxi for i = 1, . . . , n.
(2) Calculate (dxi dxj ) in the case n = 4.

117

TODO
Theorem 279. [Problem 16-20] Let M be an oriented Riemannian 4-manifold. A
2-form on M is said to be self-dual if = , and anti-self-dual if = .
(1) Every 2-form on M can be written uniquely as a sum of a self-dual form and
an anti-self-dual form.
(2) On M = R4 with the Euclidean metric, the self-dual and anti-self-dual forms in
standard coordinates are given by:
??????????
Proof. TODO

Theorem 280. [Problem 16-21] Let (M, g) be an oriented Riemannian manifold and
X X(M ). Then
XydVg = X [ ,
div X = d X [ ,
and, when dim M = 3,
curl X = (dX [ )] .
Proof. TODO

Theorem 281. [Problem 16-22] Let (M, g) be a compact, oriented Riemannian nmanifold. For 1 k n, define a map d : k (M ) k1 (M ) by d = (1)n(k+1)+1
d , where is the Hodge star operator defined in Theorem 277. Extend this definition
to 0-forms by defining d = 0 for 0 (M ).
(1) d d = 0.
(2) The formula

h, ig dVg

(, ) =
M

defines an inner product on k (M ) for each k, where h, ig is the pointwise inner


product on forms defined in Theorem 277.
(3) (d , ) = (, d) for all k (M ) and k1 (M ).
Proof. TODO

118

Chapter 17. De Rham Cohomology


Theorem 282. [Exercise 17.37] Suppose M , N , and P are compact, connected, oriented, smooth n-manifolds.
(1) If F : M N and G : N P are both smooth maps, then deg(G F ) =
(deg G)(deg F ).
(2) If F : M N is a diffeomorphism, then deg F = +1 if F is orientationpreserving and 1 if it is orientation-reversing.
(3) If two smooth maps F0 , F1 : M N are homotopic, then they have the same
degree.
Proof. For every smooth n-form on P , we have

(G F ) =
F G
M
M

G
= (deg F )
N

= (deg F )(deg G)

.
P

This proves (1). Part (2) follows from Proposition 16.6(d). If F0 and F1 are homotopic
then Proposition 17.10 shows that the induced cohomology maps F0 and F1 are equal.
For any smooth n-form on N , we have F0 F1 = d for some smooth (n 1)-form
. Then

F0
F1 =
d = 0.
M


Theorem 283. [Problem 17-1] Let M be a smooth manifold with or without boundary,
and let p (M ), q (M ) be closed forms. The de Rham cohomology class of
depends only on the cohomology classes of and , and thus there is a well-defined
p
q
p+q
bilinear map `: HdR
(M ) HdR
(M ) HdR
(M ), called the cup product, given by
[] ` [] = [ ].
Proof. TODO


References

[1] John M. Lee. Introduction to Topological Manifolds. Springer, 2nd edition, 2011.

Potrebbero piacerti anche